Download PGI PG PGI 2016 November Solved Question Paper

Download PGIMER (Post Graduate Institute of Medical Education & Research, Chandigarh) PGI 2016 November Solved Question Paper

1.Posteriorinterosseousnervesuppliesall
except:
a)Extensorcarpiradialislongus
b)Extensorcarpiulnaris
c)Extensordigitorum
d)Extensorindices
e)Flexorcarpiulnaris
CorrectAnswer-A:E
Ans.(a)Extensorcarpiradialislongus,(e)Flexorcarpiulnaris
Theposteriorinterosseousnerveisapuremotornerve
andinnervatessupinator&extensorcarpiradialis.
Itsupplies:?
1. Extensorcarpiulnaris
2. Extensordigitorum
3. Extensordigitiminimi
4. Abductorpollicislongus
5. Extensorpollicislongusandbrevis
6. Extensorindicis

2.Muscle'sofanteriorcompartmentoflegis/
are:
a)Peroneustertius
b)Peroneusbrevis
c)Peroneuslongus
d)Flexordigitorumlongus
e)Flexorhallucislongus
CorrectAnswer-A
Ans.(a)Peroneustertius
The4musclesintheanteriorcompartmentofthelegare-thetibialis
anterior,extensordigitorumlongus,extensorhallucislongus,and
fibularis(Peroneus)tertius



3.Branchesofinternalcarotidarterydirectly
arisingfromit:
a)Posteriorcommunicatingartery
b)Superiorhypophysealartery
c)Inferiorhypophysealartery
d)PosteriorcerebralArtery
e)RecurrentarteryofHeubner
CorrectAnswer-A:B:C
Ans.(a)Posteriorcommunicatingartery,(b),Superior
hypophysealartery,(c)Inferiorhypophysealartery
InternalCarotidArteryBranches:
Ophthalmicartery
Posteriorcommunicatingartery
Anteriorchoroidalartery
Anteriorcerebralartery:Orbital;FrontalandParietalbranches
Middlecerebralartery:Deeporperforatingbranch;temporalbranch;
FrontalbranchandParietalbranches



4.Allaretrueaboutvestibularnerveexcept:
a)Ithastwodivision-superiorandinferiorvestibular
b)Vestibularnucleisituatedatjunctionofponsandmedulla
c)Nervefibresrelayatscarpa'sganglion
d)Nucleusliesinmidbrainnearaqueduct
e)None
CorrectAnswer-D
Ans.(d)Nucleusliesinmidbrainnearaqueduct
Vestibularnerve:
Themainnervedividesatandwithinthevestibular(Scarpa's)
ganglionintosuperiorandinferiordivision,whichareconnectedby
anisthmus
Vestibularnucleiislocatedinfloorof4thventricle&issuppliedby
PICA.
Vestibularganglionisalsok/aScarpa'sganglion.
Vestibularnerveanastomosewithcochlearandfacialnerve.

5.Trueabouttrochlearnerve:
a)Arisefromventralaspectofbrainstem
b)EntersorbitthroughannulusofZinn
c)Lesioncausesdiplopia
d)Nucleusofthetrochlearnerveislocatedinthecaudal
mesencephalonbeneaththecerebralaqueduct
e)Damagecausesipsilateralpalsyofsuperiorobliquemuscle
CorrectAnswer-C:D
Ans.(c)Lesioncausesdiplopia,(d)Nucleusofthetrochlear
nerveislocatedinthecaudalmesencephalon
Thetrochlearnervehascertainuniquefeatures:
Itistheonlycranialnervewhosefibersoriginatetotallyfromthe
contralateralnucleus.
Itistheonlycranialnervetoemergefromthedorsalsurfaceofthe
brainstem.
Itisthemostslenderofallthecranialnerves.
Ithasthelongestintraduralcourseamongthethreeextraocular
motornerves.
Itsuppliesonlyonemusclei.e.superioroblique(Abducentcranial
nervealsosuppliesonlyonemusclei.e.Lateralrectus).

6.Content(s)ofaortichiatus?
a)Thoracicduct
b)Aorta
c)Vagusnerve
d)Inferiorvenacava
e)Azygosvein
CorrectAnswer-A:B:E
Ans.(a)Thoracicduct,(b)Aorta,(e)Azygosvein
TheaortichiatussituatedatthelevelofT12vertebra.
Structurespassingthroughaortichiatusalongwithaortaare:
Thoracicduct
Azygosvein
Hemiazygosvein

7.Whichflexortendonzoneinhandisknown
asNoman'sland?
a)ZoneI
b)ZoneII
c)ZoneIII
d)ZoneIV
e)ZoneV
CorrectAnswer-B
Ans.(b)ZoneII
ZoneIIextendsfromthemiddleofthemiddlephalanxtodistal
palmarcrease.Itcontainsbothflexortendonsuperficialisandflexor
tendonprofundus.
IthasbeencalledNoMan'sLand.'or"NoMan'sZonebecause
repairinthiszoneisverydifficult.

8.Correctstatementaboutmeiosis:
a)Somaticcellsnotdividebymeiosisbecausenumberof
chromosomesreducestohalf
b)Occuringermcellwhichresultinhaploidcells
c)Onespermatocyteproducesonespermandoneoocyte
producesoneovum
d)Germcellundergoesdivisiontoformdiploidcellandincrease
theirnumber
e)Bodyneedsmeiosistoproducelargeno.ofeggsandsperms
CorrectAnswer-A:B:E
Ans.(a)Somaticcellsnotdividebymeiosis(b)Occuringerm
cellwhichresult(e)Bodyneedsmeiosistoproduce.
Meiosis:
Meiosisisatypeofcelldivisionthatreducesthenumberof
chromosomeslntheparentcellbyhalfandproducesfourgamete
cells.
Thisprocessisrequiredtoproduceeggand,spermcellforsexual
reproduction
Meiosisbeginswithaparentcellthatlsdiploidandformsfour
daughtercellsthatarehaploid,whichhavehalfthenumberof
chromosomesofthediploidcells.

9.


Cross-sectionofmedullaatthelevelofmid-
olivarysectionthroughtheflooroffourth
ventriclecontainswhichofthefollowing
structure?

a)Trapezoidbody
b)Dorsalnucleusofvagus
c)Nucleusoftractussolitarius
d)Nucleusambigus
e)Superiorvestibularnucleus
CorrectAnswer-B:C:D
Ans.(b)Dorsalnucleusofvagus,(c)Nucleusoftractussolitarius,
(d)Nucleusambiguus

10.Truestateinent(s)aboutOlfactorysystem
:
a)Olfactorymucosacoverupper1/3ofnasalcavity
b)Olfactorypathwaypassesviathalamustoorbitofrontalcortex
c)Adaptationtoodourdeveloponlyafter1-2minutes
d)OlfactoryreceptorsactviacAMP
e)Rateofolfactorynerveimpulseschangeapproximatelyin
proportiontothelogarithmofstimulusstrength
CorrectAnswer-A:B:D:E
Ans.(A)Olfactorymucosacoverupper1/3ofnasalcavity
(B)Olfactorypathwaypassesviathalamustoorbitofrontal
cortex(D)OlfactoryreceptorsactviacAMP(E)Rateofolfactory
nerveimpulseschangeapproximatelyinproportiontothe
logarithmofstimulusstrength
Rateofolfactorynerveimpulseschangeapproximatelyinproportion
tothelogarithmoFstimulusstrength.
Theolfactoryreceptorsadaptabout5O%inthefirstsecondorso
afterstlmulatlan.Thereafter,theyadaptverylittleandveryslowly.
Adaptation:Itdevelopswithinsecondsonminutes,dependingon
thenatureofthesubstance.
Weber-FechnerLawstatesthatthesubjectivesensation(ofodor,
soundorlightintensity)isproportionaltothelogarithmof!the
stimulusintensity''
ThereceptorintheolfactorymucousmembranearecoupledtoG-
proteins.
Olfactoryregions:theUpper1/3oflateralwalls(uptosuperior

concha),correspondingpartofthenasalseptumandtheroofofthe
nasalcavityfromtheolfactoryregion'Here,mucousmembraneIs
palerincolor

11.Trueaboutspecialanatomyand
Physiologyoflung:
a)Surfactantpreventcollapseofsmallalveoliintolargerone
b)Largeralveolihasmoretendencytocollapsethansmaller
alveoliinabsenceofsurfactant
c)Surfactantdecreasechanceofcollapse
d)Surfactantincreasessurfacetension
e)Withsurfactant,largealveolitendtobecomesmallerand
smalleronestendtobecomelarger
CorrectAnswer-A:C:E
Ans.(A)Surfactantpreventcollapseofsmallalveoliintolarger
one(C)Surfactantdecreasechanceofcollapse(E)With
surfactant,largealveolitendtobecomesmallerandsmaller
onestendtobecomelarger
Upperregionalveolihavelargervolumes.
Alreadyfilledwithairandarelesscompliantcomparedtothoseto
dependentregions
LowsurFacetensionalveoliaresmall-duetothepresenceinthe
fluidliningthealveoliofsurfactant,alipidsurface-tension-lowering
agent.
Surfactantdeficiencyisanimportantcauseofinfantrespiratory
distresssyndrome(IRDS,alsoknownashyalinemembrane
disease.
Surfacetensioninthelungsoftheseinfantsishigher,andthe
alveoliarecollapsedlnmatryareas(atelectasis).

12.Allaretrueaboutacromegalyexcept:
a)IncreasedIGF-1levels
b)Excessivegrowthoccursbeforefusionoftheepiphysesofthe
longbones
c)Somatostatinanaloguescanbeused
d)Growthhormonelevelsincreased
e)Transsphenoidalsurgicalresectionisthepreferredprimary
treatmentforpituitaryadenoma
CorrectAnswer-B
Ans.B.Excessivegrowthoccursbeforefusionofthe
epiphysesofthelongbones
Inacromegaly,IGF-IlevelsareinvariablyhighandreflectaLog-
LinearrelationshipwithclrculatlngGHconcentrations.
Foracromegaly,somatostatinanaloguesandGHreceptor
antagonistsareindicated
Age-matchedserumIGF-Ilevelsareelevatedinacromegaly.
Somatostatinanaloguesareusedasadjuvanttreatmentfor
preoperativeshrinkageoflargeinvasivemacroadenomas.
Transsphenoidalsurgicalresectionbyanexperienced.surgeonis
thepreferredprimarytreatment.
Tumorsofthesomatotrophsoftheanteriorpituitary(pituitary
adenomas)secretelargeamountsofgrowthhormone,leadingto
glgantbmlnchlUrenandacromegalyinadults.
Hypersecretionofgrowthhormoneisaccompaniedby
hypersecretionofprolactinin20-40%ofpatientswithacromegaly.

13.Truestatement(S)is/are:
a)Vasopressinincreaseonlywaterreabsorption,notsolute
reabsorption
b)AldosteroneincreaseNa+reabsorptionfromtubules
c)GlomerularfiltrateofPCThassimilarosmolarityasofplasma
d)UrineishyperosmolarinearlyDCT
e)Generallyurineosmolarityequalstoplasmaosmolarity
CorrectAnswer-A:B:C
Ans(A)Vasopressinincreaseonlywaterreabsorption,not
solutereabsorption(B)AldosteroneincreaseNa+reabsorption
fromtubules(C)GlomerularfiltrateofPCThassimilar
osmolarityasofplasma
Antidiuretichormone(ADH,Vasopressin)increasespermeabilityof
distaltubules(mildaction)andcollectingducts(mainly)to
water)increaseswaterreabsorption.
Aldosteronecausesretentlonofsodlumtromthekidneyand
increasedurinaryexcretionofpotassium;ithaslittleeffectonwater
excretion.
PCT:Theosmolalityoffluidstntubuleisunchangedatapprox.
3(X)mosmie,isotonicityismaintained.
ThetubularfluidenteringtheDCTisalwayshypotonictoplasma.
ThefluidinthedescendinglimboftheloopofHenlebecomes
hypertonicaswatermovesoutofthetubuleintothehypertonic
interstitium.
Intheascendinglimbltbecomesmoredilutebecauseofthe
movementofNa+andCl-outofthetubularlumen,and.whenfluid
reachesthetopoftheascendinglt.

AdrenalmineralocorticoidssuchasaLDosteroneincreasedtubular
reabsorptionofNa+inassociationwithsecretionofK+andH+and
alsoNa+reabsorptionwithCl-.

14.Rapidlyadaptingreceptor(s)is/are:
a)Painreceptor
b)Paciniancorpuscles
c)Musclespindle
d)Golgitendonorgans
e)Meissnercorpuscles
CorrectAnswer-B:D:E
Ans.(B)Paciniancorpuscles(D)Golgitendonorgans
(E)Meissnercorpuscles
Rapidlyadapting:Rapidlyadaptingmechanoreceptorsinclude
Meissnercorpuscleend-organs,Paciniancorpuscleend-organs,
hairfolliclesreceptorsandsomefreenerveendings.
Merkel'sdiscsandMelssner'scorpusclesaretactilereceptors.
Theyarerapidlyadaptingreceptors.
Paciniancorpuscles:Theyrespondtodeformationcausedbyfirm
pressureandarequietlyadapting.

15.Whichofthefollowingis/aretrueabout
normallevel:
a)TotalCalcium:8.5-10.5mg/dL
b)Sodium:135-145mmol/L
c)Potassium:3.5-5.1mmol/L
d)Creatinine:0.6-2.6mg/dL
e)TSHlevel:0.1-3.1mIU/L
CorrectAnswer-A:B:C
Ans.(A)TotalCalcium:8.5-10.5mg/dL(B)Sodium:135-145
mmol/L(C)Potassium:3.5-5.1mmol/L
Davidson22ed/1308,HarrisonL9rh/2762,2763,
TSH-.2-4.5mU/L
Calcium(total):8.5-10.5mg/dL
Potassium-3.5-5.0meq/L
Sodium-136-146meq/L
Serumcreatinine-0.6-1.6mg/dl

16.TrueaboutActionPotentialinskeletal
musclefibersandnervefibres:
a)Skeletalmusclefibresconductionvelocityis1/4ofthick
myelinatednervefiber
b)Actionpotentialofbothqualitativelysimilar
c)Restingmembranepotentialalmostsame
d)Durationofactionpotentialsameinboth
e)Actionpotentialofbothquantitivelysimilar
CorrectAnswer-B:C
Ans.(B)Actionpotentialofbothqualitativelysimilar
(C)Restingmembranepotentialalmostsame
MuscleActionPotential-ComparisonwithNerveActionPotential
GuytonIlth/89
Restingmembranepotential:about-80to-90millivoltstoskeletal
fibers-thesameaslnlargemyelinatednervefibers.
Durationofactionpotential1to5millisecondstnskeletalmuscle
aboutfivetimesaslongasinlargemyelinatedfibers..
Velocityofconduction:3to5m/sec-about1/13ththevelocityof
conductioninlargemyelinatedteraefibersthatexciteskeletal
muscle.

17.Tissueelevationofwhichofthefollowing
causevasoconstriction:
a)Na+
b)K+
c)Mg
d)Ca2+
e)H+
CorrectAnswer-D
Ans.D.Ca2+
VascularControlbylonsandOtherChemicalFactorsGuyton
12th(SAE)/269
Anincreaseincalciumionconcentrationcausesvasoconstriction.
Anincreaseinpotassiumionconcentration,.withinthephysiological
range,causesvasodilation.
Anincreaseinmagnesiumionconcentrationcausespowerful
vasodilation.
Anincreaseinhydrogenionconcentration(decreaseinpH)causes
dilationofthearterioles.
Anionsthathavesignificanteffectsonbloodvesselsareacetateand
citrate.AnIncreaseincarbondioxideconcentrationcausesmoderate
vasodilationinmosttissuesbutmarkedvasodilationinthebrain.

18.Whichofthefollowingstatement(s)is/are
truechangesattimeofovulation:
a)GnRHleveldecreases
b)Gonadotropinhormonesurge
c)hCGsurge
d)1`Prostaglandins
e)Activationofproteolyticenzymes
CorrectAnswer-B:D:E
Ans.B,GonadotropinhormonesurgeD,1`Prostaglandins&
E,Activationofproteolyticenzymes
Ovulation:
ThemidcycleLHsurgeisresponsibleforadramaticincreasein
localconcentrationsofprostaglandinsandproteolyticenzymesin
thefollicularwall.
Thesesubstancesprogressivelyweakenthefollicularwalland
ultimatelyallowaperforationtoform.
Ifpregnancydoesoccur,placentalhCGwillmimicLHactionand
continuallystimulatethecorpusluteumtosecreteprogesterone.
FeedbackEffects:
At36-48hbeforeovulation,theestrogenfeedbackeffectbecomes
positive,andthisinitiatestheburstofLHsecretion(LHsurge)that
producesovulation.
Ovulationoccursabout9afterLHpeak-FSHsecretionalsopeaks,
despiteasmallriseininhibin,probablybecauseof.thestrong
stimulationofgonadotropesbyGnRH.
Duringthelutealphase,thesecretionofLHandFSHislowbecause

oftheelevatedlevelsofestrogen,progesterone,andinhibin.

19.Comprehensionpreservedinwhichofthe
follwingaphasia
a)Broca'saphasia
b)Conductionaphasia
c)Wernicke'saphasia
d)Gobalaphasia
e)Anomicaphasia
CorrectAnswer-A:B:E
Ans.A,Broca'saphasiaB,Conductionaphasia&E,Anomic
aphasia

Repetition
Comprehension ofSpoken Naming
Fluency
Language
Preserved
Wernicke's
Impaired
Impaired
Impaired or
increased
Preserved
Broca's
(except
Impaired
Impaired Decreased
grammar)
Global
Impaired
Impaired
Impaired Decreased
Conduction
Preserved
Impaired
Impaired Preserved
Nonfluent
Preserved
Preserved Impaired Impaired
(motor)transcortical
Fluent(sensory)
Impaired
Preserved Impaired Preserved
transcortical
No

No
Isolation
Impaired
Echolalia Impaired purposeful
speech
Preserved
exceptfor
Anomic
Preserved
Preserved Impaired word-
finding
pauses
Impairedonlyfor
Pureword
spoken
Impaired
Preserved Preserved
deafness
language
Impairedonlyfor
Purealexia
Preserved Preserved Preserved
reading

20.Vomitingcentre(s)involvedinpost-
operativevomiting
a)Areapostrema
b)Chemoreceptortriggerzone(CTZ)
c)Reticularformationlocatedinmedulla
d)Nucleustractussolitarius
e)Basalganglia
CorrectAnswer-A:B:D
Ans.A,AreapostremaB,Chemoreceptortriggerzone(CTZ)&
D,Nucleustractussolitarius
PostoperativeNauseaandVomiting(PONV):
PONVisdefinedasanynausea,retching,orvomitingoccurring
duringthefirst24-48haftersurgeryinpatients.
Accordingtoourcurrentmodel,thebrainstructuresinvolvedinthe
pathophysiologyofvomitingaredistributedthroughoutthemedulla
oblongataofthebrainstem,notcentralized,lnananatomically
defined'vomitingcentre'.
Suchstructuresincludethechemoreceptortriggerzone(cRTZ),
Locatedatthecaudalendofthefourthventricleinthearea
postrema,andthenucleustractussolitarius(NTS),locatedin
theareapostremaandlowerpons.
PONVcanbetriggeredbyseveralperioperativestimuli,including
opioids,volatileanaesthetics,anxiety,adversedrugreactions,and
motion.

21.HGPRTareinvolve,andgoutcanbea
feature.
a)HGPRTdeficiency
b)HGPRToveractivity
c)PRPPsynthetasedeficiency
d)Glucose6-phosphatasedeficiency
e)Glucosephosphatedehydrogenasedeficiency
CorrectAnswer-A:D
Answer.(a)HGPRTdeficiency,(d)Glucose6-phosphatase
deficiency
[Ref:Harper30th/354-56;Satyanarayan4th/269-70,394-951
VonGierke'sdisease(Type1glycogenstoragedisease):
HyperuricemiaoccursduetoGlucose6-phosphataseenzyme
defect.
HGPRTdeficiency(asseeninLesch-Nyhansyndrome):Increased
productionofpurines
Glucose6-phosphatasedeficiency:Purineoverproduction.
Goutisusuallyprecededandaccompaniedbyhyperurlcemta
(plasmauricacidlevel>0.4Immol/L).
Hlperuricemiaiscausedbydacreasedrenalextretlon,lncreased
productlon"orincreasedintakeoturlcacid

22.Hyperphenylalaninemiaoccursdueto:
a)Phenylalaninehydroxylasedeficiency
b)Phenylalaninehydroxylaseoveractivity
c)Dihydrobiopterinreductasedeficiency
d)Tyrosinehydroxylasedeficiency
e)Defectindihydrobiopterinbiosynthesis
CorrectAnswer-A:C:E
Answer:(a)Phenylalaninehydroxylase...,(c)Dihydrobiopterin
reductase...,(e)Defectindihydrobiopterinbiosynthesis
Hyperphenylalaninemiasarisefromdefectsinphenylalanine
hydroxylaseitself(typeI,classicphenylketonuriaorPKU),in
dihydrobiopterinreductase(typesIIandIII),orindihydrobiopterin
biosynthesis(typesIVandV).Alternativecatabolitesareexcreted.
PKUiscausedbyadcficiencyotphenylalaninehydroxyl.ase,isthe
mostcommonclinicallvencounteredinbornerrorofamino
acidmetabolism.
Hyperphenylalaninemiamayalsobecausedbydeficienciesinany
oftheseveralenzymesrequiredtosynthesizeBH4,or
indihydropteridinereductase,whichregeneratesBH4fromBH2.
BH4isalsorequiredfortyrosinehydroxylaseandtryptophan
hydroxylase,whichcatalyzereactionsleadingtothesynthesisof
neurotransmitters,suchasserotoninandreversethecentral
nervoussystem(CNS)effectsduetodeficienciesin
neurotransmitters.

23.Oxidativephoephoryletlonnotinhiblted
by:
a)Fluoride
b)2,4-dinitrophenol(DNP)
c)Oligomycin
d)Carboxin
e)Ouabin
CorrectAnswer-A:D:E
Answer:(a)Fluoride,(d)Carboxin,(e)Ouabin(Ref:Harper
30th/132-33;Lippincott6th/79;Satyanarayan4th/233-34;
Chatterjea7th/132-341
TherearethreesitesinrespiratorychainwhereATPisformedby
oxidativephosphorylation.Threesitesare-Sitel(Complex-I),Site
Il(ComplexIII)andSiteIll(ComplexIV).Complexll(Succinate
dehyfuogenaseFAD)isnotinvolvedinoxidativephosphorylation.
2,4-dinitrophenol(DNP)Dinitrocresol,Trifluorocarbonylcyanide
phenylhydrazone,Pentachlorophenol
Aspirin(inhighdose),Highconcentrationofthermogenin,thyroxine
andlongchainfreefattyacids,Antibiotics-valinomycin,gramicidinA
andnigercinareinhibitoresofoxidativephosphorylation
CarboxininhibitcomplexII,whichisnotinvolvedinoxidative
phosphorylation(sonotincludedinanswer)"?Chatterjea7h/134
"OuabainisacardiacglycosidethatactsbyinhibitingtheNW/IC-
ATPasesodium-potassiumionpump"-Harper30th/491
Fluoride:Itinhibitstheactivitiesofcertainenzymes.Sodium
fluorideinhibitsenolase(ofglycolysis)whilefluoroacetateinhibits

aconitase(ofcitricacidcycle)"-Satyanarayan4th/420.

24.TrueaboutApolipoproteii.
a)Constituteperipheralregionofplasmalipoproteins
b)DividedintoA,B,Conly
c)ApoA-Iisthemajorproteincomponentofhighdensity
lipoprotein(HDL)
d)ApoA,BandCarefurtherdivided
e)Roleinenzymeactivation
CorrectAnswer-A:C:D:E
Answer.(a)Constituteperipheralregionof...,(c)ApoA-Iisthe
majorprotein...,(d)ApoA,BandCare...,(e)Roleinenzyme...
[Ref:Harper30th/254-55;Satyanarayan4th/318;
http://noprniscairres.in/bitstream;onlinelibrary.wiley.com]
Apolipoproteins(apo)playveryimportantrolesinthesynthesisand
catabolismofplasmalipoproteins,inlipidtransport,andas
activatorsofcertainenzymesassociatedwithlipidandlipoprotein
metabolism
Apolipoproteinsaretheproteincomponentofplasma
lipoproteins
whichconsistofacoreoftriglyceridesandcholesterol
estersandaperipheralregionofphospholipid,sphingolipidand
protein.
ApoA-Iisthemajorproteincomponentofhighdensity
lipoprotein(HDL)
andaminorcomponentofchylomicronsandvery
lowdensitylipoprotein(VLDL).
Apolipoproteinsaredividedbystructureandfunctionintofive
majorclasses,AthroughE,withmostclasseshaving
subclasses,forexample,apolipoprotein(orapo)A-IandapoC-
II.



25.Aminoacidscontaininghydroxylgroup:
a)Threonine
b)Tyrosine
c)Serine
d)Tryptophan
e)Valine
CorrectAnswer-A:B:C
Answer:(a)Threonine,(b)Tyrosine,(c)SerineLippincott6th/4
Serine,threonine,andtyrosineeachcontainapolarhydroxyl
group
thatcanparticipateinhydrogenbondformation.
Thesidechainsofasparagineandglutamineeachcontainacarbon
ylgroupandanamidegroup,bothofwhichcanalsoparticipatein
hydrogenbonds"-

26.CytochromeP45Ois/areinvolvedln:
a)Hydroxylationofxenobiotics
b)Methylationofxenobiotics
c)Deaminationreaction
d)Involvedinhydroxylationofsteroids
e)Druginteraction
CorrectAnswer-A:C:D:E
Ans.(a)Hydroxylationofxenobiotics,(c)Deaminationreaction,
(d)Involvedinhydroxylationofsteroids,(e)Druginteraction
[Ref:Harper30th/584-85;KDT7th/23-26;Lippincott6th/;
Satyanarayan4th/639-40]
CytochromeP450sareinvolvedinphaseI(hydroxulation)ofthe
metabolismofxenobiotics,notinphaseII.Methylationof
xenobioticsoccurinphaseIIbymethyltransferase


27.Trueaboutroleofphospholipids:
a)Celltocellrecognition
b)Cellsignaling
c)PrecursorofSecondMessengers
d)Mediatorsofinflammation
e)Regulatemembranepermeability
CorrectAnswer-B:C:D:E
Answer:(b)Cellsignaling,(c)PrecursorofSecond
Messengers,(d)Mediatorsofinflammation,(e)Regulate
membranepermeability
(Ref:Harper30th/212,216,253-54;Robbins9th/83-84;
Satyanarayan4th/36-37)
Glycoprotein(fibronectin,laminin)isinvolveincell-cell
recognition
andadhesion.
Theinositolispresentinphosphatidylinositolasthestereoisomer,
myoinositol.Phosphorylated
phosphatidylinositols(phosphoinositides)areminorcomponentsof
cellmembranes,butplayanimportantpartincellsignalingand
membranetrafficking.
Sphingomyelinsarealsofoundinlargequantitiesinthemyelin
sheaththatsurroundsnervefibers.Theyarebelievedtoplaya
roleincellsignalingandinapoptosis.
Phosphatidylinositolisthesourceofsecondmessengers
inositoltriphosphateanddiacylglyceol,thatareinvovedthe
actionofsomehoromones.


28.Correctstatementaboutmembrane:
a)Phospholipidsundergorapidlateraldiffusion
b)Transversemovementoflipidsacrossthemembraneisfaster
thanprotein
c)Hydrophobiccoreofthephospholipidbilayerremainsconstantly
inmotionbecauseofrotationsaroundthebondsoflipidtails
d)Phospholipdsthathaveonefattyacylgroup,cannotformthe
bilayer
e)Phospholipidsspanwholebilayer
CorrectAnswer-A:C:D
Ans.a)Phospholipidsundergo...,(c)Hydrophobiccoreofthe
phospholipid...,(d)Phospholipdsthathaveonefattyacyl...
(Ref:Harper30th/215-17,478-90;Satyanarayan4th/650-51;
en.wikibooks.org]
Membranesaremainlymadeupoflipids,proteinsandsmallamount
ofcarbohydrate.Phospholipidsarethemostcommonlipidspresent
andtheyareamphipathicinnature.
Thehydrophobiccoreofthephospholipidbilayerisconstantlyin
motionbecauseofrotationsaroundthebondsoflipidtails.
Hydrophobictailsofabilayerbendandlocktogether.However,
becauseofhydrogenbondingwithwater,thehydrophilichead
groupsexhibitlessmovementastheirrotationandmobilityare
constrained.Thisresultsinincreasingviscosityofthelipidbilayer
closertothehydrophilicheads.
Thelysophosphollpi.dshaveonlyonefattyacylgroup,itcannotform
thebilayerasthepolarheadsaretoolarge,similarlycholesterolalso
cannotformbilayersastherigidfitsedringsystenxsandadditional

nonpolartailsaretoolarge".

29.DuringDNAreplicationwhichbond
breaks:
a)Phosphodiesterbonds
b)Phosphatebond
c)Hydrogenbond
d)Glycosidicbonds
e)None
CorrectAnswer-C
Answer:c.Hydrogenbond[Ref:Lippincott6th/397-400;Harper
30th/381-86;Satyanarayana4th/524-29]
WhenDNAreplicates,ahelicaseenzyme"unzips"thedoublehelix,
breakingthehydrogenbondsthatholdittogetherinthecenter
Thetwostrandsofthedoublehelixseparatewhenhydrogenbonds
betweenthepairedbasesaredisrupted.Disruptioncanoccurinthe
laboratoryifthepHoftheDNAsolutionisalteredsothatthe
nucleotidebasesionize,orifthesolutionisheated.

30.whichnNAcontalneabnormalpurineand
pyrimidine:
a)tRNA
b)23SrRNA
c)16SrRNA
d)5SrRNA
e)mRNA
CorrectAnswer-A
Answer:a.tRNA
tRNAmoleculecontainahighpercentageofunusalbases,for
example,dihydrouracilandhaveextensiveintra-chainbasepairing
thatleadstocharacteristicsecondaryandtertiarystructure"-
Lippincott6th/418.
ThetRNAmoleculescontainahighpercentageofunusualbases
(forexample,dihydrouracil)andhaveextensiveintrachainbase-
pairingthatleadstocharacteristicsecondaryandtertiarystructure.
TransferRNAisuniqueamongnucleicacidsinitscontentof
"unusual"bases.Anunusualbaseisanypurineorpyrimidinering
excepttheusualA,G,C,andUfromwhichallRNAsare

31.Componentof50Sribosomalzubunit:
a)16SRNA
b)18SRNA
c)5.8RNA
d)5SRNA
e)23SRNA
CorrectAnswer-D:E
Answerd.5SRNA,(e)23SRNA(Lippincott6th/436):
The50Ssubunitisprimarilycomposedofproteinsbutalsocontains
single-strandedRNAknownasribosomalRNA(rRNA).rRNAforms
secondaryandtertiarystructurestomaintainthestructureandcarry
outthecatalyticfunctionsoftheribosome.
Itincludesthe5SribosomalRNAand23SribosomalRNA.
50Sincludestheactivitythatcatalyzespeptidebondformation
(peptidyltransferreaction),preventsprematurepolypeptide
hydrolysis,providesabindingsitefortheG-proteinfactors(assists
initiation,elongation,andtermination),andhelpsproteinfoldingafter
synthesis.

32.TrueaboutChromatin,remodeling:
a)EnergyisrequiredtodisplacethehistoneoctamersfromDNA
ortranslocatethemontoneighboringDNAsegments
b)Histonemodificationsbyspecificenzyme
c)Donotinvolveenzymes
d)Aberrationsinchromatinremodelingproteinsmaybe
associatedwithcancer
e)None
CorrectAnswer-A:B:D
Answer:aEnergyisrequiredtodisplacethehistone...,(b)
Histonemodificationsby...,(d)Aberrationsinchromatin...
[Ref:Harper30th/735,438-39;Lippincott6th/422,460;Harrison
19th/102e-7]
Chromatinremodelingisthedynamicmodificationof
chromatinarchitecturetoallowaccessofcondensedgenomic
DNAtotheregulatorytranscriptionmachineryproteins,and
therebycontrolgeneexpression.
chromatinremodelingcomplexesdisplacethehistone
octamersfromDNAortranslocatethemontoneighboringDNA
segments,therebyexposingunderlyingDNAsequencesto
sequencespecificregulatoryfactors.
histoneacetylaseandotherenzymaticactivitiesareassociated
withthecoregulatorsinvolvedinregulationofgene
transcription.
Aberrationsinchromatinremodelingproteinsarefoundtobe
associatedwithhumandiseases,includingcancer.


33.Bestassessmentofproteinbinding
regionsonaDNAmoleculecanbedone
by:

a)DNAfootprinting
b)RTPCR
c)Microarray
d)Westernblotting
e)Northernblotting
CorrectAnswer-A
Answer-(a)DNAfootprinting[Ref:www.biotecharticles.com;
www.biologyexams4u.comLippincott6th/473]
DNAfootprinting-Anin-vitrotechniquetofindoutprotein
bindingregionsonaDNAmolecule.Thetechniqueisalso
calledasDNAseIfootprinting.Thousandsofproteins
(enzymes)areinteractingwithDNAinthenucleusfor
regulatingactivitieslikereplication,transcription,translation
etc.
DNAFootprintingisamoleculartechniqueusedtoidentifythe
specificDNAsequence(bindingsite)thatbindstoaprotein.
Thistechniquemainlyusedtoidentifythetranscriptionfactors
whichbindtopromoter,enhancerorsilencerregionofgeneto
regulateitsexpression.Thereforetheregulationoftranscript
ionofagenecanbestudiedusingthismethod.


34.Featuresofchronicmyelogenous
leukemia(CML)-
a)Bonemarrowbiopsyisnecessaryfordiagnosis
b)PresenceofBCR-ABLgenewhichdirectsthesynthesisof
BCR-ABLtyrosinekinase
c)Dasatinibisusedinimatinibresistantcases
d)Generalizedpainfullymphadenopathyispresentingfeaturein
mostcases
e)Myeloblastsusuallyconstitutemorethan10%ofallwhitecells
inchronicphase
CorrectAnswer-A:B:C
Answer-A,B,C,Bonemarrowbiopsyisnecessaryfor
diagnosis,(B)PresenceofBCR-ABLgenewhichdirectsthe
synthesisofBCR-ABLtyrosinekinase(C)Dasatinibisusedin
imatinibresistantcases
Splenomegalyispresentin90%
Imatinib,dasatinibandnilotinibspecificallyinhibitBCRABLtyrosine
kinaseactivityandreducetheuncontrolledproliferationofwhite
cells.
ThediseaseisdrivenbytheBCR-ABL1chimericgeneproduct,a
constitutivelyactivetyrosinekinase.
Commonmanifestationsareofanemiaandsplenomegaly,
lymphadenopathy,andextramedullarydisease(skinor
subcutaneouslesions)
Thebonemarrowishypercellularwithmarkedmyeloidhyperplasia
andahighmyeloid-to-erythroidratioof15-20:1.


35.TrueaboutCysticfibrosis-
a)OccursdueCFTRgenemutationonchromosome7
b)Meconiumileusispresentin>90%cases
c)CFTRgenecanbedetectedantenately
d)Poorbodygrowth
e)Alloftheabove
CorrectAnswer-A:C:D
Answer-A,C,D,OccursdueCFTRgenemutationon
chromosome7(C)CFTRgenecanbedetectedantenately
(D)Poorbodygrowth
Theprimarydefectincysticfibrosisresultsfromabnormalfunction
ofanepitheltalchloridechannelproteinencodedbythecystic
fibrosistransmembraneconductanceregulator(CFTR)geneon
chromosome7.
Contentsoftheintestinallumenaredifficulttoexcretewhichresults
inmeconiumileus.
SequencingtheCFTRgeneisthegoldstandardfordiagnosisof
cysticfibrosisPoorbodygrowth

36.Trueaboutcaspases-
a)Caspasesinitiateapoptosisbyextrinsicandintrinsicpathway
b)Caspasesareproteaseenzyme
c)Caspasesarereceptor
d)Caspasesinhibitapoptosis
e)Causesnonenzymaticdegradationofcriticalcellular
components
CorrectAnswer-A:B
Answer-A,B,Caspasesinitiateapoptosisbyextrinsicand
intrinsicpathway(B)Caspasesareproteaseenzyme
Apoptosisresultsfromtheactivationofenzymescalledcaspases.
Theprocessofapoptosismaybedividedintoaninitiationphase
(intrinsicpathway)andexecution(extrinsicpathway).
Twodistinctpathwaysconvergeoncaspaseactivation:
Themitochondrialpathwayandthedeathreceptorpathway
Caspasesareafamiyofendoproteases.
Theactivationoftheseenzymeistightlycontrolledbytheir
productionasinactivezymogensthatgaincatalyticactivityfollowing
signalingeventspromotingtheiraggregationintodimersor
macromolecularcomplexes"

37.Trueaboutminimalchangedisease-
a)Hypertensioniscommonlypresent
b)Mostcommoncauseofnephroticsyndromeinadults
c)Highdosesteroidsresultsinremissioninmostcases
d)Commonlyprogresstochronicrenalfailure
e)Reversiblelossofpodocytefunction
CorrectAnswer-C:E
Answer-(C)Highdosesteroidsresultsinremissioninmost
cases(E)Reversiblelossofpodocytefunction
Minimalchangedisease:
Alsok/alipoidnephrosa,footprocessdisease&Nildepositdisease
Thediseasesometimesfollowsarespiratoryinfectionorroutine
prophylacticimmunization'
Theonsetmaybeprecededbyanupperrespiratoryinfection,atopic
allergyorimmunisation.
Thediseasecharacteristicallyrespondtosteroidtherapy
Thebenigndisorderischaracterizedbydiffuseeffacementoffoot
processesofvisceralepithelialcell(podocytes).
mostfrequentcauseofnephroticsyndromeinchildren
Thevisceralepithelialchangesarecompletelyreversibleafrer
corticosteroidtherapy,concomitantwithremissionoftheproteinuria.
Thereiscommonlynohypertensionorhematuria.
Theappearanceofacuterenalfailureinadults.

38.Feature(s)ofAdultpolycystickidney
diseaseis/are:
a)Renalenlargement
b)Smallkidney
c)Spiderlegdeformityonintravenousurography
d)Ultrasoundshowsmultiplecysts
e)Alloftheabove
CorrectAnswer-A:C:D
Answer-A,C,D,Renalenlargement(C)Spiderlegdeformityon
intravenousurography(D)Ultrasoundshowsmultiplecysts
ADPKDischaracterizedbytheprogressivebilateralformationof
renalcrisis.
Inhieritence-autosomaldominant
Characterizedbymultipleexpandingcystsofbothkidneys.
Ingrossappearance,thekidneysarebilaterallyenlarged.
Thepainmayresultfromrenalcystinfection,hemorrhage,or
nephrolithiasis.
'Intravenousurographypolycystickidneydisease:Thespiderlegs,
deformityofthecalyces.

39.Whichis/arecausedbyprotein
misfolding:
a)Creutzfeldt-Jakobdisease
b)Bovinespongiformencephalopathy
c)Huntingtondisease
d)Alzheimerdisease
e)Parkinsondisease
CorrectAnswer-A:B:C:D:E
Answer-A,B,C,D,E,Creutzfeldt-Jakobdisease(B)Bovine
spongiformencephalopathy(C)Huntingtondisease
(D)Alzheimerdisease(E)Parkinsondisease
Theproteinsfailtofoldintotheirnormalconfiguratoryinthis
misfoldedstate,theproteinscanbecometoxicinsomeway(again
oftoxicfunction)ortheycanlosetheirnormalfunctionwhichis
knownasproteinmisfoldingdisease.
SuchdiseasesasCreutzfeldt-Jakobdisease,Alzheimer'sdisease,
Parkinson'sdisease,priondisease,amyloidosis,Bovinespongiform
encephalopathy,Huntingtondisease.

40.TrueaboutCreutzfeldt-JakobDisease:
a)Gliosisinthalamus
b)Spongiformswellingincerebralcortex
c)Brainatrophyinlatestage
d)SlowandirregularbackgroundrhythmonEEG
e)None
CorrectAnswer-B:C:D
Answer-B,C,D,Spongiformswellingincerebralcortex
(C)Brainatrophyinlatestage(D)Slowandirregular
backgroundrhythmonEEG
CJDisararedisorderthatmanifestsclinicallyasarapidly
progressivedementia.
TheprogressionofthedementiainCJDisusuallysorapidthatthere
islittleifanygrosslyevidentbrainatrophy.
Microscopically,thehallmarkisspongiformchangeofthecerebral
cortex.
Inadvancedcasesthereissevereneuronalloss,reactivegliosis.
EEGabnormaltdesarepresentinnearlyallpatients,consistingofa
slowandirregularbackgroundrhythmwithperiodiccomplex
discharges.

41.Hereditarynon-polyposiscolorectal
cancer(HNPCC)is/arecommonly
associatedwith-

a)Endometrialcancer
b)Cervicalcancer
c)Ovariancancer
d)Breastcancer
e)Thyroidcancer
CorrectAnswer-A:C
Answer-A,C,Endometrialcancer(C)Ovariancancer
Hereditarynon-polyposiscolorectalcancer(HNPCC)-
Malignancies-Colonic,endometrial,ovarian,pancreatic,gastric.
Inherictance-autosomaldominant
Gene-ADMSH2,MLH1,MSH6,PMS1,PMS2

42.FeaturesofNonbacterialthrombotic
endocarditis(NBTE)-
a)CommoninSLE
b)Presentonundersurfaceofvalve
c)Vegetativegrowthislargeandlooselyattachedtovalve
d)Mayoccurafterpost-cardiaccatheterization
e)Sourceofsystemicemboli
CorrectAnswer-B:D:E
Answer-B,D,E,Presentonundersurfaceofvalve(D)May
occurafterpost-cardiaccatheterization(E)Sourceofsystemic
emboli
Theseverrucaearetypicallysmall,singleormultiple,brownishand
occuralongthelineofclosureoftheleaflet.
VegetatianofNBTEissmallandlooselyattachedtotheunderlying
valve.
Sourceofsystemicembolithatproducesignificantinfarctsinthe
brain,heart,spleenandkidneys.
Itfrequentlyoccurswithdeepvenousthrombosis,pulmonaryemboli.

43.TrueaboutAlzheirnerdisease:
a)Mostcommoncauseofdementiainelderly
b)Unusalbefore45yearsofage
c)Plaquesconsistsoftauprotein
d)Mayhavefamilyhistory
e)Shorttermmemoryisaffectedlessthanlong-termmemory
CorrectAnswer-A:B:D
Answer-A,B,D,Mostcommoncauseofdementiainelderly
(B)Unusalbefore45yearsofage(D)Mayhavefamilyhistory
Alzheimer'sdisease(AD)isaslowlyprogressivediseaseofthe
brainthatischaracterizedbyimpairmentofmemoryandeventually
bydisturbancesinreasoning,planning,language,andperception.
Alzheimer'sdiseaseisCommonin5thand6thdecade.
Trisomy21isassociatedwithalzheimer'sdementia.
Plaquecontainingbeta-amyloidpeptide,andneurofibrillarytangles
containingtauproteinoccursinneocortex.
Thecausesincludegenetic,environmental,andlifestylefactors.
DementiaofAlzheimer'stypeisassociatedwithDepressive
symptoms,Delusions,Apraxiaandaphasia.
Recentmemoryloss(shorttermmemoryloss)isfeatureof
Alzheimer'sdisease.

44.Negribodiesinanimalcanbebestseen
in:
a)Hippocampus
b)Basalganglia
c)Cerebralcortex
d)Cerebellum
e)Thalamus
CorrectAnswer-A:D
Answer-A,D,Hippocampus(D)Cerebellum
Theyaremostprominentinpyramidalcellsofthehippocampusand
Purkinjecellsofcerebellumbuthavebeenseeninnervecells
throughoutthebrainandspinalcord.

45.Trueaboutp53-
a)Hastyrosinekinaseactivity
b)Haspro-apoptoticactivity
c)Tumoursuppressorgene
d)Hasanti-apoptoticactivity
e)None
CorrectAnswer-A:B:C
Answer-A,B,C,Hastyrosinekinaseactivity(B)Haspro-
apoptoticactivity(C)Tumoursuppressorgene
p53isatumorsuppressorgeneanditisaproapoptoticfactor,i.e.it
promotesapoptosisifrepairofDNAdamageisunsuccessfulatG1
arrest.
Theproteinkinasesthatareknowntotargetthistranscriptional
activationdomainofp53.

46.TrueaboutTakayasusyndrome:
a)InvolvessmallandMediumsizedvessels
b)Sharesmanyclinicalfeaturesofgiantcellarteritisifinvolves
aorta
c)Morecommoninmalethanfemale
d)Granulomatousvasculitis
e)Alsocalledpulselessdisease
CorrectAnswer-B:C:E
Answer-B,D,E,Sharesmanyclinicalfeaturesofgiantcell
arteritisifinvolvesaorta(D)Granulomatousvasculitis(E)Also
calledpulselessdisease
"Giantcellarteritis(GCA)isagranulomatousarteritisthat
predominantlyaffectsmedium-sizedarteriesintheheadandneck.
Itpredominantlyaffectstheaorta.
Takayasuarteritis(Pulselessdlsease):Thediseaseaffectschiefly
youngwomen.
Takayasuarteritisisagranulomatousvasculitisofmediumsized
andlargerarteriescharacterizedprincipallybyoculardisturbances
andmarkedweakeningofthepulsesintheupperextremities.

47.Whichareinheritablemalignancies:
a)Breastcancer
b)Thyroidcancer
c)Wilmstumour
d)Retinoblastoma
e)Prostatecancer
CorrectAnswer-A:B:C:D:E
Answer-A,B,C,D,E,Breastcancer(B)Thyroidcancer
(C)Wilmstumour(D)Retinoblastoma(E)Prostatecancer
Breast/ovarian-Breast,ovarian,colonic,prostatic,pancreatic
Wilm'stumour-Nephroblastoma,neuroblastoma,hepatoblastoma,
rhabdomyosarcomaRetinoblastoma-Retinoblastoma,
osteosarcoma
Prostatecancer-prostate
Cowden'ssyndrome-Breast,thyroid,gastrointestinaltract,
pancreatic

48.CausesofNon-megaloblasticmacrocytic
anameia-
a)Folatedeficiency
b)Leadtoxicity
c)Hypothyroidism
d)Liverdisease
e)VitB12deficiency
CorrectAnswer-C:D
Answer-C,D,Hypothyroidism(D)Liverdisease
Causesinclude:
Chronicalcoholism
Liverdisease
Hypothyroidism
Reticularfibrosis
Blood.disorderslikered-cellaplasia,aplastic
anemia,myelodysplasticsyndromesandmyeloidleukemia
Drugsasazathioprine
Pregnancy

49.Truestatement(s)aboutWilm'stumour-
a)Mostcommonlypresentsasasymptomaticabdominalmass
b)Hereditarypredispositionispresentin50%cases
c)Bilateralin25%cases
d)Classictriphasiccombinationofblastemal,stromal,and
epithelialcelltypesisobserved
e)Mostcommoninchildren
CorrectAnswer-A:D:E
Answer-A,D,E,Mostcommonlypresentsasasymptomatic
abdominalmass(D)Classictriphasiccombinationofblastemal,
stromal,andepithelialcelltypesisobserved(E)Mostcommon
inchildren
Itisamalignanttumourofkidneywhichisseeninchildren
Tumouriscomposedofepithelialandmesothelialelements(bone,
cartilage,muscleetc)socalledasnephroblastoma(immature
embryonictissue)
Wilmstumor,alsoknownasnephroblastomaisacomplexmixed
embryonalneoplasmofthekidneycomposedofthreeelaments:
blastema,epithelia,andstroma.
Clinicalfeatures-
Commoninfemalechildren(2-4years)
Massintheabdomen.
Abdominaldistensionduetoenlargedkidney
Rarely,Wilm'stumourisbilateral
Hematuria

50.Whichoffollowingisnotclassifiedas
Primitiveneuroectodermaltumour(PNET):
a)Retinoblastoma
b)Medulloblastoma
c)Rhabdomyosarcoma
d)Ewingsarcoma
e)Carcinoidtumour
CorrectAnswer-A:C:D:E
Answer-A,RetinoblastomaC,RhabdomyosarcomaD,Ewing
sarcomaE,Carcinoidtumour
Embryonaltumorsorprimitiveneuroectodermaltumors(PNET)are
themostcommongroupofmalignantCNStumorsofchildhood.
PNETgroupincludes-medulloblastoma,supratentorialPNET,
ependymoblastoma,medulloepithelioblastoma,andatypicalteratoid/
rhaboidtumor(ATRT).
Ewing'ssarcomaiscloselyrelatedtoPNET,butnotPNET.
Recently,Ewingsarcomaandprimitiveneuroectodermaltumor
(PNET)havebeenunifiedintoasinglecategory:
TheEwingsarcomafamilytumors(ESFT)basedonsharedclinical,
morphologic,biochemicalandmolecularfeatures

51.GlucoselevelinCSFis/arereducedin:
a)Bacterialmeningitis
b)Fungalmeningitis
c)Viralmeningitis
d)Tubercularmeningitis
e)Spirochetalmeningitis
CorrectAnswer-A:B:D
Answer-A,BacterialmeningitisB,Fungal
meningitisD,Tubercularmeningitis
Normal-45-48mg/dL
Bacterialmeningitis-Markedlyreduced(low<45)
Fungalmeningitis-Markedlyreduced(low<45)
Viralmeningitis-Normalorlow
Tubercularmeningitis-Reduced(low<45)
Spirochetalmeningitis-Normal

52.Trueaboutproliferalivephaseofwound
healing
a)Neutrophilsincreasesgradually
b)Macrophageincreasesgradually
c)CollagentypeIpresentpredominantly
d)CollagentypeIIIpresentpredominantly
e)Angiogenesisoccurs
CorrectAnswer-D:E
Answer-D,CollagentypeIIIpresent
predominantlyE,Angiogenesisoccurs
Duringproliferation,thewoundis'rebuilt'withnewgranulation
tissuewhichiscomprisedofcollagenandextracellularmatrixand
intowhichanewnetworkofbloodvesselsdevelop,aprocess
knownas'angiogenesis'.
Maturationisthefinalphaseandoccursoncethewoundhas
closed.ThisphaseinvolvesremodellingofcollagenfromtypeIIIto
typeICellularactivityreducesandthenumberofbloodvesselsin
thewoundedarearegressanddecrease.

53.Correctmatchofstainwithtissueis/are-
a)Perls'Prussianblue-ironintissue
b)VonKossa-collagen
c)Masson'strichrome-elastinfiber
d)PAS-glycogen
e)PAS-Acidicandneutralmucin
CorrectAnswer-A:D:E
Answer-A,Perls'Prussianblue-ironintissueD,PAS-
glycogenE,PAS-Acidicandneutralmucin
1.Masson'strichrome-Trichromehistologystains
Canbeusedtodistinguishbetweencellularitemsandextracellular
items
Canbeusedonconnectivetissue
2.VonKossastain-usedtoindicatecalciumandcalcium
deposits
3.Periodicacidschiff(PAS)-AMucinstain
Usedforstainingglycogen
Usedtoshowglomeruli,basementmembranes,andglycogeninthe
liver.
4.Perls'Prussianblue-Canbeusedtorevealthepresenceof
ironinbiologicaltissues


54.Whichofthefollwingis/areactionof
estrogenexcept:
a)Developmentofthealveolarsystem
b)Proliferationofstromaofbreast
c)Ductalgrowthofthebreast
d)CBoneresorption
e)Developmentoflobulesofbreast
CorrectAnswer-A:E
Ans.(A)Developmentofthealveolarsystem(E)Development
oflobulesofbreast
Oestrogencausesonlyductdevelopment.
Progesteroneisresponsibleforglandulardevelopment.
Itpromotesthegrowthofthelobulesandalveolartissueinbreast.
Promotesthedevelopmentofthealveolar(acinar)systemofthe
breast
Estrogen-functions:
Importantinmaintainingbonemassprimarilybyretargetingbone
resorption.
Producedatpubertycausegrowthofbreastsproliferationofducts
andstroma,accumulationoffat.
Stimulatestromaldevelopmentandductalgrowthinthebreast
Contributetothegrowthofaxillaryandpubiahair.
Pigmentationintheskin,mostprominentintheregianofthenipplas
andareolae.
Continuousexposuretoestrogensforprolongedperiodsleadsto
hyperplasiaoftheendometrium.

Decreasetherateofresorptionofbonebypromotingapoptosisof
osteoclastsandbyantagonizingtheosteoclastogenicandpro-
osteoclasticeffectsofparathyroidhormoneandinterleukin-6.

55.Allaretrueaboutbisphosphonatesexcept
:
a)Preventreabsorptionofbonebyosteoclast
b)Structurallysimilartopyrophosphate
c)Absorptionincreaseswithfood
d)Canbesafelygiveninliverdisease
e)None
CorrectAnswer-C
Ans.C.Absorptionincreaseswithfood
Bisphosphonates(BPNs):
BPNsareanalogiesofpyrophosphate;carbonatomreplacing
orygenintheP-O-Pskeleton
AlloralBNPsarepoorlyabsorbedandproducegastricirritationas
majorsideeffect.
Theyinhibitsboneresorptionandhaverecentlyattracted
considerableattentionbecauseoftheirabilitytoprevent
osteoporosisinadditiontotheirusefulnessinmetabolicbone
diseasesandhypercalcemia.
MOA:Localizetoregionsofboneresorption&exerttheirgreatest
effectsonosteoclasts.
Foodreducesabsorptionevenfurther,necessitatingtheir
administrationonanemptystomach.
Nearlyhalfoftheabsorbeddrugaccumulatesinbone;the
remainderisexcretedunchangedintheurine.
Decreasedrenalfunction,esophagealmotilitydisorders&peptic
ulcerdiseasearethemaincontraindications.


56.Whichofthefollowingis/arenewerdrugs
forTB:
a)Bedaquiline
b)Clofazimine
c)Coftaroline
d)Rifapentine
e)Etanercept
CorrectAnswer-A:B:D
Ans.(A)Bedaquiline(B)Clofazimine(D)Rifapentine
NewerAnti-TBdrugs:
CommunityMedicinewithRecentAdvancesbySuryakantha
4ed/371
Rifabutin
Rifapentine-400mgtwiceweekly.
Macrolides:Roxithrornycin,Clarithromycin,Azithromycin
Amikacin
Fluoroquinolones:Ciprofloxacin,ofloxacinandsparfloxacin
B-lactamantibiotic:trialsaregoingonwithamoxicillin-clavulanic
acid
Clofazimine-200mg/d
Paromomycin
Cytokineimmunotherapy:IL-2,cytokinegammainterferonand
cytokineIL-12
Bedaquiline-Multi-drugresistanttuberculosis(MDR-TB)-400mg/d

57.Allaretrueaboutoralirontherapyin
anemiaexcept:
a)Mayworseninflammatoryboweldisease
b)Ittakesminimum2weeksforreticulocytecounttoincrease
c)Generally3-6monththerapyisrequiredtoreplenishironstores
d)Gastrointestinalside-effectslimitsitsdose
e)Hblevelisgenerallyattainedin1-3month
CorrectAnswer-B
Ans.B.Ittakesminimum2weeksforreticulocytecountto
increase
OralironTherapy:
Followingoraliron,normalHblevelisusuallyobtainedwithin1to3
months.
DependingmainlyontheinitialHblevel.
Itisimportant,however,tocontinuewiththetherapyfor12-20
weeksaftertheHblevelhasreturnedtonormal,inordertoreplenish
thedepletedironstores.
Thereticulocytecountintheperipheralbloodbeginstorisewithina
week,reachesapeakat10to14daysandreturnstonormalafter3
weeks
AdverseEffectsofOraliron:
Epigastricpain,heartburn,nausea,vomiting,bloatingstainingof
teeth,metallictaste,col:la,
Alterationofintestinalflora.
Gastricirritationandconstipation.
TreatmentofironDeficiency:

Abilityofthepatienttotolerateandabsorbmedicinaliron.

58.Drugwhichcanbegivenbyinhalation
route:
a)Zileuton
b)Steroid
c)Salbutamol
d)Tobramycin
e)None
CorrectAnswer-B:C
Ans.(B)Steroid(C)Salbutamol
lnhalatlonalsteroids.Beclomethasone,dipropionate,budesonide,
fluticasonepropionate,flunisolideandciclesonide.
Salbutamol:usedinformoforal,i.m/s.candinhalation.
Zileuton:Itisavailableonlyasextendedrelease(oral)formulation.
Tobramycin:Usedinformofi.m/i.v,eyedrop

59.Liverfunctiontest(LFT)monitoringis/are
requiredinuseofwhichofthefollowing
DiseaseModifyingAntirheumaticDrugs
(DMARDs):

a)Methotrexate
b)Hydroxychloroquine
c)Sulfasalazine
d)Leflunomide
e)Gold
CorrectAnswer-A:D:E
Ans.(A)Methotrexate(D)Leflunomide(E)Gold
DMARDsUsedforthetreatmentofrheumatoidarthritis:
Hydroxychloroquine-Funduscopicandvisualfieldtestingevery12
months.
Sulfasalazine-CBCevery2-4weeksforthefirst3months,then
every3months
Methotrexate&Leflunomide-CBC,creatinine,LFTsevery2-3
months
Tocilizumab-CBCandLFTsatregularintervals

60.Drugswhichcanbeusedingestational
hypertension:
a)Metoprolol
b)Labetalol
c)Methyldopa
d)Sustainedreleasenifedipine
e)Losartan
CorrectAnswer-A:B:C:D
Ans.(A)Metoprolol(B)Labetalol(C)Methyldopa(D)Sustained
releasenifedipine
Antihypertensiveagentsusedinpregnancy:Methyldopa,
lrydralazlne,labetalol,Nifedipine,atenolol.

61.Adverseeffectsofmirtazapineis/are:
a)Insomnia
b)Sedation
c)Sexualdysfunction
d)Vomiting
e)Weightgain
CorrectAnswer-B:E
Ans.(B)Sedation(E)Weightgain
Mirtazapine
Importantadversedrugreactions
markedsedation
increasedappetite
weightgain
Somnolence(mostcommon)
Drymouth
Constipation
Dizziness
Myalgias
Increaseserumcholesterolconcentrationto20percent
Orthostatichypotension
Agranulocytosis

62.Carbonicanhydraseinhibitors(S)is/are:
a)Acetazolamide
b)Amiloride
c)Nitrofurantoin
d)Topiramate
e)None
CorrectAnswer-A:D
Ans.(A)Acetazolamide(D)Topiramate
Carbonicanhydraseinhibitor:Topiramate,Acetazolamida,
methazolamide,dichlorphenamide
orallyinthetreatmentofglaucoma-Acetazolamida,methazolamide,
dichlorphenamide
Topicallyactivecarbonicanhydraseinhibitors-Dorzolamideand
brinzolamide.

63.Whichofthefollowingis/aretrueabout
Tacrolimus:
a)Amacrolidesantibiotic
b)Structuresimilartocyclosporine
c)Derivedfromafungus
d)Tcellinhibitor
e)Hirsutismlessevidentthancyclosporine
CorrectAnswer-A:D:E
Ans.(A)Amacrolidesantibiotic(D)Tcellinhibitor(E)Hirsutism
lessevidentthancyclosporine
Tacrolimus:
Immunosuppressantischemicallydifferentfromcyclosporine,but
hasthesamemechanismofaction100timesmorepotent.
MacrolideantibioticproducedbyStreptomycestsukubaensis(a
bacteria)
MOA:
InhibitionofhelperTcellsviacalcineurin.
BindstotheimmunophilinFK-bindingprotein(FKBP)
Therapeuticapplication,clinicalefficacyaswellastoxicityprofilaare
similartocyclosporine.
Hypertension,hirsutism,gurnhyperplasiaandhyperuricemiaare
lessmarkedthanwithcyclosporine,buttacrolimusismorelikelyto
precipitatediabetes,causeneurotoxicityalopeciaanddiarrhoea.
Doselimitingtoxicityisrenal.

64.TrueaboutLowmolecularweightheparin
(IAMB):
a)Anti-factorXaassaymonitoringrequiredineverypatient
b)ItincreasesaPTTmorethanUFH
c)Canbesafelygiveninrenalfailure
d)Toxicityistotallyreversedbyprotaminesulphate
e)InactivatefactorXaselectively
CorrectAnswer-E
Ans.E.InactivatefactorXaselectively
LowMolecularWeight(LMW)HeparinsandUFH
HeparinhasbeenfractionatedintoLMWforms(MW3000-7000)by
differenttechniques..
InactivateFactorXaselectively.
TheyactonlybyinducingaconformationalchangeinATIIIand,not
byprovidingascaffoldingforinteractionofATIIIwiththrombin.
Asaresult,LMWheparinshavesmallereffectonaPTTandwhole
bloodclottingtimethanunfractionatedheparin(UFH).
Eliminatedprimarilybyrenalexcretionarenottobeusedinpatients
withrenalfailure..
SinceaPTT/clottingtimesarenotprolonged,Laboratorymonitoring
isnotneeded.
Protaminedoesnotneutralizefondaparinuxanditonlypartially
reversestheanticoagulanteffectofLMWheparins.

65.TrueaboutMafenide:
a)Canpenetrateeschars
b)Doesn'tcauseburningsensationwhenappliedtorawsurface
c)Canbeusedorally
d)Maycausemetabolicacidosis
e)None
CorrectAnswer-D
Ans.D,Maycausemetabolicacidosis
Mafenide:
TypicalsulpHonamide.
Usedonlytotally-inhibitsavarietyofgram-positiveandgram-
negativebacteria.
Biggestlimitation-Producesburningsensationandseverepain
whenappliedtorawsurface.
Mainlyemployedforbuntdressingtopreventinfection,butnotto
heatalreadyinfectedcases.

66.Whichofthefollowingis/aretrueabout
pharmacodynamicsofdrugs:
a)Affinitymeanshowstronglydrugbindstoreceptor
b)Efficacymeansmaximaleffectbyadrug
c)Irreversibleantagonistmainlyformsionicbondswithreceptor
d)Agonistpotencydependsontwoparameters:affinityand
efficacy
e)Forantagonists,efficacyiszero
CorrectAnswer-A:B:D:E
Ans.A,Affinitymeanshowstronglydrugbindsto
receptorB,EfficacymeansmaximaleffectbyadrugD,Agonist
potencydependsontwoparameters:affinityandefficacy&
E,Forantagonists,efficacyiszero
Irreversiblecompetitive(non-equilibrium)antagonismoccurswith
drugsthatpossessreactivegroupsthatformcovalentbondswith
receptor.
ReceptorBlockAntagonisminvolvestwoimportantMechanisms
Reversiblecompetitiveantagonist
Irreversible,ornon-equilibrium,competitiveantagonism.
Drugsactingonreceptorsmaybeagonistsorantagonists.
Agonistpotencydependsontwoparameters:affinity(i.e.tendency
tobindtoreceptors)andefficacy(i.e.ability,oncebound,toinitiate
changesthatleadtoeffects).
Forantagonists,efficacyiszero.

67.Allaretrueaboutplasmaproteinbinding
except:
a)Acidicdrugsgenerallybindtoplasmaalbuminandbasicdrugs
toa,acidglycoprotein
b)Plasmabindingdeterminesvolumeofdistribution
c)Moreplasmaproteinbindingmeansmorestorageinliver
d)Moreplasmaproteinbindingmeanslesspenetrationinvascular
membrane
e)Highdegreeofprotein.indinggenerallymakesthedruglong
acting
CorrectAnswer-C
Ans.C,Moreplasmaproteinbindingmeansmorestoragein
liver
Drugswhicharehighlyproteinboundorionizedremainlargely
withinthevascularcompartmentandhaveverylowvolumeof
distribution.
Acidicdrugsgenerallybindtoplasmaalbuminandbasicdrugstoa2
acidglycoprotein.
Albuminlsamajorcarrierforacidicdrugs.a1-acidglycoprotein
blindsbasicdrugs.
Clinicallysignificantimplicationsofplasmaproteinbinding:
Bindingofadrugtoplasmaproteinalsolimitsthedrug'sglomerular
fiLtration.
Drugtransportandmetabolismalsoarelimitedbybindingtoplasma
proteins.
Proteinbounddrugdoesnotcrossmembranes(exceptthrough

largeparacellularspaces,suchasincapillaries).
Tendtohavesmallervolumesofdistribution.
Boundfractionofdruginnotavailableforaction.
Plasmaproteinbindingthustantamountstotemporarystorageofthe
drug.
Highdegreeofproteinbindinggenerallymakesthedruglongacting,

68.Firstpassmetabolismissignificant
problemindruggiventhrough:
a)Sublingualroute
b)Rectalroute
c)Intramuscularroute
d)Directlyintostomach
e)Directlyintolargeintestine
CorrectAnswer-D
Ans.D,Directlyintostomach
Allorallyadministereddru.gsareexposedtodrugmetabolizing
enzymesintheintestinalwallandliver(wheretheyfirstreach
throughtheportalvein).
Druggivendirectlyintothestomachandintestinestillhavetopass
throughfirstpassmetabolismintheintestinalwallandinliver.
Approximately50%ofthedrugthatitisabsorbedfromtherectum
willbypasstheliver,thusreducingthehepaticfirst-passeffect.
Presystemicmetabolismlnthcgutandlivercanbeavoidedby
administeringthedrugthroughsublingual,transdermalorparenteral
(i.v/i.m/intradermal/s.c)routes.
Theeventoffirstpassmetabolismdiffersfordlfierentdrugsandis
animportantdatennlnantoforalbioavallabiltty.
Thehepaticfirst-passeffectcanbeavoidedtoagreatextentbythe
useofsublingualtabletsandtransdermalpreparationsandtoa
lesserextentbytheuseoFrectalsuppositories.
Subltngu.alabsorptionprovidesdirectaccesstosystemicnotportal-
velns.

Thetranscanalrouteofiersthesameadvantage.

69.Whichofthefollowingdyadsshow
clinicallysignificantdruginteractions:
a)Vancomycin--AmphotericinB
b)Rantidine-Atorvastatin
c)Warfarin--Aspirin
d)Allopurinol--Azathioprine
e)Aminoglycoside+Vancomycin
CorrectAnswer-A:C:D:E
Ans.A,Vancomycin--AmphotericinBC,Warfarin--
AspirinD,Allopurinol--Azathioprine&
E,Aminoglycoside+Vancomycin
Druginteraction:
Drugsmayinteract,butmostcanbecategorizedas
pharmacoKInetic(absorptiondistribution,metabolism,excretion),
phartnacodlmamla(aMitiveorantagonisticeffects),orcombined
interactions.
Eg:
Aminoglycosides,vancomycin,cyclosporineandothernephrotoxic
drugenhancetherenalimpairmentcausedbyamphotericinB
Allopurinolinhibitsthedegradationofi-mercaptopurineand
azathioprine;theirdosesshouldbereducedto?.
Enhanced'anticoagulantactionofwarfarin:
HighdosesoFsalicylateshavesynergistichypoprothrombinemic
action&alsodisplacewarfarinfromproteinbindingsite.

70.Trueaboutosmoticdiuretics:
a)Osmoticdiureticshavetheirmajoreffectinthedistal
convolutedtubule
b)Contraindicatedincongestiveheartfailure
c)CausesHyperkalemia
d)Increasesrenalbloodflow
e)None
CorrectAnswer-B:C:D
And.B,ContraindicatedincongestiveheartfailureC,Causes
Hyperkalemia&D,Increasesrenalbloodflow
OsmoticDiuretics:
Majoreffectintheproximaltubuleandthedescendinglimbof
Henle'sloop.
InhibitsTransportprocessesinthethinkAscLH.
Uses:
Usedtoincreasewaterexcretioninpreferencetosodiumexcretion.
ExtracellularVolumeexpansion-Effectcancomplicateheartfailure
andmayproducefloridpulmonaryedema-ContraindicatedinCHF.
Causesdehydration,Hyperkalemia,andHypernatremia
Headache,nausea&vomitingarecommonlyobservedinpatients
treatedwithosmoticdiuretics.

71.Trueabouteffectofsteroidintakein
inflammatoryconditions:
a)Proanaboliceffectonmuscles
b)Tglucoseinplasma
c)-vefeedbackoncorticotropin-releasinghormone(CRH)
production
d)Maycauseosteoporosis
e)None
CorrectAnswer-B:C:D
Ans.B,TglucoseinplasmaC,-vefeedbackoncorticotropin-
releasinghormone(CRH)production&D,Maycause
osteoporosis
Glucocorticoids:
GivenchronicallysuppressthepituitaryreleaseofACTH
Glucocorticoidsincreaseserumglucoselevel.
GlucocorticoidsstimulateRNAandproteinsynthesisintheliver,
theyhavecatabolicandantianaboliceffectsintymphofuland
connectivetissue,muscla,peripheralfat'andsldn.
CortisolhasanegativefeedbackonACTHandCRHproduction.

72.DrugthatcanpotentiateTorsadesde
pointes:
a)Amiodarone
b)Sotalol
c)Chlorpromazine
d)Cisapride
e)Aspirin
CorrectAnswer-A:B:C:D
Ans,A,AmiodaroneB,SotalolC,Chlorpromazine&D,Cisapride
TorsadesdePointes(VentricularTachycardia)
Antiarrhythmics:
Quinidine,procainamide,disopyramide,propafenone,amiodarone
Antimalarials:
Quinine,mefloquine,artemisinin,halofantrine
Antibacterials:
Sparfloxacin,moxifloxacin
Antihistamines:
Terfenadine,astemizole,ebastine
Antidepressants:
Amitriptylineandothertricyclics.
Antipsychotics:
Thioridazine,pimozide,aripiprazole,ziprasidone
Prokinetic:
Cisapride

73.WhichoftheFollowingarethegrounds
fordivorceforfemalesinIndia
a)Impotenceofmalepartner
b)Extramaritalaffairofmaleaprtner
c)Infertilityoffemale
d)Unemploymentofmalepartner
e)Ifshewasmarriedbeforetheageoffifteenandshewantto
renouncesthemarriagebeforesheattainseighteenyearsof
age
CorrectAnswer-A:B:E
Answer:(a)Impotenceofmalepartner,(b)Extramaritalaffairof
maleaprtner,(e)Ifshewasmarriedbeforetheageof
fifteen...(Ref:http://www.indidivorce.com/grounds-for-divorce-
in-india.html:Parikh7h/367,386.6`h/5.1.5.24]

Adulteryisconsideredasanoffenceagainstmarriagebyboththe
PenallawandtheMatrimoniallawinIndiaandanyonecommitting
anadulterousactcanbepunishedunderlaw"
Impotencyandinfertilityarecompletelydistinctterms.Unlike
impotency,infertilitycannotbegroundsfordivorce,theBombayhigh
court(HC)ruledonMonday.Jul24,2012.
HinduMarriageAct,1955?Adultery,Cruelty,Desertion,Mental
disorder,Leprosy,VenerealDisease,ifhusbandhasindulgedin
rape,bestialityandsodomy,ifshewasmarriedbeforetheageof
fifteenandrenouncesthemarriagebeforesheattainseighteen
yearsofage.
Sterilefemaleisnotagroundfordivorce(Prematureejaculation,if

leadstoimpotencythengroundfordivorce,otherwisenotgroundfor
divorce.
Impotencyisagroundforannullingmarriage

74.Bitemarkmayaidin:,
a)Assessmentofage
b)Identificationofaccused
c)Assessmentofheight
d)Assessmentoftypeofteeth
e)CollectionofDNAsample
CorrectAnswer-B:C:E
Answer:(B,IdentificationofaccusedD,Assessmentoftypeof
teeth&E,Collectionof...[Ref:Ready33'?/98-100:Parikh7`h/83-
84]
Bitemarkscanpermitpreciseidentification
becausethe
alignmentofteethispeculiartoeachindividual.
Thefirststepinanalyzingthebiteistoidentifyitashuman.Animal
teethareverydifferentfromhumans'teeth,sotheyleavevery
differentbite-markpatterns.Next,thebiteisswabbedfor
DNA,
whichmayhavebeenleftinthesalivaofthebiter.The
dentistmustalsodeterminewhetherthebitewasself-inflicted.

75.Featureofgunshotinjuryis/are:
a)Gutterinjury
b)Greasecollar
c)Radiatingwound
d)Tissueopeningonoppositeend
e)alloftheabove
CorrectAnswer-E
Answer-E,alloftheabove[Ref:Reddy33rd/224,32"d/205,235;
Parikh7`h/249-53,6`"/4.43-44;ForensicAnthropologyby
StevenByers4"/256]
Theimpactofbulletsonbonemaycausefracturelinestoform.The
twotypeoffracturelinesaredistinguishableoncranialvault:
radiatingandconcentric.
Radiatingfracturelinesoriginatefromthesiteofimpactwherethey
moveoutwardinanydirection(thisisespeciallyseeninentrance
wound).
SmudgeRing/LeadRing/GreaseCollar!DirtCollarisduetothewipe
ofthesoftmetalofthebullet,ordirtpresentonit,orgreasecarried
fromthebarrelandisdepositedroundtheentrancewoundinternal
totheabradedcollar.
Gutterfracture:Theyareformedwhenpartofthethicknessofthe
boneisremovedsoastoformagutter,e.g.,inobliquebulletwound"
Bulletwound:Inexitwound,sizeisbiggerthanbullet.

76.Whichofthefollwingis/aretrue
regardingPerjury:
a)Wilfullygivingfalsestatementunderhe/sheeitherknowsor
believestobefalseordoesnotbelievetobetrue
b)S.190IPCdealswithperjury
c)Voluntarilygivingfalseevidenceunderoathwhichhe/sheeither
knowsorbelievestobefalseordoesnotbelievetobetrue
d)Thewitnessisliabletobeprosecutedforperjury
e)None
CorrectAnswer-A:C:D
Ans.(A)Wilfullygivingfalsestatementunderhe/sheeither
knowsorbelievestobefalseordoesnotbelievetobetrue
(C)Voluntarilygivingfalseevidenceunderoathwhichhe/she
eitherknowsorbelievestobefalseordoesnotbelievetobe
true(D)Thewitnessisliabletobeprosecutedforperjury


77.TrueaboutCorpusdelicti:
a)Medicalnegligence
b)Bodyofoffence
c)Itincludesbodyofthevictimandotherfactswhichare
conclusiveofdeathbyfoulplay
d)Theessenceofcrime
e)None
CorrectAnswer-B:C:D
Ans.(B)Bodyofoffence(C)Itincludesbodyofthevictimand
otherfactswhichareconclusiveofdeathbyfoulplay(D)The
essenceofcrime


78.Whichisnotmethodofcrimescene
examination:
a)Grid
b)Strip
c)Wheel
d)Composite
e)Pointtopoint
CorrectAnswer-D:E
Ans.(d)Compositeand(e)Pointtopoint
[www.universalclass.com/articles/law/processing-a-crime-
scene.]
Sixbasiccrimescenesearchpatterns-Stripmethod,Wheelmethod,
Spiralmethod,Zonemethod,GridmethodandLinemethod.
Theuseofanyoranumberofthesesearchmethodswillbe
determinedbythelocationandsizeoftheparticularcrimescene.

79.AllisaretrueaboutRotavirusinfection
except:
a)Mostcommonlyseeninadultof>30yearagegroup
b)Persontopersontransmissionmayoccur
c)Severityofdiseasedecreaseswitheachrepeatinfection
d)Commonestcauseofdiarrheaininfantsandchildren
e)Singleinfectionprovidelifelongimmunityagainstreinfection
CorrectAnswer-A:E
Answer:(a)Mostcommonlyseenin...(e)Singleinfection
provide...
[Ref:Ananthanarayan9th/560-61;Park23'223;Harrison
19th/1287-88;Jawetz27h/534-35;Greenwood16th/525-26]
Reinfectionsarecornnon,buttheseverifirofdkeasedzcreaseswith
eachrepeatintectlon.Therefore,severerotavirusinfectionsareless
commonamongolderchildrenandadultsthanamongyounger
individuals.
Rotadiarrheaisusuallyseeninchildrenbelowtheageoffiveyears,
butismostfrequentb/w6and24monthsofage.

80.Trueaboutpseudomonasaeruginose:
a)Notlysinedecarboxylasepositive
b)Oxidasepositive
c)Producepyocyaninpigment
d)Gram-negativebacilli
e)Has6-12flagella
CorrectAnswer-A:B:C:D
Answera)Notlysinedecarboxylasepositive(b)Oxidase
positive(c)Producepyocyaninpigment(d)Gram-negative
bacilli
[Ref:Ananthanarayan9th/314-16;Harrison19th/1042-43
;Jawetz27th/137-39;Greenwood16th/282,16]
Itispositiveintheindophenoloxidasetest,andisSimmon'scitrate
positive,1-argininedihydrolasepositive,1-lysinedecarboxylase
negative,and1-ornithinedecarboxylasenegative.
P.aeruginosa:Motilebyvirtueofoneortwopolarflagella"
Paeruginosaisanonfastidious,motile,gram-negativerodthat
growsonmostcommonlaboratorymedia,includingbloodand
MacConkeyagars.
TWooftheidentifyingbiochemicalcharacteristicsofpaeruginosa
areaninabilitytofermentlactoseonMacConkeyagarandapositive
reactionintheoxidasetest.

81.Whichofthefollowingis/areDNAviruses:
a)Herpesvirus
b)Hepadnaviridae
c)Parvovirus
d)Orthomyxoviridae
e)Enteroviruses
CorrectAnswer-A:B:C
Answer:(a)Herpesvirus,(b)Hepadnaviridae,(c)Parvovirus
[Ref:Ananthanarayan9th/428,439-40;Harrison19th/214e-1
;Jawetz27th/852]
herpesvirusesconsistsofarelativelylarge,double-stranded,linear
DNAgenomeencasedwithinanicosahedralproteincagecalledthe
capsid,whichiswrappedinalipidbilayercalledtheenvelope.
PartiallydsDNAcirculargenome,about3.2kb.
Parvovirusesarelinear,nonsegmented,single-strandedDNA
viruses,withanaveragegenomesizeof5-6kb.

82.TrueaboutZIKAvirus:
a)Belongtoflavivirus
b)Firstcasedetectedin1953inNigeria
c)RTPCRisusefulindiagnosis
d)Causesmacrocephaly
e)Maypresentswithconjunctivitis
CorrectAnswer-A:C:E
Answer:(a)Belongtoflavivirus,(c)RTPCRisusefulin
diagnosis,(e)Maypresentswithconjunctivitis(Ref:Harrison
19th/1314;www.cdc.gov;www.nytimes.corn]
ItisspreadmostlybythebiteofaninfectedAedesspecies
mosquitoes(A.aegptiandA.albopictus).T
Itcanbepassedtoapregnantwomantoherfetus.Infectionduring
pregnancycancausecertainbirthdefects..
Real-timereversetranscription-polymerasechainreaction(RTPCR)
testingshouldbeperformedonserumcollectedduringthefirsttwo
weeksaftersymptomonset.
There'snovaccineorspecifictreatmenttortlwdisease.Treatment
insteadfocusesonrelievingsymptomsandincludesrest,
rehydrationandmedicationsforfeverandpain.
Amaculopapularrash,conjunctivitis,myalgia,andarthralgiausually
accompanyorfollowthosemanifestations.

83.Parasitewhichinfectsthroughingestion
ofaquaticvegetation:
a)Fasciolahepatica
b)Fasciolopsisbuski
c)Paragonimuswestermani
d)Watsoniuswatsoni
e)Gastrodiscoideshominis
CorrectAnswer-A:B:D:E
Answer:(a)Fasciolahepatica,(b)Fasciolopsisbuski,(d)
Watsoniuswatsoni,(e)Gastrodiscoideshominis
[Ref:Paniker'sParasitology7th/151;ChatterjeeParasitology
13"/174;Harrison19th/245e-1]
modeofinfectionofFasciolahepatica'Thedefinitivehostsheep
and,man,getinfectionbyingestionofmetacercariaeencystedon
aquaticvegetation.
InfectiveformofFasclolopslsBush:Encystedmetacercariaeon
aquaticvegetarian.
SecondintermediatehostoffasciolopsisBusKiEncystmentoccurs
onaquaticplants,rootsofthelotus,bulbofwaterchestnutwhichact
assecondintermediatehost.
SecondintermediatehostofGastrodiscoideshominis:aquaticplant.
Thecercariaeencystonwaterpl.ants.Manandanimalsbecome
infectedbyfeedinguponvegetationsharbouringthemetacercaria".

84.whichofthefollowingdyadsofvector
withdiseaseis/arecorrectlymatched:
a)Ratflea-Endemictyphus
b)Sandflea-Orientalsore
c)Blackfly-Kafaazar
d)Cyclops-Dracunculus
e)Louse-ChagasDisease
CorrectAnswer-A:B:D
Answer:(a),(b)and(d)[RefPark23'/768;Paniker'sParasitology
7th/223]
RatfleaBubonicplague,endemictyphus,chiggerosis,
hymenolepisdiminuta.
SandflyKala-azar,orientalsore,sandflyfever,oroyafever.
CyclopsGuinea-wormdisease,fishtopeworm(D.lotus).

85.Allarefeaturesofscrubtyphusexcept:l
a)Blackeschar
b)Maculo-papularrash
c)Morecommoninruralareas
d)Ciprofloxacinisdrugofchoice
e)Tickbornedisease
CorrectAnswer-D:E
Answer:(d)Ciprofloxacinisisdrugofchoice,(e)Tickborne
disease
(Ref:Ananthanarayan9th/408;Harrison19th/1159;Park
23rd/300;MedicalmicrobiologybyGreenwood16th/372]
"Scrubtyphus:Onetypicalfeatureisthepunched-outulcercovered
withablackenedscab(eschar)whichindicatesthelocationofmite
bite"
"Scrubtyphus,:Mosttravel-acquiredcasesoccurduringvisitstorural
areasinendemiccountriesforactivitiessuchascamping,hikingor
rafting,buturbancaseshavealsobeendescribed.Tetracyclineis
drugofchoice"-Park23rd/300
""Scrubtyphus,originallyfoundinscrubjungles,hasalsobeen
identifiedinavarietyofotherhabitats,suchassandybeaches,
mountaindesertsandequatorialrain-forests"-Ananthanarayan
9th/408

86.Unlikenocardia,Actinomycosisis;
a)Facultativeanaerobes
b)Notacidfast
c)EndogenousCauseOfdisease
d)Environmentalsaprophyte
e)Growatwiderangeoftemperaturerange
CorrectAnswer-A:B:C
Ans.(a)Facultativeanaerobes,(b)Notacidfast,(c)
Endogenouscauseofdisease[Ref:Ananthonarayangh/j9l-
93;Jawetz2vh/295,198-99;Greenwoodl6h/221-22;Hanison
lN/l088
Actinomycetearefacultativeanaerobes,butoftenfailtogrow
aerobicallyonprimaryculture.Theygrowbestunderanaerobicor
microaerophilicconditionswiththeadditionof5-10%carbondioxide.
Facultativeanaerobes,Growat35-37?C,Oralcommensals,Non-
acidfastmycelia,Endogenouscauseofdisease

87.Allofthefollowingarecausedby
dermatophytesexcept:
a)Madurafoot
b)Athlete'sfoot
c)Athlete'sfoot
d)Favus
e)None
CorrectAnswer-A
Answer:(A)Madurafoot[Ref:Ananthanarayan9th/596-97;
Harrison19th/1358]
"Madurafoot(eumycetomaornadurantycosis)iscausedbyfungi-
scedosporium,madurellamycetomatisandM.grisea,acremonium
spp.,exophialaspp.,aspergillusspp.fusariumspp."
"Favus:Achronictypeofringworminwhichdensecrusts(scutula)
developinthehairfollicles,leadingtoalopeciaandscattering.
"Kerlon:Severeboggylesionswithmarkedinflammatoryreaction
thatsometimesdevelopsinscalpinfectionduetodermatophytes"

88.Whichofthefollowinghasleastminimum
infectivedose(MID)requiredforcausing
infection:

a)Salmonellatyphi
b)Campylobacterjejuni
c)Shigelladysentery
d)Vibriocholera
e)None
CorrectAnswer-C
Ans.(c)Shigelladysentery[Ref:Ananthanarayan
9th/287,295;Greenwood16th/261,252,289]
"Shigellacausebacillarydysentery.Infectionoccurbyingestion.The
minimuminfectivedoseislow:asfewasI0-I(Nbacilliarecapableof
initiatingthedisease,probablybecausetheysurvivegastricacidity
betterthanotherenterobacter"-Ananthanarayan.

89.Whichtypeofbacteriacannotsurvivein
absenceofoxygen:
a)Obligateaerobe
b)Facultativeanaerobes
c)Microaerophilic
d)ObligateAnaerobes
e)Facultativeaerobes
CorrectAnswer-A
Ans.(a)Obligateaerobe[Ref:Ananthanarayan50/24-
25;Greenwood16th/41]
Aerobicbacteriarequireoxygenforgrowth.Theymaybeobligate
aerobes
likethecholeravibrio,whichwillgrowonlyinthepresence
ofoxygen,
orfacultativeanaerobeswhichareordinarilyaerobicbut
canalsogrowintheabsenceofoxygen,thoughlessabundantly.
Mostbacteriaofmedicalimportancearefacultativeanaerobes.
Anaerobicbacteria,suchasclostridia,growintheabsenceof
o>rygenandtheobligateanaerobesmayevendieonexposureto
oxygen.
Microaerophilicbacteriaarethosethatgrowbestinthepresenceof
lowoxygentension.

90.AllaretrueaboutHINIinfluenzaexcept:
a)ZanamivircommonlygiventhroughIVroute
b)Fatalitymoreinsomehighriskgroup
c)RT-PCRisusedforinvestigation
d)WHOlatesttrivalentinfluenzavaccinecontainstwoinfluenzaA
subtypes(H3N2andH1N1)andoneinfluenzaBcomponent
e)CDClatestquadrivalentinfluenzavaccinecontainstwo
influenzaAsubtypes(H3N2andH1N1)andtwoinfluenzaB
component
CorrectAnswer-A
Answer(a)ZanamivircommonlygiventhroughIVroute[Ref:
Park23'd/l56-59;AnanthanarayanEh/4gg-504;Harrison1Eh/t
209-t4;KDT7h/802-03]

WHO:Itisrecommendedthattrivalentvaccinesforuseinthe2017
southernhemisphereinfluenzaseasoncontainthefollowing
1. AnA/Michigan/45/2015(H1N1)pdm09-likevirus;
2. AnA/HongKong/4801/2014(H3N2)-likevirus;and
3. AB/Brisbane/60/2008-likevirus.
RT-PCRprovidesthemosttimelyandsensitivedetectionofthe
infection
PandemicinfluenzaA(Hlnl)Treatment:oseltamiviradultoraldose
is75mgtwicedailyfor5days.Zanamivirdoseistwoinhalation(2x
5mg)twicedailyfor5days

91.Trueaboutserummarkerofinactive
carrierphaseofchronicHepatitisB:
a)HbsAg+ve
b)HbeAg+ve
c)Anti-HBeantibodypositive
d)LowlevelDNA
e)IncreasedALT
CorrectAnswer-A:C:D
Answer:(a)HbsAg+ve,(c)Anti-HBeantibodypositive,(d)
LowlevelDNA(Ref:Ananthanarayan9`h/543-48;Harrison
19th/2032-33,2007;Davidson22"?/950-52;Park23rd/215.1
ChronicHBVInfection:Inactivecarriersarepatientswithcirculating
hepatitisBsurfaceantigen(HBsAg),normalserum
aminotransferaselevels,undetectableHBeAg,andlevelsofHBV
DNAthatareeitherundetectableorpresentatathresholdof103
IU/mL.
Therelativelyreplicativephaseischaracterizedbythepresencein
theserumofHBeAgandHBVDNAlevelswellinexcessof103-104
IU/mL,sometimesexceeding109IU/mL;bythepresenceintheliver
ofdetectableintrahepatocytenucleocapsidantigens(primarily
hepatitisBcoreantigen[HBcAg]);byhighinfectivity;andby
accompanyingliverinjury.

92.Whichofthefollowingmechanismis/are
usedbybacteriatoescapehostdefence
mechanism:

a)Mycobacteriumtuberculosispreventintracellularkillingby
inhibitingphagolysosomeformation
b)StreptococcuspyogenesbyMprotein
c)Neisseriameningitidisbycapsularpolysaccharide
d)Staphylococcusaureusbyiron-regulatedoutermembrane
proteins
e)PolysaccharidecapsulesofH.influenzae
CorrectAnswer-A:B:C:E
Answer:(a)Mycobateriumtuberculosis...(b)Streptococcus
pyogenes...(c)Neisseiameningtitides...(e)Polysaccharide...
[Ref:Ananthanarayan9`'/350,212;
textbookofbacteriology.net/antiphago;Jawetz27h/158-65;
Greenwood16th/244]
Thebacteriasurviveinsideofphagosomesbecausetheyprevent
thedischargeoflysosomalcontentsintothephagosome
environment.Specifically,phagolysosomeformationisinhibitedin
thephagocyte.
ThisisthestrategyemployedbySalmonella,M.
tuberculosis,Legionella
andchlamydiae.
Survivalinsidethephagolysosome-Mycobacteria(including
M.tuberculosisandMycobacteriumleprae).
MproteinandfimbriaeofGroupAstreptococciSurface
slime
(polysaccharide)producedasabiofilmbyPseudomonas
aeruginosa


MproteinandfimbriaeofGroupAstreptococciSurface
slime
(polysaccharide)producedasabiofilmbyPseudomonas
aeruginosa
antiphagocyticsubstancesonbacterialsurfaces
include:PolysaccharidecapsulesofS.pneumoniae,Haemophilus
influenzae,Treponemapallidum
andKlebsiellapneumoniae

93.TrueaboutGasgangrene:
a)Onsetisusuallyacute
b)Painlesscondition
c)Woundisswollen
d)Atfirstwoundisduskyorred,laterbecomespale
e)Causedbygram+veorganism
CorrectAnswer-A:C:E
Ans:(a)Onsetisusuallyacute,(c)Woundisswollen,(e)
Causedbygram+veorganism
[Ref:Davidson22''d/305;Land826`/57;;Harrison19th/990-95;
Ananthanarayan9th/257-59;Jawetz27th/186-87;Greenwood
16th/231-35]
Gasgangrene(clostridia)myonecrosis)isdefinedasacuteinvasion
ofhealthylivingmuscleundamagedbyprevioustrauma,andismost
commonlycausedbyC.perfringens.
Severepain,crepitus,brawnyindurationwithrapidprogressionto
skinsloughing,violaceousbullae,andmarkedtachycardiaare
characteristicsfoundinthemajorityofpatients.
TraumaticgasgangreneC.perfringensmyonecrosis(gasgangrene)
isoneofthemostfulminantgram-positivebacterialinfectionsof
humans.
Theinfectionischaracterizedbythesuddenonsetofexcruciating
painattheaffectedsiteandtherapiddevelopmentofafoul-smelling
woundcontainingathinserosanguineousdischargeandgas
bubbles.
Thewoundproducesathin,brown,sweet-smellingexudate,in
whichGramstainingwillrevealbacteria.


94.Allaretrueaboutdenguevirusexcept:
a)Belongtoflaviviridae
b)TypeDEN4ismostcommoninIndia
c)Mainvectorsareaedesaegyptiandaedesalbopictus
d)VirushaspositivesenseRNA
e)VectorissensitivetoDDT
CorrectAnswer-B
Ans:(b)TypeDEN4ismostcommon...(Ref:Park23'/246-56;
Ananthanarayan9th/523;Harrison19th/1318-19;Jawetz27th/552-
541
BelongtogenusflaviviruswithpositivesenseRNA(Harrison
19th214e-1table)
Allthe4serotypesi.edengu1,2,3and4havebeenisolatedinIndia
butatpresentDENV-1andDENV-2serotypesarewidespread,
Vector-AedesaegyptiandAedesalbopictusarethetwomost
importantvectorsofdengue

95.Whichofthefollwingis/aretrueabout
Dengufever:
a)PositiveTourniquettestmeansmorethan10petechiaeper
squareinch
b)Causedbyflavivirus
c)Aedesaegypticusandalbopictusaremostimportantvectorin
India
d)IgM/IgGratiosmaybeusedtodistinguishprimaryfrom
secondaryinfection
e)Novaccineavailableatpresent
CorrectAnswer-A:B:C:D:E
Answer:A,PositiveTourniquettestmeans...B,Caused
by...C,Aedesaegypticusandalbopictus...D,IgM/IgGratiosmay
beused...E,Novaccineavailable...(Ref.:Park23'/246-56;
Davidson22'/323,-Ananthanarayan9th/523;Harrison19th/1318-
19;Jawetz27h/552-54]
Positivetorniquettest(i,e.10ormorepetechiaepersquareinch)is
mostcommonhemorrhagicphenomenon.InDHFthetestusually
givesadefinitepositivewith20petechiaeormore-Park23'd/249
Vaccine-Sofarthereisnosatisfactoryvaccineandnoimmediate
prospectofpreventingthediseasebyimmunization-Park23'/254
"ThediagnosisismadebyIgMELISAorpairedserologyduring
recoveryorbyantigen-detectionELISAorRT-PCRduringtheacute
phase"-Harrison19m/1318-19

96.Autoclaveis/areusedforsterilizationof:
a)Woodenmaterial
b)Metallicinstrument
c)Plastic
d)Glasswares
e)Fibro-opticbronchoscope
CorrectAnswer-B
Ans:(b)Metallicinstrument[Ref:Ananthanarayan50/37,30-32;
Greenwood16th/77-78;Chakraborty2nd/45-46;en.wikipedia.org]
Autoclavesinoperationtheatersisusedtosterilizesurgical
instrument,OTgarments,linen,gloves,masks,gownetc.However,
itisnotsuitableforplastics"-CommunityMedicinewithRecent
AdvancesbySuryakantha

97.Whichofthefollowingis/aretrue
abouttuberculosisinindiaexcept?
a)Indiahasapproximately1/41hofGloballoadofTB
b)MDR-TBamongnotifiednewpulmonaryTBpatientsisabout
5%
c)5%ofTBpatientsestimatedtobeHIVpositive
d)MDR-TBamongretreatmentcasesisabout15%
e)incidenceisaround2millionnewTBcasesannually
CorrectAnswer-D
Answer:D,MDR-TBamongretreatmentcasesisabout
15%(Ref:Park23rd/176-77;CommunityMedicinebyPiyush
Gupta1'x/192-97;CommunityMedicinewithRecentAdvances
bySuryakantha4"/364-70]
IndiaisthehighestTBburdencountryintheworldintermof
absolutenumberofincidentcasesthatoccureachyear.Itaccounts
forone-fourthoftheestimatedglobalincidentTBcasesin2013"
MDR-TBamongnotifiednewpulmonaryTBpatientswasabout
2.2%andamongretreatmentcaseswasabout15%
Currently,multidrug-resistantTBisaglobalconcernandis
encounteredin3%ofallnewcasesand12%ofretreatmentcases.
Approximately5%ofTBpatientsestimatedtobeHIVpositive-
PiyushGupta1"/194,Park23rd/177

98.Importanceoflepromintestareallexcept:
a)Onlyhasepidemiologicalsignificance
b)Prognosticvalue
c)Tellsaboutimmunitystatusofleprosypatients
d)Differentiatebetweendifferenttypesofleprosy
e)Predictivevalue
CorrectAnswer-A
Answer:(a)Onlyhasepidemiologicalsignificance(Ref:Park
23rd/320-21;CommunityMedicinewithRecentAdvancesby
Suryakantha4th/539-40
Thetesthaspredictivevalueaswell.Itgivesanindicationoftherisk
ofthediseaseamongcontactsofopencases.
Thetesthasanepidemiologicalvalueaswell.Itindicatesthe
incidenceandprevalenceofinfectionamongchildren.Inthefirst6
monthsoflife,mostchildrenareleprominnegative.Theybecome
positiveprogressivelyastheirageadvances.
Thetwodrawbacksthatstandinthewayofthistestbringusedfor
diagnosisare:(i)positiveresultsinnon-cases,and(ii)negative
resultsinlepromatousandnear-lepromatouscases

99.Aleprosypersonispresentedwith
involvementsofsuralandradialnerve.
Whichtypeofregimenyouwillgive:

a)MultibacillarytreatmentX9month
b)MultibacillarytreatmentX12month
c)MultibacillarytreatmentX15month
d)PaucibacillarytreatmentX6month
e)SingledosetreatmentofRifampicin,Ofloxacinand
Minocycline(ROM)
CorrectAnswer-B
Answer(b)MultibacillarytreatmentX12month[Ref:Park
23rd/323-24;CommunityMedicinebyPiyushGuptalst/282-83;
CommunityMedicinewithRecentAdvancesbySuryakantha
4th/535-45;NeenaKhanna5th/272-84]

Itisapureneuritictypeofleprosy.ItisacaseofMultibacillary
leprosyfortherapeuticpurpose(accordingtoWHOclassification,
morethanonenervetrunkinvolvementistermedasmultibacillary
fortreatmentpurpose.
manyconsiderthatpureneuriticleprosybelongstothe
paucibacillarygroupsinceallofthemareacid-fastbacillinegativeon
skinsmearsbydefinitionandaremostlyleprominpositive.
AccordingtopresentNLEPguidelinesinIndia,whenonenerve
trunkisinvolvedinleprosyitisconsideredaspaucibacillary,and
whenmorethanonenervetrunkisinvolved,itisconsideredas
multibacillaryfortherapeuticpurposes.

100.Whichofthefollowingistrueaboutdata
representation:
a)Histogramisusedforpresentationofdiscretedata
b)Randomdotsinscatterdiagram--nocorrelation
c)Pictogramisrepresentedbysmallpicturesorsymbols
d)Piechartisrepresentedbyquadrangularfigures
e)Regressiongraphissaidtobelinearwhentheincreaseor
decreaseinthevariablesremainsproportionalindifferent
subjects
CorrectAnswer-B:C:E
Answer:4.(b)Randomdotsinscatterdiagram...,(c)Pictogram
isrepresented...,(e)Regressiongraphissaidlinear...
[Ref:Park23rd/845-47;CommunityMedicinebyPiyushGupta
1st/652;CommunityMedicinewithRecentAdvancesby
Suryakantha4th/694-99,729-30]
Therewillbeasmanypointsasthereareindividualsinthe
observation.Whenallthepointareplotted,thediagramgivesthe
pictureofascatter.Hencethename'Scatterdiagram'(Dot
diagram).
Thedirectionofscatterhelpstodeterminethepresenceorabsence
oftheassociation.Ifthescattertakesthedirectionmidwaybetween
thetwoaxes,itsignifiespositiveassociation(correlation)
Ifittakesadirectionatrightanglestomidwayscatteritindicates
negativeassociation.
Ahaphazardscatterrepresentsneitherpositivenornegative
association.


101.TrueaboutProgramevaluationand
reviewtechnique(PERT):
a)Betterthancriticalpathmethodforsmallproject
b)RecurrentactivitiesisbettermonitoredthanCPM
c)Activitiesaredividedintosmallgoals
d)Mainobjectivetomonitorcost
e)Itisamanagementtechniquebetterfornon-researchactivities
thanCPM
CorrectAnswer-C
Answer:(c)Activitiesaredividedintosmallgoals(Ref:Park
23rd/872;CommunityMedicinebyPiyushGupta1st/783;
CommunityMedicinewithRecentAdvancesbySuryakantha
4'h/860]
PERT(ProgrammeEvaluationandReviewTechnique)isa
managementtechniquewhichmakespossiblemoredetailed
planningandmorecomprehensivesupervision.
Itaidsinplanning,schedulingandmonitoringtheproject;itallows
bettercommunicationb/wthevariouslevelsofmanagement;it
identifiespotentialproblems;itfurnishescontinuous,timelyprogress
reports;itformsasolidfoundationuponwhichtobuildanevaluation
andcheckingsystem
TheessenceofPERTistoconstructanArrowDiagram.The
diagramrepresentsthelogicalsequenceinwhicheventsmusttake
place

102.Whichofthefollowingis/aremorein
humanmilkthancowmilk:
a)Protein
b)Iron
c)Carbohydrate
d)Fat
e)Energy
CorrectAnswer-B:C
Answer:(b)Iron,(c)Carbohydate[Ref:Park23rd/630;
CommunityMedicinewithRecentAdvancesbySuryakantha
4th/620]


103.WhichofthefollowingIs/aretrue:
a)Serialinterval=gapintimeb/winvasionbyaninfectiousagent
andtheappearanceofclinicalfeature
b)Latentperiod=theperiodfromdiseaseinitiationtodisease
detectioninnon-infectiousdisease
c)Incubationperiod=timeb/wtheonsetoftheprimarycaseand
thesecondarycase
d)Generationtime=timeperiodbetweentheonsetoftheinfection
andthemaximuminfectivityofthehost
e)Communicableperiod=Itisaperiodduringwhichthereservoir
isinfectioustoothers
CorrectAnswer-B:D:E
Ans.B,Latentperiod=theperiodfromdisease...D,Generation
time=timeperiodbetween...andE,Communicableperiod=It...
[Ref:Park23rd/99-100;CommunityMedicinewithRecent
AdvancesbySuryakantha4th/281-82]

Thetermlatentperiodisusedinnon-infectiousastheequivalentof
incubationperiodininfectiousdisease.Latentperiodhasbeen
definedas"theperiodfromdiseaseinitiationtodiseasedetection.
Generationtimeisdefinedas"theintervaloftimeb/wreceiptof
infectionbyahostandmaximalinfectivityofthathost"
Ingeneral,generationtimeisroughlyequaltotheincubationperiod.
However,thesetwotermsarenotthesame
CommunicablePeriodisdefinedas"thetimeduringwhichan
infectiousagentmaybetransferreddirectlyorindirectlyfroman
infectedpersontoanotherperson,fromaninfectedanimaltoman,
orfromaninfectedpersontoananimal,includingarthropods


104.Teststocheckpasteurizationofmilk:
a)Phosphatasetest
b)Standardplatecount
c)Methylenebluetest
d)Nitricacidtest
e)Coliformcount
CorrectAnswer-A:B:C:E
Answer:(a)Phosphatasetest,(b)Standardplatecount,(c)
Methylenebluetest,(e)Coliformcount
(Ref:Park23rd/655;CommunityMedicinewithRecent
AdvancesbySuryakantha4th/186]
Nitricacidtestisdoneepidemicdropsy(argemoneoildetection)"-
Park23rd/658
Phosphatasetest:Thistestiswidelyusedtochecktheefficiencyof
pasteurization.Thistestisbasedonthefactthatrawmilkcontains
anenzymecalledphosphatasewhichisdestroyedonheatingata
temperaturewhichcorrespondscloselywiththestandardtimeand
temperaturerequiredforpasteurization
MethylenebluetestPasteurizationofMilkPark23rd/655
Itkillsnearly90%ofthebacteriainmilkincludingthemoreheat-
resistanttuberclebacillusandtheQfeverorganisms.Butitwillnot
killthermoduricbacterianorthebacterialspores

105.Allistrueaboutforeignbodyimpaction
inearexcept-
a)Objectslocatedmedialtoisthmusofcanalisdifficulttoremove
b)Syringingisusedforremovalofvegetativeforeignbody
c)Syringingusesroomtemperaturewater
d)Blunthookisusedtoremoveroundedforeignbody
e)GAispreferredinchildrentoremoveforeignbodies
CorrectAnswer-B
Answer-B.Syringingisusedforremovalofvegetativeforeign
body
Methodsofremovingaforeignbodyinclude:forcepsremoval,
syringingsuction,microscopicremovalwithspecialinstrumentsand
postauralapproach.
Foreignbodiesofvegetableoriginsuchasnuts,peasandbeans,
arehygroscopicandshouldnotbesyringed.
Noattemptsshouldbemadetoremovesmoothsphericalobjects
suchasbeadsbyforceps.
Beadswhichhaveadiameterlessthanthatoftheisthmuscanbe
syringed;largeronesarebetterremoved,withahook.

106.Allaretrueaboutepistaxisexcept:
a)Keisselbach'splexusissourcein90%cases
b)Ifanteriorpackingisleftinnoseformorethan48hrsantibiotic
coverageisgiven
c)Anteriornasalpackiseasytoinsertandlesstraumaticthan
balloontemponade
d)Trottermethodisfirstaidmethod
e)Cauteristaionisdoneinrefractorycasesundergeneral
anaesthesia
CorrectAnswer-C
Answer-C.Anteriornasalpackiseasytoinsertandless
traumaticthanballoontemponade
Keisselbach'splexus:Thisplexusisthecommonestsiteofbleeding
(90%ofcases)
Anteriornasalpacking-Prophylacticantibodiesshouldbeusedit
packisinaplaceformorethan24hours.
Aballoontamponademaybeusedasanalternativetoanterior
nasalpackingandThisislesstraumaticasitisbestsuitedfpr
epistaxis.
Trotter'smethod-Patientmayputinthesittingpositionwiththehead
bendingforwardswithmouthopen.
Nasalendoscopyassistedbipolarcauterizationundergeneral
anaesthesiamaybedonetocoagulatethebleederincase
ofepistaxlsisrefractorytoconservativemeasures.

107.TrueaboutSecretoryotitismedia:
a)TypeCtympanogrammaybeseeninearlystageofotitismedia
witheffusion
b)Flattympanogramispresent
c)Leadstoconductivedeafness
d)Presenceofcleftpalatereducesitschance
e)MostcommoncauseisEustachiantubedysfunction
CorrectAnswer-A:B:C:E
Answer-A,B,C,E,TypeCtympanogrammaybeseeninearly
stageofotitismediawitheffusion(B)Flattympanogramis
present(C)Leadstoconductivedeafness(E)Mostcommon
causeisEustachiantubedysfunction
Thisisaninsidiousconditioncharacterizedbyaccumulationofnon-
purulenteffusioninthemiddleearcleft.
Eustachiantube(ET)dysfunctionisconsideredthemajoretlologic
factorinthedevelopmentofmiddleeardisease.
TypeBtympanogram:Aflatordome-shaped,graph.Seeninmiddle
earfluidorthicktympanicmembrane.
TypeCtympanogram:SeeninEustachiantubeobstructionorearly
stageofotitismediawitheffusion.
Hearingloss-Hearinglossisofconductivetypeof20-40dB.
MalfunctioningofEustachiantube(causesincludepalataldefects
e.gcleftpalate,palatalparalysis)andincreasedsecretoryactivityof
middleearmucosa.

108.Whichcausereddishlesionontongue:
a)Medianrhomboidglossitis
b)Hairyleukoplakia
c)Lichenplanus
d)Geographictongue
e)Fordyce'sspots
CorrectAnswer-A:D
Answer-A,D,Medianrhomboidglossitis(D)Geographic
tongue
RedLesionsofOralCavitylesion-
Papillomas
Pemphigoid
Erythroplakia
Granular-celltumour
Epulides
Hemangioma

109.TrueStatementregardingwaxinear-
a)Syringingandinstrumentalmanipulationaregenerallydoneto
removeimpactedwax
b)Ifwaxishardandimpacted,ceromiilyticsubstancesisusedto
softenwax
c)Insyringingfluidisinjectedalongthelowerwallofthemeatus
d)Waxhasantibacterialproperty
e)None
CorrectAnswer-A:B:D
Answer-A,B,D,Syringingandinstrumentalmanipulationare
generallydonetoremoveimpactedwax(B)Ifwaxishardand
impacted,ceromiilyticsubstancesisusedtosoftenwax
(D)Waxhasantibacterialproperty
WaxhasacidicpHandisbacteriostaticandfungistatic.
Ifwaxistoohardandimpacted,toberemovedbysyringeor
instrument,itshouldbesoftenedbydropsof5%sodiumbicarbonate
inequalpartsofglycerineandwater.
Hydrogenperoxide,liquidparaffinoroliveoilmayalsoachieve
thesameresult.Commercialdropscontainingceruminolyticagent
paradichlorobenzene2%canalsobeused.
Waxisremovedeitherbyinstrumentalmanipulationorbysyringe.
Theauricleispulledupwardsandbackwardstostraightenoutthe
meatus,andthefluidisinjectedalongtheupperwallofthemeatus.

110.Trueaboutspasmodicdysphonia-
a)Aneurologicalproblem
b)Mostlypsychogenicinorigin
c)Hyperadductionofvocalcordmaybeseen
d)Botulinumtoxinrelivesspasm
e)Speechtherapyisbeneficial
CorrectAnswer-A:C:D:E
Answer-A,C,D,E,Aneurologicalproblem(C)Hyperadduction
ofvocalcordmaybeseen(D)Botulinumtoxinrelivesspasm
(E)Speechtherapyisbeneficial
"spasmodicdysphoniaisaneurologicaldisorderaffectingthevoice
musclesinthelarynx,orvoicebox.
Etiologyisunknownbutltisusuallystress-related.
Botulinumtoxinlnjectionintothelaryngealmuscleshasbeentriedin
thetreamentofspasticdysphonia.
Voicetherapyisusefultoimprovevoiceonlywhencombinedwith
injection

111.Feature(s)ofperitonsillarabscess:
a)Foulbreath
b)Hotpotatovoice
c)Shiftingofuvulainoppositeside
d)Difficultyinswallowingevenownsaliva
e)Alwayspresentsasb/1severepaininthroat
CorrectAnswer-A:B:C:D
Answer-A,FoulbreathB,HotpotatovoiceC,Shiftingofuvulain
oppositesideD,Difficultyinswallowingevenownsaliva
Clinicalfeaturesaredividedinto:?
General:Theyareduetosepticaemiaandresembleanyacute
infection.
Theyincludefever(upto104?F),chillsandrigors,generalmalaise,
bodyaches,headache,nauseaandconstipation.
Local:
Severepaininthroat.Usuallyunilateral.
Odynophagia.Itissomarkedthatthepatientcannotevenswallow
hisownsalivawhichdribblesfromtheangleofhismouth.Patientis
usuallydehydrated.
Muffledandthickspeech,oftencalled"Hotpotatovoice".
Foulbreathduetosepsisintheoralcavityandpoorhygiene.
Ipsilateralearache.ThisisreferredpainviaCNIXwhichsupplies
boththetonsilandtheear.
Trismusduetospasmofpterygoidmuscleswhichareinclose
proximitytothesuperiorconstrictor.

112.Trueaboutantrochoanalpolyp-
a)Startsasedemaofmaxillarysinusmucosa
b)Suppressedbysteroids
c)Comesoutviaaccessoryostiumandgrowsinthechoanaand
nasalcavity
d)Morecommoninadultsthanchildren
e)Commonlypresentsasunilateralnasalobstruction
CorrectAnswer-A:C:E
Answer-A,Startsasedemaofmaxillary...C,Comesoutvia
accessoryostium...E,Commonlypresentsasunilateral...
Age-Commoninchildren
Aetiology-Infection
Number-Solitary
Laterality-Unilateral
Origin-Maxillarysinusneartheostium
Growth-Growsbackwardstothechoana;mayhangdownbehind
thesoftpalate
Sizeandshape-Trilobedwithantral,nasalandchoanalparts.
Choanalpartmayprotrudethroughthechoanaandfillthe
nasopharynxobstructiongbothsides
Recurrence-Uncommon,ifremovedcompletely
Treatment-Polypectomy;endoscopicremovalorCaldwellLuc
operationifrecurrent.

113.Trueaboutcomponentofvision2020-
a)Cataractsurgeryshouldbeperformedatprimarylevel
b)Retinalsurgeryshouldbeperformedattertiarylevel
c)Needtodevelop10centreofexcellenceattertiaryleveland
100trainingcentreatadvancedtertiarylevel
d)Ophthalmianeonatorumisincludedinchildhoodblindness
e)Primaryvisioncentercoversapopulationof50000
CorrectAnswer-B:D:E
Answer-B,Retinalsurgeryshould...D,Ophthalmia
neonatorum...E,Primaryvisioncenter...
Attheprimarylevel,thehealthworkerscreensforcataractand,
reportsthosewithvisionlessthanalocallydeterminedguideline.
Atthesecondarylevel,cataractsurgeryshouldbeperfomedwith
equalemphasisonthequalityandquantityofsurgery
Atthetertiarylevelliestheprovisionoffacilitiesforsurgical
treatmentofcomplicatedcasessuchascongenitalcataract,
subluxatedlens,complicatedcataractsandcataractassociatedwith
systemicdiseases.
TheGovernmentofIndiahasadopted'Vision2020:RighttoSight'
under'NationalProgrammeforControlofBlindness'.
Targetdiseasesidentifiedforinterventionunder'Vision
2020'initiativeinIndiaincluded-

1. Cataract,
2. Childhoodblindness,
3. Refractiveerrorsandlowvision,
4. Cornealblindness,
5. Diabeticretinopathy,

6. Glaucoma,and
7. Trachoma(focal)
Childhoodblindness-
Commoncausesofchildhoodblindnessareophthalmia
neonatorum,injuries,congenitalcataract.
Eyecareinfrastructuredevelopment-
PrimarylevelVisionCentre:Thereisaneedtodevelop20000vision
centres.
Secondarylevel-secondarylevelpopulatianof500000
EachadvancedtertiaryLevelcenterofexcellencewillcatertoa
populationof50millions.

114.TrueaboutNodularepiscleritis-
a)CanbeassociatedwithSLE
b)Takelongertimetoresolvethandiffusevariety
c)Moresymptomaticthandiffusetype
d)Painless
e)Elevatedhardnodule
CorrectAnswer-A:B:C:E
Answer-A,CanbeassociatedwithSLEB,Takelongertimeto
resolvethandiffusevarietyC,Moresymptomaticthandiffuse
typeE,Elevatedhardnodule
Thisisabenigninflammatoryaffectationofthedeepsubconjunctival
connectivetissues,includingthesuperficial
sclerallamellae,andfrequentlyaffectsbotheyes.
Twotypesofpresentationsmayoccur:
1. simpleordiffuseepiscleritis;and
2. nodularepiscleritis.
Innodularepisclerittsacircumscribednoduleofdenseleucocytic
infiltrationwhichisishard,tenderandimmovable.
Nodularepisleritistendstobemoresymptomaticandtakeslongerto
resolve.
Nodularepiscleritisischaracterizedbyapinkorpurpleflatnodule.
Bothepiscleritisandscleritisaremainlyseeninadultscanbe
associatedwithotherconditionssuchasrheumatoidarthritisand
systemiclupuserythematosus(SLE).
Theremaybelittleornopain.

115.Whichtypeofcataractis/areassociated
withMyotonicdystrophy:
a)Posteriorsubcapsular
b)Anteriorsubcapsular
c)Nuclearcataract
d)Corticalcataract
e)Alloftheabove
CorrectAnswer-A
Answer-A.Posteriorsubcapsular
Myotonlcdystrophyisassociatedwith,posteriorsubcapsulartypeof
presenilecataract.Christmastreecataractistypicallyseeninthis
condition.

116.WhatistheWHOcriteriafordefining
blindness:
a)<3/60visionwithavailablecorrection
b)<6/60visionwithavailablecorrection
c)<3/60visionwithbestcorrection
d)<6/60visionwithbestcorrection
e)<3/60visionwithoutcorrection
CorrectAnswer-C
Answer-C.<3/60visionwithbestcorrection
AsperWHO,blindnessisdefinedasVisualacuityoflessthan3/60
inthebettereyewithbestpossiblecorrection.

117.TrueaboutAcanthamoebickeratitis-
a)Moreinthosewearingrigidgaspermeablethansoftcontact
lensesuser
b)Canoccurasopportunisticinfectioninpatientswithherpetic
keratitis
c)Canbeculturedonnon-nutrientagarenrichedwithE.coli
d)Painfulcondition
e)Radialkerato-neuritismayoccur
CorrectAnswer-B:C:D:E
Answer-B,Canoccurasopportunisticinfection..C,Canbe
culturedonnon-nutrient...D,PainfulconditionE,Radialkerato-
neuritismayoccur
Cornealinfectionwithacanthamoebaresultsfromdirectcorneal
contactwithanymaterialorwatercontaminatedwiththeorganism.
Itsoccurrenceisfrequentlyassociatedwiththewearingofsoft
contactlenses.
Acanthamoebakeratitiscanalsooccurasopportunisticinfectionin
patientswithherpetickeratitis,bacterialkeratitis,bullouskeratopathy
andneuroparalytickeratitis.
Symptoms-
Veryseverepain
Initiallesionsofacanthamoebakeratitisareintheformfineepithelial
andsubepithelialopacities,andradialkerato-neuritis.
Laboratorydiagnosis:Cultureonnon-nutrientagat(E.colienriched)

118.Trueaboutferritinlineineye:
a)Ferry'sLine-cornealepithelialironlineattheedgeoffiltering
blebs
b)StockersLine-Cornealepithelialironlineattheedgeof
pterygium
c)Hudson-Stahliline-visibleallaroundthebaseofconein
Keratoconus
d)Fleischer'sring:Horizonatlcornealepithelialironlineatthe
inferioronethirdofcorneaduetoaging
e)Coat'swhitering-Aformofirondepositatthelevelof
Bowman'slayerofcornea
CorrectAnswer-A:B:E
Answer-A,Ferry'sLine-cornealepithelial...B,StockersLine-
Cornealepithelial...E,Coat'swhitering-Aformofirondeposit
...
Ferry'sLine=cornealepithelialironlineattheedgeoffiltering
blebs.
StockersLine-Cornealepithelialironlineattheedgeofpterygium
Hudson-StahilLine=Horizontalcornealepithelialironlineatthe
inferioronethirdofcorneaduetoaging.
Fleischer'sring:VisibleallaroundthebaseofconeinKeratoconus
Coat'sring:remnantsofaforeignbody.Theremnantsarefineiron
depositsinthecornea.

119.Whichofthefollowingisthefeatureof
sodiumfluoresceinangiography
comparedtoindocyaninegreen
angiography:

a)Inchoroidalcirculationitpassesfreelyacrosstheendothelium
b)Diffusefreelythroughretinalcapillaries
c)Albuminbindingislessthanindocyaninegreen
d)Bind<50%toalbumin
e)Stimulatedbyalongerwavelengthoflight
CorrectAnswer-A:C
Answer-A,C,Inchoroidalcirculationitpassesfreelyacross
theendothelium(C)Albuminbindingislessthanindocyanine
green
Uponenteringthecirculation,approximately8O%ofthedye
moleculcsbindtoplasmaproteins.
Indocyaninegreen:bindsprimarily(95%)toalbumin.
Inthechoroidalcirculation,fluoresceinpassesfreelyacrossthe
endotheliumofthecapillariestotheextravascularspaces
Aphysiologicalbarriertothedyepresentsthepassageacross
Bruch'smembraneandtheintactretinalpignentepithelium.
Thefunduscamerahasamechanismtousebluelight(420-49Onm
wavelength)forexcitingthefluoresceinpresentinbloodvessels.

120.Trueaboutphthisisbulbi-
a)Sizeofeyedecreases
b)Removedbyenucleationoperationespeciallyifpainful
c)IOPincreasesinlatestage
d)CalcificationmayoccurinBowman'slayerofcornea
e)None
CorrectAnswer-A:B:D
Answer-A,B,D,Sizeofeyedecreases(B)Removedby
enucleationoperationespeciallyifpainful(D)Calcificationmay
occurinBowman'slayerofcornea
Itisthefinalstageendresultofanyformofchronicuveitis.
Asaresultofittheeyebecomessoft,shrinksandeventually
becomesasmallatrophicglobe(phthisisbulbi).
Commonlytheretinalpigmentedepitheliummayundergoa
metaplasialeadingtointraocularossification(calcification)inthe
end-stageofphthisisbulbi.
Enucleation+/-prosthesisinsertlonisperformedifthereis
associatedchronicpainorforcosmeticreasons.

121.Advantageofcontinuouscurvilinear
capsulorhexisovercanopener
technique-

a)Preferredmethodofanteriorcapsulotomyinphaco
emulsification
b)Morechancesofposteriorcapsularopacification
c)Keepsthenucleusinplace
d)Resistsradialtears
e)Helpsinstabilizingandcenteringthelensimplant
CorrectAnswer-A:C:D:E
Answer-A,C,D,E,Preferredmethodofanteriorcapsulotomy
inphacoemulsification(C)Keepsthenucleusinplace
(D)Resistsradialtears(E)Helpsinstabilizingandcentering
thelensimplant
Can-opener'stechnique.Initanirrigatingcystitome(orsimplya26
gaugeneedle,bentatitstip)isintroducedintotheanteriorchamber
andmultiplesmallradialcutsaremadeintheanteriorcapsulefor
360degree.
Condnuouscircularcapsulorhexis(CCC)-Recentlythisisthemost
commonlyperformedprocedure.Inthistheanteriorcapsuleistorn
inacircularfashioneitherwiththehelpofanirrigatingbent-needle
cystitomeorwithacapsulorrhexisforceps.
Can-openercapsulotomycanbeusedwithphacoemulsificotion.
CCCresistsradialtears
CCCstabilizesthenucleus.
CCCalsohelpsstabilizeandcentrethelensimplant.


122.Whichofthefollowingis/arefeatureof
aorticstenosis-
a)Durationbetweenonsetofsymptomanddeathisgenerally10-
15year
b)Anginaoccursmainlybecauseoffixedcoronarybloodflows
c)Noincreaseincardiacoutputdespiteexercise
d)Ejectionsystolicmurmurradiatingtoneckmaypresent
e)Leftventricularhypertrophy
CorrectAnswer-C:D:E
Answer-C,D,E,Noincreaseincardiacoutputdespiteexercise
(D)Ejectionsystolicmurmurradiatingtoneckmaypresent
(E)Leftventricularhypertrophy
Theaveragetimetodeathaftertheonsetofvarioussymptoms
isasfollows:
anginapectoris,3years
syncope,8years
dyspnea,2years
congestiveheartfailure,1.5-2years.
Mildormoderatestenosis:
usuallyasymptomatic
Exertionaldyspnoea
Angina
Exertionalsyncope
Suddendeath
Episodesofacutepulmonaryoedema
Signs

Ejectionsystolicmurmur
Slow-risingcarotidpulse
Thrustingapexbeat(LVpressureoverload)
Narrowpulsepressure
Signsofpulmonaryvenouscongestion(e.g.crepitations)
ThemurmnurofASischaracteristicallyanejection(mid)systolic
murmur.
TheLVbecomesincreasinglyhypertrophiedandcoronarybloodflow
maythenbeinadequate.

123.Trueaboutorganophosphate-induced
delayedpolyneuropathy:
a)Usuallyoccursafter2-3monthofacuteexposure
b)Involvesbothsensoryandmotornerve
c)Steroidisusedfortreatment
d)Incompleterecovery
e)None
CorrectAnswer-A:B:D
Answer-A,B,D,Usuallyoccursafter2-3monthofacute
exposure(B)Involvesbothsensoryandmotornerve
(D)Incompleterecovery
Organophosphate-induceddelayedpolyneuropathy(OPIDN)isa
rarecomplicationthatusuallyoccurs2-3weeksafteracute
exposure.Itisamixedsensory/motorpolyneuropathy.
Recoveryisoftenincomplete

124.UnlikeChildPughscoring,MELDscore
have:
a)Bedsideassessmenteasy
b)Prothrombintimeexpressedasinternationalnormalizedratio
(INR)
c)Serumcreatinineestimation
d)Fourcomponentisusedinscoring
e)Albuminlevelestimation
CorrectAnswer-B:C
Answer-B,Prothrombintimeexpressedasinternational
normalizedratio(INR)and(C)Serumcreatinineestimation
Thisscoreiscalculatedironthreenoninvasiuevariables:the
prothrombintimeexpressedastheinternationalnormalizedratio
(INR),theserumbilirubinlevel,andtheserumcreatinine
concentration
MELDiscurrentlyusedtoestablishprioritylistingforliver
transplantation.

125.Riskfactorsassociatedwithhealthcare
associatedpneumonia(HCAP)-
a)Acutecarehospitalizationforatleast2daysinthepreceding
90days
b)Homeinfusiontherapy
c)Immunosuppressivediseaseorimmunosuppressivetherapy
d)Antibiotictherapyinthepreceding90days
e)Hospitalizationfor>48h
CorrectAnswer-A:B:C:D:E
Answer-(A)Acutecarehospitalizationforatleast2daysinthe
preceding90days(B)Homeinfusiontherapy
(C)Immunosuppressivediseaseorimmunosuppressive
therapy(D)Antibiotictherapyinthepreceding90days
(E)Hospitalizationfor>48h
Acutecarehospitalizationforatleast2daysinthepreceding90
days
Residenceinanursinghomeorextendedcarefacility
Homeinfusiontherapy,includingchemotherapy,withinthepast30
days
Long-termdialysiswithinthepast30days
Homewoundcare
Familymemberwithaninfectioninvolvingamultipledrugresistant
pathogen
Immunosuppressivediseaseorimmunosuppressivetherapy

126.Allarefeature(s)ofsarcoidosisexcept:
a)HighCD4:CD8ratio
b)Hypercalciuriaandhypercalcimiamaybepresent
c)fSerumlevelsofangiotensin-convertingenzyme(ACE)
d)Schaumanandasteroidbodiesarepathognomic
e)None
CorrectAnswer-D
Answer-D.Schaumanandasteroidbodiesarepathognomic
Hypercalcemiaand/orHypercalciuriaoccursinabout10%of
sarcoidosispatients.
Bronchoalveolarlavagefluidlnsarcoidosisisusuallycharacterized
byanincreaseinlymphocyteandahighCD4/CD8ratio.
"Schaumanandasteroidbodies-althoughcharacteristic,thesecells
arenotpathognomicofsarcoidosisbecausetheymaybe
encounteredinothergranulomatousdiseases.
Thegranulomaisthepathologichallmarkofsarcoidosis.
"Serumlevelsofanglotensin-convertingenzyme(ACE)canbe
helpfulinthediagnoslsofsarcoidosis.

127.Transudativepleuraleffusionoccursin:
a)Urinothorax
b)Dresslersyndrome
c)Nephroticsyndrome
d)Myxedema
e)Congestiveheartfailure
CorrectAnswer-A:C:D:E
Answer-(A)Urinothorax(C)Nephroticsyndrome
(D)Myxedema(E)Congestiveheartfailure
Congestiveheartfailure
Cirrhosis
Pulmonaryembolization
Nephroticsyndrome
Peritonealdialysis
Superiorvenacavaobstruction
Myxedema
Urinothorax

128.Trueaboutprimarysclerosing
cholangitis:
a)Involvesonlyintrahepaticbileduct,notextrahepaticbileduct
b)AssociatedwithInflammatoryboweldisease
c)Causesmacronodularcirrhosis
d)Periductalfibrosisofsmallerbileducts
e)None
CorrectAnswer-B:D
Answer-(B)AssociatedwithInflammatoryboweldisease
(D)Periductalfibrosisofsmallerbileducts
PSCischaracterizedbylnflammationandobliterativefibrosisof
lntrahepaticandextrahepaticbileductswlthdilationofpreserved
segments.
Inflammatoryboweldisease,particularlyulcerativecolitis,coexists
inapproximately70%ofindividualswithPSC.
Primarysclerosingcholangltiscausesmicronodularcirrhosis.
Followingchangesareseen-fibrosingcholangitis,periductalfibrosis,
dilationofinterveningbileductsandcholestatiswithfullblown
pictureofbiliarycirrhosis"

129.Trueaboutinsulinoma:
a)Encapsulated
b)Mostlymultiple
c)AssociatedwithMEN-I
d)Enucleationisthetreatmentofchoiceforbenigntumour
e)Histologysimilartonormal(3-cells
CorrectAnswer-A:C:D:E
Answer-(A)Encapsulated(C)AssociatedwithMEN-I
(D)Enucleationisthetreatmentofchoiceforbenigntumour
(E)Histologysimilartonormal(3-cells
Insulinomaisusuallysolitaryandwellencapsulatedtumour
10%aremultiple(alwaysassociatedwithMEN1)and10%are
malignant.
Microscopically,thetumouriscomposedofcordsandsheetofwell-
dffirentiatedBeta-cellswhichdonotdifferfromnormalcells.
Enucleationisthetreatmentofchoiceforbenigninsulinomas.

130.TrueaboutHypersensitivity
pneumonitis:
a)Occursduetoinorganicantigen
b)IncreasedCD8+Tcellsinbronchoalveolarlavage
c)Manifestsmainlyasanoccupationalandenvironmentdisease
d)Forsevereacutecases,oralsteroidsisgivenfor3-4weeks
e)Interstitialinflammatoryinfiltrateisseeninlung
CorrectAnswer-B:C:D:E
Answer-(B)IncreasedCD8+Tcellsinbronchoalveolarlavage
(C)Manifestsmainlyasanoccupationalandenvironment
disease(D)Forsevereacutecases,oralsteroidsisgivenfor3-
4weeks(E)Interstitialinflammatoryinfiltrateisseeninlung
Itismanifestedmainlyasanoccupationaldisease,inwhich
exposuretoinhaledorganicagentsleadstoacuteandeventually
chronicpulmonarydisease.
Bronchoalveolarlavagespecimensalsoconsistentlydemonstrate
increasednumbersofbothCD4+andCDB+Tlymphocytes.
HistologyshowsevidenceofaninterstitialinflammatoryinfiItratein
thelung.
Inacutecases,prednisoloneshouldbegivenfor3-4weeks,starting
withanoraldoseof40mgperday.

131.AllaretrueaboutSjOgren'ssyndrome
execept-
a)Bilateralparotidglandenlargement
b)Parotidglandenlargementmaybepainful
c)Xerostomiamaypresent
d)Progressiontolymphoma
e)Malesareaffectedmorethanfemales
CorrectAnswer-E
Answer-E.Malesareaffectedmorethanfemales
Sjogren'ssyndromeisanautoimmunedisorderassociatedwith
parotidglands.
Itaffectswomenmore(40-60years)
ClinicalFeatures-
Dryeyes(keratoconjuctivitissicca)
Xerostomia
Vaginaldryness
Raynaud'sphenomenon
Lymphoma
Splenomegaly

132.FeatureofFelty'ssyndromeis/are-
a)Seropositiveforrheumatoidfactor
b)Splenomegaly
c)Longstandingrheumatoidarthritis
d)Neutrophilia
e)Keratoconjunctivitissicca
CorrectAnswer-A:B:C:E
Answer-(A)Seropositiveforrheumatoidfactor
(B)Splenomegaly(C)Longstandingrheumatoidarthritis
(E)Keratoconjunctivitissicca
Feltysyndromeisapotentiallyseriousconditionthatisassociated
withrheumatoidarthritis.
Clinicalfeatures-
Lymphadenopathy
Vasculitis,
legulcers
Splenomegaly
Weightloss
Recurrentinfections
Skinpigments
Keratoconjunctivitissicca
SeropositiveforRF

133.MalignanciesassociatedwithAIDS-
a)PrimaryCNSlymphoma
b)Cervicalcancer
c)Kaposisarcoma
d)Ovariancancer
e)Endometrialcancer
CorrectAnswer-A:B:C:D
Answer-(A)PrimaryCNSlymphoma(B)Cervicalcancer
(C)Kaposisarcoma(D)Ovariancancer
TheneoplasticdiseasesconsideredtobeAIDSdefiningconditions
areKaposi'ssarcomanon-Hodgkin'slymphoma,andinvasive
cervicalcarcinoma,ovariancarcinoma.

134.Malignancyassociatedwith
hypercalcemia:
a)Breastcancer
b)Smallcelllungcancer
c)Non-smalllungcancer
d)Prostatecancer
e)Multiplemyeloma
CorrectAnswer-A:C:D:E
Answer-A,BreastcancerC,Non-smalllungcancerD,Prostate
cancerE,Multiplemyeloma
Lungcarcinoma,breastcarcinoma,andmultlplemyelomaaccount
formorethan50%ofallcasesofmalignancy-associated
hypercalcemia.
Gastrointestinaltumarsandprostatecarcinomaarelesscommon
causesofhypercalcemia.

135.TrueaboutSevereCombined
Immunodeficiency(SCID):
a)Adenosinedeaminaseenzymemaybegivenfortreatment
b)Haematopoieticstemcelltransplant(HSCT)iscurative
c)MostcommoninheritanceisXlinkedrecessiveandautosomal
recessive
d)Lymphocytosisispresentinmostcases
e)Increasedriskofinfectionbypneumocystisjiroveci
CorrectAnswer-A:B:C:E
Answer-A,Adenosinedeaminaseenzyme...B,Haematopoietic
stem...C,Mostcommoninheritance...E,Increasedriskof
infection...
Themostcommonform,accountingfor5o%to6o%ofcases,X-
linkedandinheritedintheautosomalrecessivemode.
Adenosinedeaminasedeficiency:Thisthefirstimmunodeficiency
diseaseassociatedwithanenzymedeficiency.
PersonswithSCIDhavesevereinfectionsbyCandidaalbicans,
Pneumocystisjiroveci,Pseudomonas,cytomegalovirus,varicella.
HSCtransplantationisthemainstayoftreatment.

136.Proximalrenaltubularacidosis(RTA)
is/areassociatedwith:
a)Fanconianemia
b)Multiplemyeloma
c)Leadpoisoning
d)Sjogren'ssyndrome
e)SLE
CorrectAnswer-A:B:C
Answer-A,FanconianemiaB,MultiplemyelomaC,Multiple
myeloma
lnheritedFanconi'ssyndromeCystinosis.
HeavymetaltoxicitYLead,cadmiumandmercuryPoisoning
Wilson'sdisease
DrugsCarbonicanhydraseInhibitorslfosfamide
ParaproteinaemiaMyeloma
Amyloidosis
Hyperparathyroidism

137.Highaniongapacidosisis/areassociate
a)Lacticacidosis
b)Ethyleneglycolpoisoning
c)Aspirinoverdose
d)Diarrhea
e)Renaltubularacidosis
CorrectAnswer-A:B:C
Answer-A,LacticacidosisB,Ethyleneglycol
poisoningC,Aspirinoverdose
Lacticacidosis
Ketoacidosis(diabetic,alcoholic,starvation)
Toxins(ethyleneglycol,methanol,glycol,pyroglutamicacid)
Renalfailure(acuteandchronic)

138.Tubularproteinuriais/areseenin:
a)Multiplemyeloma
b)Wilsondisease
c)Leadpoisoning
d)Fanconisyndrome
e)None
CorrectAnswer-B:C:D
Answer-B,WilsondiseaseC,LeadpoisoningD,Fanconi
syndrome
Tubularproteinuriaoccursasaresultoffaultyreabsorptionof
normallyfilteredproteinsintheproximaltubule,suchasBeta2-
microglobulinandimmunoglobulinlightchains.
Causesincludeacutetubularnecrosis,toxiinjury(lead,
aminoglycosides),drug-inducedinterstitialnephritis,andhereditary
metabolicdisorders(WisondiseaseandFanconisyndrome).

139.AllaretrueCeliacdiseaseexcept-
a)Antiendomysialantibodyispresent
b)Oat,ryeandbarleycanbesafelygiven
c)Associatedwithdermatitisherpetiformis
d)Associatedwithgliadin
e)Noriskfordevelopmentofcancer
CorrectAnswer-B:E
Answer-B,Oat,ryeandbarleycanbesafelygivenE,Noriskfor
developmentofcancer
Itisaninflammatorydisorderofthesmallboweloccurringin
geneticallysusceptibleindividuals,whichresultsfrom
intolerancetowheatglutenandsimilarproteinsfoundinrye,barley
and,toalesserextent,oats.
Serumantibodies-IgAantigliadin,antiendomysial,andanti-tTG
antibodies-arepresent.
Celiacdiseaseisassociatedwithdermatitisherpetiformis(DH).
Themostimportantcomplicationofceliacdlseaseisthe
developmentofcancer.

140.RespiratoryfailuretypeIIis/areseenin-
a)Myastheniagravis
b)AcuteexacerbationinCOPD
c)AcutesevereAsthma
d)Pulmonaryedema
e)Pulmonaryembolism
CorrectAnswer-A:B:C
Answer-A,MyastheniagravisB,Acuteexacerbationin
COPDC,AcutesevereAsthma
Acutesevereasthma
AcuteexacerbationofCOPD
Upperairwayobstruction
Acuteneuropathies/paralysis
Narcoticdrugs
Primaryalveolarhypoventilation
Flailchestinjury

141.Photosensitivityis/arenotseenin-
a)Acuteintermittentporphyria
b)Variegateporphyria
c)Porphyriacutaneatarda
d)Congenitalerythropoieticporphyria
e)Erythropoieticprotoporphyria
CorrectAnswer-B
Answer-B.Variegateporphyria
Ferrochelatase-erythropoieticprotoporphyria
Protoporphyrinogenoxidase-Variegateporphyria
PBGdeaminase-acuteintemittent
Uroporphyrinogensynthetase-Congenitalerythropoieticporphyria
Uroporphyrinogendecarboxylase-porphyriacutaneatarda

142.Trueabout4thheartsound:
a)Lowpitch
b)Presentduringearlydiastole
c)Absentinatrialfibrillation
d)Producedintheventricleduringventricularfillingphase
e)Presentinsevereleftventricularhypertrophy
CorrectAnswer-A:C:D:E
Answer-A,LowpitchC,AbsentinatrialfibrillationD,Produced
intheventricleduringventricularfillingphaseE,Presentin
severeleftventricularhypertrophy
Fourthheartsounds(S4):
Lowpitched
Pre-systolicsoundproducedintheventricleduringventricularfilling
Producedduringsecondrapidfillingphase(beforeS1)
Bestheardwithbellofstethoscope.
Theright-sidedS4ispresentinpatientswithrightventricular
hypertrophysecondarytoeitherpulmonicstenosisorpulmonary
hypertension.

143.Trueaboutatrialflutter:
a)Narrow-complextachycardiaofupto150/min
b)Pwaveabsent
c)Associatedwith2:1,3:1or4:1AVblock
d)Besttherapyiscatheterablation
e)Occurduetomacrore-entrycircuitwithintherightatrium
CorrectAnswer-A:C:D:E
Answer-A,Narrow-complex...C,Associatedwith...D,Best
therapy...E,Occurduetomacrore-entry...
Atrialflutterischaracterisedbyalarge(macro)re-entrycircuit,
usuallywithintherightatriumencirclingthetricuspidannulus.
Theatrialrateisapproximately300/min,andisusuallyassociated
with2:7,3:1or4:1AVblock
Atrialfluttershouldalwaysbesuspectedwhenthereisanarrow-
complextachycardiaof150/min.
Forrecurrentepisodesofcommonatrialflutter,catheterablationof
thecavotricuspidisthmusabolishesthearrhythmiainover90%of
patient.

144.PositiveECGsign(s)ofischemiain
Treadmilltestis/are-
a)UpslopingdepressionoftheSTsegmentmVbelowbaseline
b)DownslopingdepressionoftheSTsegment>0.1mVbelow
baseline
c)JunctionalST-segment
d)Tachycardia
e)Ventricularprematurebeats
CorrectAnswer-B
Answer-B.DownslopingdepressionoftheSTsegment>0.1mV
belowbaseline
ThcischemiaST:-segmentresponegenerallyisdefinedasflator
downslopingdepressionoftheSTsegmant>O.1mV
belowbaseline(i.e.,thaPRsegnent)andlastinglongerthanO.08s.
UpslopingorjunctionalST-segmentchangesarenotconsidered
characteristicofischemiaanddonotconstituteapositivetest.

145.Whichofthefollowingis/areincludedin
managementofacuteischemicstroke-
a)UnfractionedHeparin
b)LMWH
c)Streptokinase
d)Aspirin
e)Recombinanttissueplasminogenactivator(rt-PA)
CorrectAnswer-D:E
Answer-D,AspirinE,Recombinanttissueplasminogenactivator
(rt-PA)
RecombinantTissueplasminogenActivator(RtPA)istheonly
thrombolyticagentthatisapprovedforthetreatmentofacute
ischaemicstroke.
Useofaspirinwithin48hofstrokeonsetreducedbothstroke
recurrenceriskandmortalitymininally.

146.Finding(s)Inhemolyticanemiais/are:
a)Increaseinconjugatedbilirubin
b)Increaseinunconjugatedbilirubin
c)Increaseinurineurobilinogen
d)Increaseinfaecalstercobilinogen
e)Increasedbilirubininurine
CorrectAnswer-B:C:D
Answer-B,IncreaseinunconjugatedbilirubinC,Increasein
urineurobilinogenD,Increaseinfaecalstercobilinogen
DecreaseHaemoglobin
IncreasedUnconjugatedbilirubin
Increaselactatedehydrogenase
IncreaseReticulocytes
Increaseurobilinogen

147.Trueaboutidiopathicthrombocytopenic
purpura:
a)Inchildren,itisusuallyanchronicdisease
b)Self-limitedcourseinacuteform
c)Inadults,itisamoreacutedisease
d)Immune-mediateddestructionofplatelets
e)None
CorrectAnswer-B:D
Answer-B,Self-limitedcourseinacuteformD,Immune-
mediateddestructionofplatelets
Itisanacquireddisorderinwhichthereisimmune-mediated
destructionofplateletsandpossiblyinhibitionofplateletrelease
fromthemegakaryocyte.
lnchildren,itisusuallyanacutediseasemostcommonlyfollowing
aninfection,andwithaselflimitedcourse.
Inadults,itisamorechronicdisease.

148.Trueaboutcoagulationdisorders-
a)InDICbothPTandaPTTincrease
b)HemophiliaCisaXlinkedrecessivecondition
c)FactorVIIIcanbegiveninhemophiliaB
d)HemophiliaAisinheritedasX-linkedrecessive
e)None
CorrectAnswer-A:D
Answer-A,InDICbothPTandaPTTincreaseD,HemophiliaAis
inheritedasX-linkedrecessive
HemophiliaA:InheritanceisX-linkedrecessive,leadingtoaffected
malesandcarrierfemales.
CommonfindingsincludetheprolongationofPTand/oraPTT;

149.Plexiformlesionisprominentinwhich
groupofpulmonaryhypertension-
a)Recurrentthromboemboli
b)Interstitiallungdiseases
c)FamilialpulmonaryHTN
d)Congenitalheartdiseasewithleft-to-rightshunts
e)Pulmonaryhypertensionassociatedwithhuman
immunodeficiency
CorrectAnswer-C:D:E
Answer-C,FamilialpulmonaryHTND,Congenitalheart
disease...E,Pulmonaryhypertensionassociated...
Plexiformlesionsaremostprominentinidiopathicandfamilial
pulmonaryhypertension,unrepairedcongenitalheartdiseasewith
lefttorightshuntsandpulmonaryhypertensionassociatedwith
humanimmunodeficiency.

150.Inwhichofthefollowingcondition,non-
hepaticsurgeryisassociatedwithmost
adverseoutcome:

a)Child-PughscoreB
b)Child-PughscoreC
c)Acuteviralhepatitis
d)Acutealcoholichepatitis
e)Chronicviralhepatitis
CorrectAnswer-B
Answer-B.Child-PughscoreC
Table3.Child-PughGradingSystem
Class
TotalPoints
A:well-compensateddisease
5-6
B:functionalcompromise-worseningdisease 7-9
C:decompensateddisease
10-15

151.Trueaboutmultiplesclerosis:
a)Periventricularinvolvementcan'tbeseenbyimagingstudies
b)Mayproducemasslesion
c)Autoimmuneinflammatorycondition
d)OligoclonalbandsmaybepresentinCSF
e)Spinalcordinvolvementmayoccur
CorrectAnswer-B:C:D:E
Answer-B,MayproducemasslesionC,Autoimmune
inflammatoryconditionD,Oligoclonalbandsmaybepresentin
CSFE,Spinalcordinvolvementmayoccur
"ElevatedIgGincerebrospinalfluidanddiscretebandsofIgG
(oligoclonalbands)arepresentinmanypatients.
ItisanautoimmunediseaseoftheCNScharacterizedbychronic
inflammationdemyelination,gliosis(scarring)andneuronalloss;the
coursecanberelapsing-remittingorprogressive.

152.Safesttransplantationapproachinliver
disease-
a)Directlytransplantingembryonicstemcellintheliver
b)Transplantingdonorhepatocytesintoliver
c)Transplantingmesenchymalstemcellfromadiposetissueto
liver
d)Injectingerythropoietinintobody
e)None
CorrectAnswer-C
Answer-C.Transplantingmesenchymalstemcellfromadipose
tissuetoliver
Today,autologous(fromthepatient)adiposetissuestemcellarethe
onlystemcellsthathavebeenusedclinicallyfortreatingliver
disease.
Manytrialshaveshownthatpatientswithlivercirrhosishave
benefittedfromautologousadiposetissuederivedmesenchymal
stemcells

153.Trueaboutpostoperativeileus:
a)Colonrecoversearlierthansmallintestine
b)Smallintestinemostcommonlyaffected
c)Adhesionismostcommoncause
d)Usuallyresolveswithin48-72hour
e)Radiographsshowdiffuselydilatedbowelwithairinthecolon
andrectum
CorrectAnswer-D:E
Answer-D,Usuallyresolveswithin48-72hourE,Radiographs
showdiffuselydilatedbowelwithairinthecolonandrectum
Ileusthatoccursimmediatelyaftersurgeryintheabsenceof
precipitatingfactorsandresolveswlthln2to4daysisreferredtoas
primaryorpostoperativeileus.
Postoperativeileusaffectsthestomachandcolonprimarily.
Afterlaparotomy,smallbowelmotilityretunwithinseveralhours,
gastricmotilitywithin24to48hours,andcolonicmotilityin48to72
hours.
Abdominalradiographsrevealdiffuselydilatedbowelthroughoutthe
intestinaltractwithairinthecolonantdrectum.

154.Trueaboutsecondarybacterial
peritonitis-
a)MarkedleukocytosiswithleftshiftoftheWBCstobandforms
b)Usuallycausedbypolymicrobialinfection
c)Maybeassociatedwithappendicularperforation
d)Associatedwithcirrhosisoftheliver
e)None
CorrectAnswer-A:B:C
Answer-A,Markedleukocytosiswithleft...B,Usuallycausedby
polymicrobial...C,Maybeassociatedwith...
Secondaryperitonitisdevelopswhenbacteriacontaminatethe
peritoneumasaresultofspillagefromanIntraabdominalviscus
Patientsarefebrile,withmarkedleukocytosisandaleftshiftofthe
WBCstobandforms.
Peritonealinfectionisusuallycausedbytwoormorebacterial
strains.

155.Correctstatementaboutintestinal
ischemia:
a)Watershedzonesareusuallyaffected
b)Arterialischemiclesionhasbetterdemarcationthanischemia
causedbyimpairedvenousdrainage
c)Cryptsmaybehyperproliferative
d)Microscopicexaminationmayshowcoagulativenecrosisofthe
muscularispropria
e)Surfaceepitheliumisnormal
CorrectAnswer-A:B:C:D
Answer-AWatershedzones...,B,Arterialischemiclesionhas
better...C,CryptsmaybehyperproliferativeD,Microscopic
examination...
Intestinalresponsestoischemiaoccurintwophases-

1. Theinitialhypoxicinjuryoccursattheonsetofvascular
compromise.
2. Reperfusioninjury,isinitiatedbyrestorationofthebloodsupplyand
itisatthistimethatthegreatestdamageoccurs.
Colonisthemostcommonsiteofgastrointestinalischemia,mucosal
andmuralinfarctionmayinvolveanylevelofthegutfromstomach
toanus.
Therebcoagulativenecrosisofthemuscularispropriawithin2to4
days,andperforationmayoccur.
Microscopicexaminationofischemicintestinedemonstratethe
characteristicatrophyorsloughingofsurfaceepithelium.
Cryptsmaybehyperproliferative.


156.TrueaboutColorectalcancer:
a)Rightsidecancerusuallypresentswithobstructivesymptoms
b)MaybeassociatedwithHNPCC
c)Left-sidedcancerpresentswithalterationinbowelhabit
d)Leftsidecolonismorecommonlyinvolved
e)All
CorrectAnswer-B:C:D
Answer-B,MaybeassociatedwithHNPCCC,Left-sidedcancer
presentswithalterationinbowelhabitD,Leftsidecolonismore
commonlyinvolved
Tumorsoftheleftsideofthecolonwhicharefarmorecommon.lt
usuallypresentwithachangeinbowelhabitorrectal
bleeding,whilemoreproximallesionstypicallypresentlaterwithiron
deficiencyanaemiaoramass.
HereditaryNonpolyposisColonCancer(Lynch'sSyndrome):Itis
characterizedbythedavelopmentofcolorectalcarcinomaatan
earlyage.
Tumorsoftheleftcoloncangraduallyoccludethelumen,causing
changesinbowelhabitswithalternatingconstipationandincreased
frequencyofdefecation.

157.Trueaboutthyroidcancer:
a)Follicularcancerismorecommoninradiationexposedpatient
b)Harthlecellcarcinomahasbetterprognosisthanfollicular
cancer
c)Follicularcancerhavebetterprognosisthanpupillarycancer
d)Insularvariantofpapillarycarcinomahasbetterprognosisthan
papillarycancer
e)MedullarycarcinomaisassociatedwithMEN-2b
CorrectAnswer-E
Answer-E,MedullarycarcinomaisassociatedwithMEN-2b
MedullarythyroidcarcinomainMEN2Bdevelopsearlierandismore
aggresivethaninMEN2A.
PapillaryCarcinomasarethemostcommonformofthyroidCancer.
Harthlecellcarcinomamayhaveaworseclinicalprognosis.

158.TrueaboutSchatzkiring:
a)Containtrueesophagealmuscle
b)Concentricsymmetricnarrowingofloweresophagus
c)Consistsofesophagealmucosaaboveandgastricmucosa
below
d)Dysphagiaisusuallytosolidfoods
e)Associationwithrefluxdisease
CorrectAnswer-B:C:D:E
Answer-B,Concentricsymmetricnarrowingofloweresophagus
CConsistsofesophagealmucosaaboveandgastricmucosa
below,D,DysphagiaisusuallytosolidfoodsE,Associationwith
refluxdisease
Schatzki'sringisathinsubmucosalcircumferentialringinthedistal
oesophagus,usuallyatthesquamocolumnarjunction.
Itconsistsofesophagealmucosaaboveandgastricmucosabelow.
Itdoesnothaveacomponentoftrueesophagealmusclethe
dysphagiaisusuallytosolidfoodsonlyandcomesonabruptlywith
nearlycompleteobstruction.
Thereisastrongassociatianwlthrefluxdisease.

159.Trueaboutintestinaltypeofgastric
cancer-
a)Incidencedecreasednowadays
b)AssociatedwithH.pyloriinfection
c)Morecommoninmalethanfemale
d)Betterprognosisthandiffuse
e)Morecommonthandiffusetype
CorrectAnswer-A:B:C:D
Answer-A,IncidencedecreasednowadaysB,Associatedwith
H.pyloriinfectionC,MorecommoninmalethanfemaleD,Better
prognosisthandiffuse
"Forunclearreason,theincidenceandmortalityratesforgastric
cancerhavedecreasedmarkedlyduringthepast65years.Gastric
cancerincidencehasdecreasedworldwidebutremainshighin
Japan,China,ChileandIreland"-Harrison16/e,p524
Morecommoninwomen.
Theprognosisbelessfavourable.
Intestinalmetaplaslaoccursduetopersistentirritationofthegastric
mucosa,mostcommonlyfromH.pyloriinfection.
TheLaurensystemseparatesgastricadenocarcinomaintointestinal
ordiffusetypesbasedonhistology.

160.CorrectstatementaboutIntussusception
inchildrenis/areallexcept-
a)Appendixisthemostcommonleadingpoint
b)Ileocolicisthemostcommonsite
c)Usuallypresentswithpainwithoutvomiting
d)Canbecorrectedbyairandbariumenema
e)Diagnosisisconfirmedonabdominalultrasound
CorrectAnswer-A:C
Answer-A,Appendixisthemostcommonleading
pointC,Usuallypresentswithpainwithoutvomiting
Mostintussusceptionsinchildrenareseenfromtwomonthstotwo
yearsofageintussusceptionissecondarytoapathologicallead
polnt,suchasaMeckel'sdiverticulum,entericduplicationcystor
evenasmallbowellymphoma.
Classically,apreviouslyhealthyinfantpresentswithcolickypainand
vomiting.

161.Non-absorbablesuture(s)is/are:
a)Silk
b)Catgut
c)Polypropylene
d)Polyester
e)Nylon
CorrectAnswer-A:C:D:E
Answer-A,SilkC,PolypropyleneD,PolyesterE,Nylon
Non-absorbablesuturesare:Silk,Linen,Surgical
Steel,Linen,Polyester,Polybutester,Polypropylene,polyethylene,
Nylon.

162.Correctstatementaboutsurgicalsite
infection-
a)CanoccursduetoenvironmentalflorainOT
b)Superficialinfectionrequirere-surgicalexplorationofwound
c)Doesnotoccurlaterthan14dayspost-operative
d)Shavingisbeneficialforprevention
e)Canoccurevenafter1yrincaseofimplantleftinsitu
CorrectAnswer-A:B
Answer-A,Canoccursduetoenvironmentalflorain
OTB,Superficialinfectionrequirere-surgicalexplorationof
wound
TheycanoccuranytimefromOto3Odaysaftertheoperationorup
to7yearafteraprocedurethathasinvolvedtheimplantationofa
foreignmaterial.
Preoperativehairremoval(clipping)shouldbedoneimmeditately
beforeanoPeration.

163.Regardingpre-operativeantibiotic
prophylaxis,whichofthefollowing
statement(s)is/arecorrect:

a)Shouldbestarted2daysbeforesurgery
b)Shouldbegivenhalfanhourpriortosurgery
c)Notneededincleansurgery
d)Shouldbegivenmorethan1hourbeforesurgery
e)None
CorrectAnswer-B
Answer-B.Shouldbegivenhalfanhourpriortosurgery
"Antibioticprophylaxisisindicatedformostcleancontaminatedand
contaminated.
Antibioticsstartedaslateas7to2hoursafterbacterial
contaminationaremarkedlylesseffective.

164.GradeIII(contaminated)woundis/are-
a)Appendicularperforation
b)Woundcontaminatedwithgrossfecalmaterialspillage
c)Woundinurinarytractwithoutunusualcontamination
d)Surgeryoveracleansite
e)Incisionthroughabscess
CorrectAnswer-B
Answer-B.Woundcontaminatedwithgrossfecalmaterial
spillage
Contaminatedwounds(classIII)includeopenaccidentalwounds
encounteredearlyafterinjury,thosewithextensiveintroductionof
bacteriaintoanormallysterileareaofthebodyduetomajorbreaks
insteriletechnique(e.g.,opencardiacmassage),grossspillageof
viscuscontentssuchasfromtheintestine,orincisionthrough
inflammed,albeitnonpurulenttissue.

165.Trueabouttesticularcancer-
a)Seminomaismorecommonthannon-seminoma
b)Cryptorchidismisassociatedwithseminoma
c)RetroperitonealLNdissectionisdoneforgradeIItesticular
tumour
d)Usuallypresentsaspainlesstesticularlump
e)Seminomasmetastasisemainlyviahaematogenousroute
CorrectAnswer-A:B:C:D
Answer-A,Seminomaismore
common...B,Cryptorchidism...C,RetroperitonealLN
dissection...D,Usuallypresents...
Seminomaisthemostcommontypeofgermcelltumour.
Amajorriskforthedevelopmentoftesticularcanceris
cryptorchidism.
Seminomasmetastasisemainlyviathelymphaticsand
haematogenousspreadisuncommon.
Usuallythepatientpresentswithapainlesstesticularlump.

166.Trueaboutobstructivejaundice-
a)Cholangiocarcinomacauseintermittentjaundice
b)Courvoisierlawrelatedtopancreaticheadcancer
c)Cholangitispresentswithfeverandjaundice
d)Increaseinurineurobilinogen
e)Increaseinfecalurobilinogen
CorrectAnswer-B:C
Answer-B,Courvoisierlawrelatedtopancreatichead
cancerC,Cholangitispresentswithfeverandjaundice
examinationoftendemonstratesclinicalsignsofjaundice,cachexia
isoftennoticeableandapalpablegallbladderispresentifthe
obstructionisinthedistalcommnonbileduct(Courvoisier'ssign).
StoneinCBD:Intermitentpain,intermitentfeverandlntermittent
jaundiceareclassicalofstoneinCBD.
Cholangitis:Themostcommonpresentationisfever,epigastricor
rightupperquadrantpain,andjaundiceandisknownasCharcot's
triad.
UrineUrobilinogen:Absent
StoolsSterocobilinogen-Absent

167.Truestatementregardingsurviving
sepsisguideline:
a)ActivatedproteinCisuseful
b)qS0FAshouldbeassessedincaseofprolongstayofpatientin
ICU
c)SuggestagainsttheuseofIVimmunoglobulinsinpatientswith
sepsis
d)Recommendtheuseoferythropoietinfortreatmentofanemia
associatedwithsepsis
e)Recommendempiricbroad-spectrumtherapy
CorrectAnswer-B:C:E
Answer-B,qS0FAshouldbeassessed...C,Suggestagainstthe
useofIV...E,Recommendempiric...
RecombinanthumanactivatedproteinCwascompletelyomitted
fromthe2Ol2guidelines.
"AhigherSOFAscoreisassociatedwithanincreasedprobabilityof
mortality.
Patientswithsuspectedinfectionwhoarelikelytohaveaprolonged
ICUstayortodieinthehospitalcanbepromptlyidentifiedatthe
bedsidewithqSOFA.
Recommendempiricbroad-spectrumtherapywithoneormore
antimicrobialsforpatientspresentingwithsepsisorsepticshockto
coveralllikelypathogens.
AgainstusingIVhydrocortisonetotreatsepticshockpatientsif
adequatefluidresuscitationandvasopressortherapyareableto
restorehemodynamicstability.

Recommendagainsttheuseoferythropoietinfortreatmentof
anemiaassociatedwithsepsis.

168.Truestatementregardingsurviving
sepsisguideline:
a)ActivatedproteinCisuseful
b)qS0FAshouldbeassessedincaseofprolongstayofpatientin
ICU
c)SuggestagainsttheuseofIVimmunoglobulinsinpatientswith
sepsis
d)Recommendtheuseoferythropoietinfortreatmentofanemia
associatedwithsepsis
e)Recommendempiricbroad-spectrumtherapy
CorrectAnswer-B:C:E
Answer:(b)qS0FAshouldbeassessedin...,(c)Suggestagainst
theuseofIV...,(e)Recommendempiricbroad-...
[Ref:Harrison19th/1758;wwwfoamcast.org/2017/01/19;
jamanetwork.com/journakSchwartz9th/128-29]


169.Trueaboutmanagementofpepticulcer-
a)Vagotomy-decreasesacidsecretion
b)Earlydumpingsyndromeoccursduetohypoglycaemia
c)Latedumpingconsistsofabdominalandvasomotorsymptoms
d)BillrothIgastrectomyhashighrateofmorbidityandmortality
e)Highlyselectivevagotomyhaslowincidenceofsideeffects
CorrectAnswer-A:D:E
Answer-A,Vagotomy-decreasesacidsecretionD,BillrothI
gastrectomyhashighrateofmorbidityandmortalityE,Highly
selectivevagotomyhaslowincidenceofsideeffects
Highlyselectivevagotomyhasmostsatistactoryoperationfor
duodenalulceration,withalowincidenceofsideeffects.
BillrothIcarrieswithitthemorbidityandmortalityassociatedwith
anygastricresection
Invagotomy,sectionofthevagusnerves,whfuharecritiaally
involvedlnthesecretionofgastricacid.,reducesthemaximalacid
outputbyapprorximately5OPercent.
EarlydumpingconsistsofabdominalandvasomotorsymPtoms.
Latedumpingisreactivehypoglycaemia.Thecarbohydrateloadin
thesmallbowelcausesariseintheplasmaglucose,which
inturn,causesinsulinlevelstorise,causingasecondary
hypoglycemia.

170.Trueaboutsurgicalocclusivedressing-
a)Maintainmoistureinwound
b)Providesmechanicalsupport
c)Preventsmicrobialentry
d)WideMeshishelpful
e)Usedforhighlyexudativewounds
CorrectAnswer-A:B:C
Answer-A,MaintainmoistureinwoundB,Providesmechanical
supportC,Preventsmicrobialentry
Wounddressingscanbecategorizedintofourclasses:
nonadherentfabrics;
absorptivedressings;
occlusive
dressings;
creams,
ointmentsandsolutions
"Occlusivedressingclassprovidesmoistureretention,mechanial
protectionandabarriertobacteria.
Widemeshgauzeiscompositionofabsorptivedressing.

171.Whichofthefollowingnippledischarge
ismostprobablyphysiological-
a)B/Lspontaneousdischarge
b)B/Lmilkydischargewithsqueezingfrommultipleducts
c)U/Lbloodydischarge
d)U/Lbloodydischargewithsqueezingfromasingleduct
e)U/Lspontaneousserousdischarge
CorrectAnswer-B
Answer-B,B/Lmilkydischargewithsqueezingfrommultiple
ducts
Nippledischargeisclassifiedaspathologicifitisspontoneous,
unilateral,blaody,serous,clear,orassociatedwlthamass.
Physiologicaldischargeisusuallybilateral,involvesmultipleducts,
andisassoctatedwithnipplestimulationorbreastcompression.
Unilateral,localizedtoasingleduct.
Themostcommoncauseofspontaneousnippledischargefroma
singleductisasolitaryintraductalpapilloma.

172.TrueaboutFatnecrosisinwomen:
a)Historyoftraumacanbeelicitedin>90%cases
b)Morecommoninlactatingwomen
c)Canbeeasilydifferentiatedfrommalignancyonmammography
d)Nomalignant
e)None
CorrectAnswer-D:E
Answer-D,NomalignantE,None
Traumaispresumedtobethecause,thoughonlyabout5O%of
patientsgiveahistoryofinjury.
Fatnecrosisisararelesionofthebreast.
Acarcinoma,evendisplayingskintetheringandnippleretraction,
andbiopsyisrequiredfordiagnosis.
Fatnecrosiscanmimiccancerbyproducingapalpablemassora
densltyonmamtnographythatmaycontaincalcifications.

173.Trueaboutomphaloceleisall,EXCEPT:
a)Umbilicalcordinsertsintothesac
b)Incidenceofapproximately1in5000livebirths
c)Abdominalwalldefectmeasures4cmindiameter
d)Visceracoveredbyperitoneum
e)Noneoftheabove
CorrectAnswer-E
Answer-E.Noneoftheabove
Omphalocelereferstoacongenitaldefectoftheabdominalwallin
whichthebowelandsolidvisceraarecoveredbyperitoneumand
amnioticmembrane.
Theumbilicalcordinsertsintothesac.
Theabdominalwalldefectmeasures4cmindiameter.
Omphalocelehasanincidenceofapproximately1in5000live
births.
Theabdominalviscera(commonlyliverandbowel)arecontained
withinasaccomposedofperitoneumandamnionfromwhichthe
umblicalcordarisesattheapexandcenter.

174.Feature(s)ofoesophagealatresiawith
trachea-oesophagealfistula:
a)Droolingofsaliva
b)Associatedwitholigohydramnios
c)Nasogastrictubecannotpassesintostomach
d)MaybeassociatedwithVACTERLanomalies
e)Abdominaldistension
CorrectAnswer-A:C:D:E
Answer-A,C,D,E,Droolingofsaliva(C)Nasogastrictube
cannotpassesintostomach(D)Maybeassociatedwith
VACTERLanomalies(E)Abdominaldistension
Affectedinfantspresentsoonafterbirthwithdroolingandcyanotic
episodesonattemptingtofeed.
Theremayhavebeenpolyhydramniosduetofailuretoswallow
amnioticfluid.
Thediagnosisisconfirmedwhenanasogastrictubegoesnofurther
thantheupperoesophagealpouchonthechestx-rayand
abdominalgassignifiesthetracheo-oesophagealfistula.
TheVACTERLascociation(uertebral,anorectal,cardiac,
tracheoesophageal,renal,andlimbanomalies)ispresentof25%of
cases.
Astheneonatecoughsandcries,airistransmittedthroughthe
fistulaintothestomach,resultinginabdominaldistension.

175.Correctstatement(s)abouthypospadias
is/are:
a)Urethralopeningonventralaspect
b)Chordaeondorsalaspect
c)Maybeassociatedwithpeniletorsion
d)Proximalvarietiesaremorecommon
e)Couponsspongiosumisdeficient
CorrectAnswer-A:C
Answer-A,C,Urethralopeningonventralaspect(C)Maybe
associatedwithpeniletorsion
Thereisincompletedevelopmentoftheprepuce,calledadorsal
hood,inwhichtheforeskinisonthesidesanddorsalaspectofthe
penileshaftandabsentventrally.
Thereisavariabledegreeofchordae(aventralcurvatureofthe
penismostapparentonerection)
"Peniletorsionisafairlycommoncongenital(presentfrombirth)
conditionthatcanaffectanymaleinfant.

176.TrueaboutBronchogeniccyst:
a)Morecommoninanteriormediastinumthanmiddle
mediastinum
b)Mayhavemalignantpotential
c)Producesymptomswhichareusuallycompressiveinnature
d)Air-fluidmaybeseenonchestX-ray
e)Mostlyasymptomatic
CorrectAnswer-B:C:D:E
Answer-B,C,D,E,Mayhavemalignantpotential(C)Produce
symptomswhichareusuallycompressiveinnature(D)Air-fluid
maybeseenonchestX-ray(E)Mostlyasymptomatic
Bronchogeniccystsarethemostcommonprimarycystsofthe
anteriormediastinum.
ThediagnosisisconfirmedbyCTasasphericalfluid-ormucus
fillednonenhancingmass.Anairfluidlevelmaybepresentonchest
Xray.
Twothirdsofbronchogeniccystsareasymptomatic.
Malignantdegenerationhasbeenreported.

177.ApatienthasGlasgowcomascalescore
-E1VTM2.Whatisnotrequiredfor
his/hermanagement:

a)Headendelevation
b)Totalparenteralnutrition
c)Stressulcerprophylaxis
d)Centrallineplacement
e)None
CorrectAnswer-E
Answer-None
Domain
Response
Score
Spontaneous
4
Tospeech
3
Eyeopenng
Topain
2
None
1
Oriented
5
Confused
4
Bestverbalresponse Inappropriate
3
Incomprehensible
2
None
1
Obeying
5
Localizing
4
Withdrawal
Bestmotorresponse
3
Flexing
4
Extending
1
None

6
5
Deepcomeordeath
4
Totalscore
Fullyalertandoriented 5
3
1

178.Trueaboutpyomyositis-
a)Staphylococcusaureusisthemostcommoncausative
organism
b)Streptococcuspyogensisthemostcommoncausative
organism
c)Quadricepsisoneofthemostcommonlyaffectedmuscle
d)Treatmentinvolvesdrainageofabscesswithoutantibiotic
coverage
e)BestdetectedbyMRI
CorrectAnswer-A:C:E
Answer-A,C,E,Staphylococcusaureusisthemostcommon
causativeorganism(C)Quadricepsisoneofthemost
commonlyaffectedmuscle(E)BestdetectedbyMRI
Pyomyositisisanacutebacterialinfectionoftheskeletalmuscle
causedbyStaphylococcusaureus.
Itmaybeprimaryorsecondarytoapenetratinginjuryorcontiguous
anatomicinfection.
Ittypicallyaffectsthemajormusclesofthelowerextremityandthe
glutealmuscles.
Patientsusuallypresentswithpain,tendernessandedemaofthe
involvedmusclegroup.
Abscesscandevelopinthemusclegroups.Diagnosisisconfirmed
byneedleaspirationoroperativeincisionanddrainage.
Treatmentinvolvesadequatedrainagealongwithantibiotictherapy
againstStaphylococcusaureus.
Pyomyositisischaracterizedbyalocalizedinfectiousprocess,
conventionalpurulence,lackofsurroundingtissuenecrosis


179.Allofthefollowingaretrueabout
prostatespecificantigenexcept-
a)Normallevelis10-14ng/ml
b)ItmayElavatedinBPHandCaProstate
c)Itisproducedbyprostate
d)Itisaglycoprotein
e)None
CorrectAnswer-A
Answer-A.Normallevelis10-14ng/ml
Itisaglycoproteinproducedonlyintheprostaticcells(bothbenign
&malignant).
Normalserumlevellessthan4ng/ml
4-10ng/ml>thisrangeiscommonforbothBHPandCa.
Morethan10ng/mlapprox75%willhavecancer.

180.Feature(s)ofPattersonKellysyndrome:
a)Dysphagia
b)Morecommoninmalethanfemale
c)Achlorhydria
d)Treatmentconsistsofdilationofoesophagusbyoesophageal
bougies
e)All
CorrectAnswer-A:C:D
Answer-A,C,D,Dysphagia(C)Achlorhydria(D)Treatment
consistsofdilationofoesophagusbyoesophagealbougies
Classicalfeaturesofthissyndromeincludedysphagia,irondeficieny
anaemia,glossitis,angularstomatitis,koilonychia(spooningofnails)
andachlorhydria.
Affectsfemalespast40years
Dilatatianofthewebbedareabyoesophagealbougies

181.Allarefeature(s)ofBeckwidth-Wideman
syndromeexcept:
a)Omphalocele
b)Macrosomia
c)Hyperglycemia
d)Visceromegaly
e)None
CorrectAnswer-C
Answer-C.Hyperglycemia
Beckwidth-Wiedemannsyndromeisanovergrowthsyndromethatis
characterizedbyvisceromegaly,macroglossia,
macrosomia,mphaloceleandhyperinsulinemichypoglycemia.

182.Correctstatementaboutelectrocautery-
a)Inmonopolarmodeoneelectrodeusedatsurgicalfieldand
secondelectrodeisattachedtopatientplate
b)Patientplateshouldhaveconductivejellytoensureproper
contactwithbody
c)Burnmaybeseenatsiteofpatientplate
d)Cuttinghasmorevoltagethancoagulation
e)None
CorrectAnswer-A:B:C:D
Answer-A,B,C,D,Inmonopolarmodeoneelectrodeusedat
surgicalfieldandsecondelectrodeisattachedtopatientplate
(B)Patientplateshouldhaveconductivejellytoensureproper
contactwithbody(C)Burnmaybeseenatsiteofpatientplate
(D)Cuttinghasmorevoltagethancoagulation
High-frequancyalternatingcurrentcanbedeliveredineitherunipolar
orbipolarfashion.
Theunipolar(ormonopolar)deviceiscomposedofagenerator,an
electodeforapplication,andanelectrodeforthereturningcurrentto
completethecircuit.
Complicationsofdiathermy:Electrocution,Explosionanl,Burns.

183.Mostcommonpost-operatives
complaintsofpatientsis/are:
a)Pain
b)Nausea
c)Vomiting
d)Shivering
e)Sedation
CorrectAnswer-A:B:C
Answer-A,B,C,Pain(B)Nausea(C)Vomiting
Nausea,vomitingandpainaremostcommonpost-opcomplications.

184.Trueaboutabdominalaorticaneurysm-
a)Mostcommonbelowrenalartery
b)Maypresentsaspulsatileabdominalmass
c)Atherosclerosisismostcommoncause
d)Forasymptomaticaneurysmsrepairisindicatedifthediameter
is>4cm
e)None
CorrectAnswer-A:B:C
Answer-A,B,C,Mostcommonbelowrenalartery,(B)May
presentsaspulsatileabdominalmass(C)Atherosclerosisis
mostcommoncause
9O%ofabdominalaorticaneurysm(AAA)ofsize>4cmindiameter
isduetoatherosclerosis.
Malearemorefrequentlyaffectedthanfemale.
Theaneurysmmostcommonlyarisesbelowthelevelofrenalartery.
Itisusuallydetectedonroutineexaminationasapalpable,pulsatile,
expansile,andnontendermass.

185.Trueaboutconstitutionalgrowthdelay-
a)Baselinegrowthhormonedecreased
b)IGF-flevelsislowforchronologicalage
c)Growthdelayonlyoccursafter2-3yearsofage
d)Pubertyspurtisdelayed
e)Finalheightiswithinnormallimits
CorrectAnswer-B:D:E
Ans.(b)IGF-Ilevelsislowforchronologicalage,(d)Puberty
spurtisdelayed,(e)Finalheightiswithinnormallimits
ConstitutionalGrowthDelay
Thesechildrenarebornwithanormallengthandweightandgrow
normallyforfirst6-12monthsoflife.
Theirgrowththenshowsadecelerationsothattheheightand
weightfallbelowthe3rdcentile.
By3yrofage,normalheightvelocitylsresumedandtheycontinue
togrowjustbelowandparalleltothe3rdcentilewithanormalheight
velocity.
Theonsetofpubertyandadolescentgrowthspurtlsalsodelayedin
thesechildrenbutfinalheightiswithinnormallimits.
Boneageislowerthanchronologicalageandcorrespondstothe
heightage.
Historyofdelayedpubertyanddelayedheightspurtisusually
presentinoneorbothparents.
IGF-1levelstendtobelowerchronologicalagebutwithinthenormal
rangeforboneage
Pubertalgrowthspurtisdelayed.
Growthhormoneresponsestoprovocativetestingtendtobelower

thaninchildren

186.Featureofpallidbreathholdingspellin
comparisontocyanoticbreathholding
spell:

a)Morecommonthancyanoticbreathholdingspell
b)Elicitedbypainfulstimulus
c)Bradycardiaisprominent
d)Atropineisgiveninrefractorycases
e)None
CorrectAnswer-B:C:D
Ans.(b)Elicitedbypainfulstimulus.(c)Bradycardiais
prominent,(d)Atropineisgiveninrefractorycases
Breathholdingspells
Breathholdingspellisaparoxysmaleventoccuringin0.1%-5%of
healthychildrenfromtheageof6monthsto6years.
Thenameforthisbehaviourmaybemisnomerinthatitconnotes
prolongedinspiration.Infact,breath-holdingoccursduringexpiration
andisreflexive(notvolitional)innature.
Therearetwomajortypesofbreathholdingspells?
1.Cyanoticform(morecommon):
Temporarydisappearanceoradecreaseinintensityofthesystolic
murmurisusualasflowacrosstherightventricularoutflowtract
diminishes.
Paroxysmalhypercyanoticattacks(hypoxic,"blue,"or"tet"spells)
areaparticularproblemduringthe1st2yearsoflife.
Theinfantbecomeshyperpneaandrestless,cyanosisincreases,
gaspingrespirationsensue,andsyncopemayfollow.

Mostfrequentlyinmorningoninitiallyawakeningorafterepisodesof
vigorouscrying
2.Pallidform:
Triggeredbysuddenfrightorpainorfallingwithaminorinjurytothe
head
Childmaygaspandgiveabriefcry
Childbecomespale,losesconsciousnessandbecomeslimp
Childmaybecomesweatyandmaystiffenandhaveafewbody
jerksorlosebladdercontrol.
Treatment:
Asubgroupofinfantswithbreathholdingspellshaveirondeficiency
anemia.Irontherapymaytreatnotonlytheanemia,butalsothe
breath-holdingspells.
Pallidinfantilesyncopemayrespondtoatropinesulfate,whichis
usedonanongoingbasisifspellsarefrequent,orintermittentlyif
spellsaresituationallypredictable(suchaswithvenepuncture).

187.Whichofthefollowingfindingisnormal
ininfant?
a)Papilledemaisrareinraisedintracranialpressure
b)Floppyinfant
c)Strokingpatellartendonofonesideleadstocontractionon
oppositeside
d)Elbowcrossmidlineifpassivelydonebyexaminer
e)Parachutereflex
CorrectAnswer-A:C:E
Ans.(a)Papilledemaisrareinraised,(c)Strokingpatellar
tendonofonesideleadstocontractiononoppositeside(e)
Parachutereflex
Inincreasedintracranialtension(ICT),thereisseparationofthe
cranialsutures,widefontanelsandincreasedheadcircumference.
TheMacewen'sorcrackpotsignindicatesraisedintracranial
pressureaftersuturesandfontanelhaveclosed.
Papilledemaisunusualininfantunlesstheincreaseinintracranial
pressureisveryrapid.
Botulismcausesacuteflaccidparalysisfloppyinfantwhichisnot
anormalfinding.
Thekneejerkinaninfantmayproduceacrossedadductor
response(tappingthepatellartendoninonelegcausescontraction
intheoppositeextremity),which,ifpresent,doesnotbecome
abnormaluntil6-7moofage.
Whentheupperextremityofanormalterminfantispulledgently
acrossthechest,theelbownormallydoesnotquitereachthe

midsternum(scarfsign).Theelbowofahypotoniainfantextends
beyondthemidlinewithease"
Theparachutereflexisdemonstratedbysuspendingthechildbythe
trunkandbysuddenlyproducingforwardflexionasifthechildwere
tofall.Thechildspontaneouslyextendstheupperextremitiesasa
protectivemechanism.Theparachutereflexappearsbeforethe
onsetofwalking.

188.Whichvaccineisrecommendedat2
yearsofageaccordingtolatestIAP
guidelines:

a)MMR
b)Pneumococcalconjugate
c)VaricellaRubella
d)BoosterofTyphoidConjugateVaccine
e)IPVbooster
CorrectAnswer-D
Ans.(d)BoosterofTyphoidConjugateVaccine
Recommendedageatwhichthevaccinesshouldbereceived
andtypeofvaccine:

AGE
VACCINE
AtBirth
HepatitisB
DTaP-Diphtheria,Tetanus,Acellular
Pertussis
IVP-InactivatedPoliovaccine
2months
HepatitisB
Pneumococcalvaccine
HIB-HaemophilusinfluenzaTypeB
Rotavirusvaccine
DTaP
IVP
4months
Pneumococcalvaccine
HIB
Rotavirusvaccine

DTaP
IVP
HepatitisB
6months
Pneumococcalvaccine
HIB
Influenzavaccine**
Rotavirusvaccine
MMR-Measles,Mumps,Rubella
12months
Pneumococcalvaccine
HepatitisA
DTaP
15months
HIB
Varicella
18months
HepatitisA
2years
BoosterofTyphoidConjugateVaccine
DTaP
MMR
4to6yearsofage
IVP
Varicella
Tdap
11yearsofageto
Meningococcalvaccine
adult
HPV(humanpapillomavaccine)

189.Achildispresentingfeaturesofrickets
includingchangesonbonesandhas
hypophosphatemia.Whichofthe
followingistrue:

a)ItiscommonlycausedbyXlinkedrecessivedisorder
b)Normalzoneofprovisionalcalcificationadjacenttothe
metaphysisispresent
c)Thereisdefectofmineralizationofmatrix
d)CRFmaybethecause
e)Renaltubuledysfunctionleadstohypophosphatemia
CorrectAnswer-C:E
Ans.(c)Thereisdefectofmineralizationofmatrix,(e)Renal
tubuledysfunctionleadstohypophosphatemia
X-linked.hypophosphatemicrickets(XLH)inheritedindominant
manner(notrecessive)
Radiologicalchangesarecharacteristicallyseenatmetaphysis.
Thefirstchangeislossofnormalzoneofprovisionalcalcification
adjacenttothemetaphysis.
Rickets,adiseaseofgrowingbone,occursinchildrenonlybefore
fusionoftheepiphyses,andisduetounmineralizedmatrixatthe
growthplates.
RADIOGRAPHICFINDINGS:
Thickeningandwideningofepiphysis
Cuppingandfrayingofmetaphysis



Irregularmetaphysealmargins
Flaringofanteriorendsofribs
Ricketicrosary
Bowingofdiaphysis


190.Whichofthefollowingis/aretrueabout
atrialseptaldefect(ASD):
a)Ostiumprimumismostcommontype
b)Surgeryusuallydonebefore3year
c)Secondheartsound-Wideandfixed
d)Softdelayeddiastolicrumbleatleftlowerleftsternalborder
e)None
CorrectAnswer-B:C:D
Ans.(b)Surgeryusuallydonebefore3year,(c)Secondheart
sound-Wideandfixed,(d)Softdelayeddiastolicrumble
ClinicalmanifestationsofASD
PatientswithASDaregenerallyasymptomatic.
Mildeffortintoleranceandrespiratorytractinfectionmayoccur.
CHFisrare.
Physicalexamination
Parasternalimpulse
Systolicthrillat2ndleftinterspace.
AccentuationofS,duetoloudtricuspidcomponent.
WidesplitandfixedS2.
Ejectionsystolicmurmuratthesecondandthirdleftinterspaces.
Delayeddiastolicmurmuratthelowerleftsternalborder.
ASDwithmitralstenosisLutembachersyndrome.
Chestx-rayinASD
Mildtomoderatecardiomegalyarterysegment.
Prominentpulmonary
Rightatrialandrightventricularenlargement.

Relativelysmallaorticshadow
Plethoriclungfields.

191.RiskfactorforNeuraltubedefectis/are:
a)Diabeticmother
b)MTHFRmutation
c)Antiepilepticdrugintake
d)Methotrexateintake
e)All
CorrectAnswer-E
Ans.E.AllA,DiabeticmotherB,MTHFR
mutationC,AntiepilepticdrugintakeD,Methotrexateintake
ETIOLOGY
:
Teratogens-(hyperthermia,sulphas,antihistaminic,nutrition
deficienciesandanticonvulsantsuse)
Moststronglytied=carbamazepine,valproicacid(folateantagon)
Folatedeficiency

192.Trueaboutbreastmilkjaundiceis/are:
a)Appearsafterweek
b)Typicallybilirubinlevelisaround10-20ng/d1
c)Phototherapyisuseful
d)Managedconservatively
e)Diaperstainingispresentingfeature
CorrectAnswer-A:B:C:D
Ans.(a)Appearsafter1week,(b)Typicallybilirubinlevelis
around10-20ng/d1(c)Phototherapyisuseful,(d)Managed
conservatively

BREASTMILKJAUNDICE:
Occurslaterinnewbornperiod,withbilirubinlevelpeakingin6thto
14thdays.
First,atbirth,thegutissterile,andnormalgutfloratakestimeto
establish.
Breastmilkcontains:
GlucuronidaseIncreasedeconjugationandenterohepatic
recirculationofbilirubin.
Highepidermalgrowthfactor(EGF)IncreaseBilirubinuptakein
thegut(enterohepaticcirculation)
Second,breast-milkofsomewomencontains3-alpha-20-beta
pregnanediol.
Itinhibitsuridinediphosphoglucuronicacid(UDPGA)glucuronyl
transferaseresponsibleforconjugationandsubsequentexcretionof
bilirubin.
Inthenewbornliver,activityofglucuronyltransferaseisonlyat0.1-
1%ofadultlevels,soconjugationofbilirubinisalreadyreduced.

Third,lipoproteinlipaseinbreastmilkproducesincreasedFFAthat
inhibithepaticglucuronyltransferase,whichdecreasesconjugation
ofbilirubin.
TREATMENT:
Phototherapy:
Anynewbornwithatotalserumbilirubingreaterthan359mol/l(21
mg/dL)shouldreceivephototherapy

193.Highriskinfantare:
a)Birthorder>3
b)Twinning
c)Birthweight<3kg
d)Failuretogainweightfor3consecutivemonths
e)Artificialfeeding
CorrectAnswer-B:C:E
Ans.(b)Twinning,(d)Failuretogainweightfor3consecutive
months,(e)Artificialfeeding
Highriskinfantrisk
Birthweightlessthan2.5kg
Twins
Birthorder5ormore
Artificialfeeding
Weightbelow70%ofexpectedweight(i.eIIandIIIdegreeof
malnutrition)
Failuretogainweightduringthreesuccessivemonths
ChildrenwithPEM,diarrhoeaWorkingmother/oneparent

194.Trueaboutcephalohematoma:
a)Maximumatbirththenregress
b)Occursduetoforcepinjurytoperiosteum
c)Edematousswellingofsofttissue
d)Localizedcollectionofbloodbelowperiosteum
e)Mayextendacrossthemidlineandacrosssuturelines
CorrectAnswer-B:D
Ans.(b)Occursduetoforcepinjurytoperiosteum,(d)
Localizedcollectionofbloodbelowperiosteum
Cephalohematoma:

Itiscausedbyinjurytotheperiosteumoftheskullduringlaborand
delivery.
Thisleadstodevelopmentofhemorrhageoveroneorbothparietal
boneswithpalpableedgesappreciatedasthebloodreachesthe
limitsoftheperiosteum.
Itisacollectionofbloodlnb/wthepericraniumand.theflat
bone(subperiosteal)oftheskullusuallyunilateraland,overa
parietalbone

195.Trueaboutnewborns:
a)Apgarscoreprovideanimmediateestimateofthephysical
conditionofthebaby
b)APGARscoringisdoneatlmin
c)APGARscoringat5minhasnoprognosticvalue
d)NormalrespiratoryrateisRRis30-60breaths/min
e)Normalheartrate>100beats/min
CorrectAnswer-A:B:D:E
Ans.a.Apgarscoreprovideanimmediateestimateofthe
physicalconditionofthebaby;b.APGARscoringisdoneatl
min;d.NormalrespiratoryrateisRRis30-60breaths/min;e.
Normalheartrate>100beats/min
APGAR:
Ababy'sfirsttest
Quickassessmentofthenewborn'soverallwell-being
Givenone-minuteafterbirthandfiveminutesafterbirth
Rates5vitalareas
APGARat1min,indicatorsforneonatalresuscitation
LatertimesAPGARscore(after5minutes)indicatesaboutlong
termneurologicaldamage(notneonatalmortality)
Signs
0
1
2
Heartbeatsper
Absent
Slow(<100)
>100
minute
Respiratoryeffort
Absent
Slow,irregular
Good,crying
Someflexionof
Muscletone
Limp
Activemotion
extremities
No
Reflexirritability
Cryorcough

No
Reflexirritability
response Grimace
Cryorcough
Blueor
Bodypink,extremities Completely
Color
pale
blue
pink

196.Maximumrisktofoetusoccurswhen
maternalinfectionwithrubellaoccurs
duringwhichofthefollowingtime:

a)6-12week
b)12-l8week
c)14-20week
d)20-24week
e)32-36week
CorrectAnswer-A
Ans.(a)6-12week
Ingeneral,theearlierinpregnancyinfectionoccurs,thegreaterthe
damagetothefetus.Maximumdamagetothefetusoccurswhen
infectionisacquiredinthefirsttrimesterofpregnancy.
Duringacuterubellainpregnancy,therateofcongenitalinfectionis
over90%inthe12firstweeksofpregnancy,approximately60%in
weeks13to17,25%inweeks18to24andthenincreasesagain
duringthelastmonthofpregnancy

197.Trueaboutparacetamoltoxicityin
children:
a)HypernatremiaiscommonsideeffectofN-acetylcysteine
b)Liverfailuremayoccurafter3-4days
c)N-acetylcysteineisverylesseffectiveifgivenafter24hourof
paracetamolingestion
d)N-acetylcysteinebegivenorallyorIV
e)Renaldamagealsomayoccur
CorrectAnswer-B:C:D:E
Ans.b.Liverfailuremayoccurafter3-4days;c.N-
acetylcysteineisverylesseffectiveifgivenafter24hourof
paracetamolingestion;d.N-acetylcysteinebegivenorallyorIV;
e.Renaldamagealsomayoccur

Acetaminophenintoxicationlsacommoncauseofacuteliverfailure
inadolescentsandadults.
Acetaminophentoxicityresultsfromtheformationofahighly
reactiveintermediatemetabolite,N-acetyl-p-benzoquenoneimine
(NAPQI).
Theacutetoxicdoseofacetaminophenisgenerallyconsideredto
be>200mg/kglnchildrenyoungerthan12yrofage,asingle
ingestionof>7.5gisconsideredaminimumtoxicdosein
adolescentsandadults.
Adolescentshaveahigherincidenceoftoxicplasmaconcentration
afteringestionthandochildren,andtheirexposuresareoften
associatedwithintentionaloverdose.
AnIVpreparationofNACLsalsoavailable

N-acetylcysteinecancausenausea&vomitinganddiarrhoeaor
constipation.
Rarely,itcancauserashes,fever,headache,drowsiness,lowblood
pressureandliverproblems.

198.Allaretrueaboutanteriorcruciate
ligamentexcept?
a)Commonlyoccursasaresultoftwistingforce
b)MaybeassociatedwithSegondfracture
c)Rarelyassociatedwithmeniscalinjury
d)Lachmantestishighlysensitivetestfortear
e)None
CorrectAnswer-C
Ans.(c)Rarelyassociatedwithmeniscalinjury


Symptoms:
SignsandsymptomsofanACLinjuryusuallyinclude:
Aloud"pop"ora"popping"sensationintheknee
Severepainandinabilitytocontinueactivity
Rapidswelling
Lossofrangeofmotion
Afeelingofinstabilityor"givingway"withweightbearing
Complications
Higherriskofdevelopingosteoarthritisintheknee.Arthritismay
occurevenifyouhavesurgerytoreconstructtheligament.
Segondfractureduetoavulsionattheanterolateralcapsular
attachment.SegondfractureshaveaveryhighassociationwithACL
tearsand,meniscalinjuries
Diagnosis:
Injurytotearofanteriorcruciateligamentcanbedetectedusing
lachmanandanteriordrawertest.
Lachmanntestisasimilartesttoanteriordrawertestinwhich

anteriorglideofthetibiaisjudgedwiththekneein10-15degreesof
flexion.

199.PiraniscoringofCTEVincludesall
except?
a)Curvatureofthemedialborderofthefoot
b)Severityofthemedialcrease
c)Positionofthelateralpartoftheheadofthetalus
d)Emptinessoftheheel
e)Severityoftheposteriorcrease
CorrectAnswer-A
Ans.(a)Curvatureofthemedialborderofthefoot
PIRANISCORING
ThePiraniscoreisasimple,easytousetoolforassessingthe
severityofeachofthecomponentsofaclubfoot.
PiraniScoring:
Thecomponentsarescoredasfollows:
Eachcomponentmayscore0,0.5or1
Hindfootcontracturescore(HCFS):
Midfootcontracturescore(MFCS):
1. Posteriorcrease
2. Emptyheel
3. Rigidequinus
Midfootcontracturescore(MFCS):
1. Medialcrease
2. Curvatureoflateralborder
3. Positionofheadoftalus



200.TrueabouttuberculosisofSpine:
a)Middlepathregimenisusedinmanagement
b)Posteriorelementsofthespineismostcommonlyaffected
c)Commonlyspreadbyhematogenousroutefromlung
d)Acuteonsetparaplegiahasworseprognosis
e)Lowerthoracicandupperlumbarismostcommonsite
CorrectAnswer-A:C:E
Ans.a.Middlepathregimenisusedinmanagement;C.
Commonlyspreadbyhematogenousroutefromlunge.Lower
thoracicandupperlumbarismostcommonsite
Route:
Lymphogenousandhematogenousspreadhasbeenimplicated.in
thoracolumbarlesions.
Site:
UpperthoracicspinelsthemostcommonsiteofspinalTBin
children,thelowerthoracicandupperlumbarvertebraeareusually
affectedinadults
Paradiscalisthecommonesttype.
Acuteonsetparaplegiahasabetterprognosis
Management:
1. Rest,
2. DrugsIntensivePhase:HRO(5-6months);ContinuationPhase:HZ
(3-4months)+HR(4-5months);ProphylacticPhase:HE(4-5
months),
3. RadiologicalFollow-up(X-ray,MRI),d.GradualMobilisation+/-
Spinalbraces.managementofAbscess/Sinuses,
4. ManagementofNeurologicalcomplications

5. Surgery(ExcisionalsurgeryDefinitiveSurgery:IndicationofSurgery
inPott'sspine),
6. Post-operativecare

201.Whichofthefollowingis/areindication
ofsurgicalmanagementoffractureof
shaftofhumerus:

a)Fractureinelderly
b)Radialnerveinvolvementaftermanipulation
c)Pathologicalfractures
d)Vascularinjury
e)Multiplefractures
CorrectAnswer-B:C:D:E
Ans.(b)Radialnerveinvolvementaftermanipulation,(c)
Pathologicalfractures,(d)Vascularinjury,(e)Multiplefractures
FracturedShaftofHumerus:OperativeTreatment-Indications:
Severemultipleinjures:
Anopenfracture.
Segmentalfractures.
Displacedintra-articularextensionofthefracture
Apathologicalfracture.
Afloatingelbow(simultaneousunstablehumeralandforearm
fractures)
Radialnervepalsyaftermanipulation.
Non-union
Problemswithnursingcareinadependentperson

202.Whichofthefollowingis/aretrueabout
Ewingsarcoma:
a)Vascularorigin
b)Ewing'ssarcomaissecondmostcommonprimarymalignant
bonetumourinchildrenafterOsteosarcoma
c)Metaphysisoflongboneismostcommonsite
d)Feverandweightlossmaybepresent
e)Surgeryisveryusefulinmanagement
CorrectAnswer-B:D
Ans.b.Ewing'ssarcomaissecondmostcommonprimary
malignantbonetumourinchildrenandadolescentafter
Osteosarcoma;d.Feverandweightlossmaybepresent
EwingSarcoma:
Ewing'ssarcomaarisesfromprimitiveneuroectoderm.
MostcommonSite:Femurdiaphysis>tibiadiaphysis
Clinicalfeatures:
Occursbetween10-20yearsofage.
Thepatientpresentswithpainandswelling.
Historyoftraumaprecedingonset,butitisusuallyincidental.
Oftenthereisanassociatedfever,inwhichcaseitmaybeconfused
withosteomyelitis.
Treatment
Thisisahighlyradio-sensitivetumourmeltsquicklybutrecurs.
Treatmentconsistsofcontroloflocaltumourbyradiotherapy,and
controlofmetastasisbychemotherapy.
Themostcommonprimarymalignantbone

tumorsareosteosarcoma(35%),chondrosarcoma(25%),and
Ewing'ssarcoma(16%).
Lessfrequently(5%)occurringtumorsarechordoma,malignant
fibroushistiocytomaofbone,andfibrosarcomaofbone.
Themostcommonmalignantpediatricbonetumors
includeosteosarcomaandEwingsarcoma.

203.Trueaboutgiantcellsarcoma?
a)Mostcommonagegroupaffectedis20-40year
b)Proximalfemurismostcommonsiteaffected
c)Pulmonarymetastasisoccurin<3%ofcases
d)Alocallyaggressivetumor
e)Mayinvolvesacrum
CorrectAnswer-A:C:D:E
Ans.a.Mostcommonagegroupaffectedis20-40year;c.
Pulmonarymetastasisoccurin<3%ofcases;d.Alocally
aggressivetumor;e.Mayinvolvesacrum
GCT:
Itperhapsrepresentsthemostaggressivebenigntumorand
threatensthetruedefinitionofabenigncancerbecausebenign
pulmonarymetastasisdevelopinapproximately1%to2%ofgiant
celltumors.
CLINICALFEATURES
Thetumourisseencommonlyintheagegroupof20-40yearsi.e.,
afterepiphysealfusion.
Thebonesaffectedcommonlyarethosearoundthekneei.e.,lower-
endofthefemurandupper-endofthetibia.
Lower-endoftheradiusisanothercommonsite.
Thetumourislocatedattheepiphysis.
Itoftenreachesalmostuptothejointsurface.
Commonpresentingcomplaintsareswellingandvaguepain.
Sometimes,thepatient,unawareofthelesion,presentsforthefirst
timewithapathologicalfracturethroughthelesion.

204.Spursignis/areseenin:
a)Supracondylarfractureofhumerus
b)Radialheadfracture
c)Acetabulumfractureofpelvis
d)Talusfracture
e)None
CorrectAnswer-C
Ans.(c)Acetabulumfractureofpelvis
Spurcells-Theyareirregularlydistortedredcellscontainingseveral
irregularlydistributedthornlikeprojections.
Cellswiththismorphologicabnormalityarealsocalled
acanthocytes.
TheyareseeninSplenectomisedpatientsandpatientswithliver
disease.

205.Cause(s)ofavascularnecrosisof
femoralHead:
a)Fractureoffemoralneck
b)Steroiduse
c)alcoholuse
d)Sicklecelldisease
e)Caissondisease
CorrectAnswer-A:B:C:D:E
Answer:A,FractureoffemoralneckB,SteroiduseC,alcohol
useD,SicklecelldiseaseE,Caissondisease
Causeofavascularnecrosisoffemoralhead:
ldiopathic-commonest
Alcoholism
Steroidtherapy
Sicklecelldisease
Patientofrenaldialysis
Patientonanticancerdrug
Post-partumnecrosis
Goucher'sdisease
Caisson'sdisease

206.Whichofthefollowingarenotincluded
inGurd'scriteria?
a)Centralnervoussystemdepressiondisproportionateto
hypoxaemia
b)Tachycardia<1l0bpm
c)Deepveinthrombosis
d)Axillaryorsubconjunctivalpetechiae
e)HypoxaemiaPao2<60mHg,Fio2=0.4
CorrectAnswer-C
Ans.c.Deepveinthrombosis

207.Correctstatementabouthandinfection?
a)OpeningofFelonbyfishmonthincisionispreferredincision
technique
b)Felonismiddlevolarpulpinfection
c)Apicalsubungualinfection-V-shapedpieceisremovedfromthe
centerofthefreeedgeofthenailalongwithalittlewedgeofthe
fullthicknessoftheskinoverlyingtheabscess
d)Whenthepusextendsbeneaththenail,itisnecessaryto
removethesomepartofnailforadequatedrainageofpus
e)None
CorrectAnswer-C:D
Ans.c.Apicalsubungualinfection-V-shapedpieceisremoved
fromthecenterofthefreeedgeofthenailalongwithalittle
wedgeofthefullthicknessoftheskinoverlyingtheabscess;
d.Whenthepusextendsbeneaththenail,itisnecessaryto
removethesomepartofnailforadequatedrainageofpus
Inapicalsubungualinfection:Fordrainage,asmallVshapedpiece
lsremovedfromthecentreofthatfreeedgeofthenailalongwitha
littlewedgeofthefullthicknessoftheskinoverlyingtheabscess.
Inacuteparonychia:Whenthepusextendsbeneaththenail,itis
necessarytoremovetheproximalone-thirdofthenailforadequate
drainage.
DONOTperforma"fishmouth"incisionsincethismayresultsin:
Unstablefingerpad,neuroma.,and/orlossofsensation"
Thefelonshouldbeincisedintheareaofmaximumswellingand
tenderness.

208.Trueaboutanatomyofvagina:
a)Coveredbycolumnarepithelium
b)Coveredbynon-keratinizedstratifiedsquamousepithelium
c)Vaginalsecretionisfromtransudationofvaginalepithelium
d)Suppliedbycervicovaginalbranchoftheuterineartery
e)Anteriorwallislongerthanposteriorwall
CorrectAnswer-B:C:D
Ans.b.Coveredbynon-keratinizedstratifiedsquamous
epithelium;c.Vaginalsecretionisfromtransudationofvaginal
epithelium;d.Suppliedbycervicovaginalbranchoftheuterine
artery
HISTOPATHOLOGY:
Thevaginaiscomposedof4histologicallayers(internalto
external):
Nonkeratinizedstratifiedsquamousepithelium
Elasticlaminapropria
Fibromuscularlayer
Adventitia
BLOODVESSELANDNERVESUPPLIES:
Arterialsupplytothevaginaisviatheuterineandvaginalarteries;
bothbranchesoftheinternaliliacartery.
Venousreturn:vaginalvenousplexus,whichdrainsintotheinternal
iliacveinsviatheuterinevein.
Lymphaticdrainage:iliacandsuperficialinguinallymphnodes.
Theparasympatheticandsympatheticnervessupplyingthevagina
arederivedfromtheuterovaginalnerveplexus.Theuterovaginal
plexusliesinthebaseofthebroadligament,eithersideofthe

supravaginalpartofthecervix.
Inferiorfibresfromtheuterovaginalplexussupplythesuperiorpart
ofthevagina.Thesearederivedfromtheinferiorhypogastric
plexusandthepelvicsplanchnicnerves.
Theinferiorpartofthevaginaisinnervatedbyabranchof
thepudendalnervecalledthedeepperinealnerve.

209.Whichoffollowingis/arenotfertility
awarenessbasedmethods:
a)Withdrawalmethod
b)Rhythmmethod
c)Cervicalmucusmethod
d)MTPpill
e)Sympto-thermalmethod
CorrectAnswer-A:D
Ans.(a)Withdrawalmethod,(d)MTPpill
Allfamilyplanningmethodsthatattempttoidentifyfertiletimein
eachcycleandthenmodifysexualbehaviourarecalledFertility
AwarenessBasedMethods(FAB)
Theyareasfollows:
Rhythmmethod/calendarmethod
Basalbodytemperaturemethod
Cervicalmucusmethod/Billingmethod.
Sympto-Thermalmethod
Standarddaysmethodusingcyclebeads
Twodaymethod
Ovulationdetection
Coitusinterruptus
Lactationamenorrhoeamethod(LAM)
Abstinence

210.Predisposingfactor(s)forobstetrical
haemorrhageis/are:
a)Obesity
b)Placentalabruption
c)Oligohydramnios
d)Smoking
e)Instrumentaldelivery
CorrectAnswer-A:B:D:E
Ans.(a)Obesity,(b)Placentalabruption,(d)Smoking,(e)
Instrumentaldelivery
Predisposingfactor(s)forobstetricalhaemorrhage:
AbnormalPlacentation
Placentaprevia
Placentalabruption
Placentaaccreta/increta/percreta
Ectopicpregnancy
Hydatidiformmole
TraumaDuringLaborandDelivery
Episiotomy
Complicatedvaginaldelivery
Low-ormidforcepsdelivery
Cesareandeliveryorhysterectomy
Uterinerupture-riskincreasedby:
Previouslyscarreduterus
Highparity
Hyperstimulation

Obstructedlabor
Intrauterinemanipulation
Midforcepsrotation
SmallMaternalBloodVolume
Smallwomen
Pregnancyhypervolemianotyetmaximal
Pregnancyhypervolemiaconstricted
Severepreeclampsia
Eclampsia
OtherFactors
Obesity
NativeAmericanethnicity
PreviousPPH
UterineAtony
Overdistendeduterus
Largefetus
Multiplefetuses
Hydramnios
Distentionwithclots
Anesthesiaoranalgesia
Halogenatedagents
Conductionanalgesiawithhypotension
Exhaustedmyometrium
Rapidlabor
Prolongedlabor
Oxytocinorprostaglandinstimulation
Chorioamnionitis
Previousuterineatony
CoagulationDefects-lntensifyOtherCauses
Placentalabruption
Prolongedretentionofdeadfetus
Amnioticfluidembolism
Saline-inducedabortion
Sepsissyndrome
Severeintravascularhemolysis
Massivetransfusions
Severepreeclampsiaandeclampsia

Congenitalcoagulopathies
Anticoagulanttreatment

211.Allaretrueaboutpuerperalsepsis
except:
a)Temperature>380C(100.4oF)
b)Caesareansectionhasnoincreasedriskforsepsis
c)GroupAbeta-hemolyticstreptococcusisoneofcommon
causativeorganism
d)Instrumentdeliveryincreasesrisk
e)Retainedplacentaisacause
CorrectAnswer-B
Ans.(b)Caesareansectionhasnoincreasedriskforsepsis
Puerperalpyrexia--isdefinedasariseoftemperaturereaching
100.4?F(38?C)ormore(measuredorally)on2separateoccasions
at24hoursapart(excludingfirst24hours)withinfirst10days
followingdelivery.
Anyinfectionofgenitaltractwhichoccursasacomplicationof
deliveryiscalledasPuerperalsepsis.
MostcommonsiteofPuerperalinfection--Placentalsite.
MostcommonmanifestationofPuerperalinfection--Endometritis.
MostcommoncauseofPuerperalsepsis--Streptococcus.
Mostcommonrouteofinfection--Directspread.
Caesareansectioniseasilythemostcommonidentifiableriskfactor
fordevelopmentofpuerperalinfection.

212.Truestatementregardinginvestigationin
endometrialcancer:
a)MRIissuperiortoCTindetectingmyometrialinvolvement
b)CTissuperiortoMRIindetectingomentalmetastasis
c)USGisinitialinvestigationtobeperformed
d)USGisthebestinvestigation
e)None
CorrectAnswer-A:B:C
Ans.a.MRIissuperiortoCTindetectingmyometrial
involvement;b.CTissuperiortoMRIindetectingomental
metastasisandc.USGisinitialinvestigationtobeperformed
DiagnosisofEndometrialCarcinoma
CTscanofpelvisandabdomenmaybeusedtodetectlymphnode
metastases".
MRIcandetectMyocardialinvasion
SensitivityofPETindetectingpelvicnodemetastasesis80%
comparedtoMRI(70%)andCT(48%)"
"CTisusefulinthediagnosisoflymphnodemetastasisanddepthof
myometrialinvasioninendometrialcancer"
"MRIissuperiortoCTorultrasoundindiagnosingadenomyosis,
myomasandendometrialcancer(includingmyometrialinvasion)

213.TrueaboutDelivery,ofHIV+vewoman-
a)VaginaldeliveryhaslowerriskfortransmissionofHIVtochild
thanCS
b)VaginaldeliveryhashigherriskfortransmissionofHIVtochild
thanCS
c)Instrumentationhasnoriskofincreasedinfection
d)Verticaltransmissionislessincaseswithpretermbirth
e)NONE
CorrectAnswer-B
Ans.(b)VaginaldeliveryhashigherriskfortransmissionofHIV
tochildthanCS
Vaginalandemergencycaesareansectiondeliveries,prematurity,
andlowCD4cellcountweremoststronglyassociatedwithinfants
infectionstatusinunivariateanalyses.
Childrendeliveredvaginallyorbyemergencycaesareansection
weremorelikelytobeinfectedthanthosedeliveredbyelective
caesareansection,withareductioninriskof79%associatedwith
thelatter(P<0.001).
Similarly,infantsdeliveredbefore37weeksweremorethantwiceas
likelytobeinfectedthaninfantswhowerenotpremature.
Caesareansectionbeforeonsetoflabourandruptureofmembranes
approximatelyhalvestheriskofmother-to-childtransmission.
Transmissionrate:
Duringpregnancy:5?10%
Duringlabouranddelivery:10?15%
Duringbreastfeeding:5?20%
Overallwithoutbreastfeeding:15?25%

Overallwithbreastfeedingtosixmonths:20?35%
Overallwithbreastfeedingto18?24months:30?45%

214.Whichofthefollowingis/aretrueabout
combinedoralcontraceptivepills:
a)Reducesriskofvenousthromboembolism
b)Reducesriskofbenignbreastdisease
c)Protectsagainstendometrialcancer
d)DecreasedBonedensity
e)None
CorrectAnswer-B:C
Ans.(b)Reducesriskofbenignbreastdisease,(c)Protects
againstendometrialcancer
OCPS
ADVANTAGES:
Controlsfertility
TreatsMenorrhagia&polymenorrhoea.
Relievedysmenorrhoeaandpremenstrualtension
Preventsanaemia
Lowerschancesof
Fibrocysticdisease
Ovariancyst
Ovarian,uterine&anorectalmalignancy
PID
Ectopicpregnancy
Usefulinacne,PCODandendometriosis
PreventRA
NoncontraceptivebenefitsofOCPs:
Cyclestabilization

Cureofmenstrualdisorder-usefulinmenorrhagia&polymenorrhea
Preventsanemia.
Reducestheincidenceofectopicpregnancy.
Protectionagainstcancer?Ovarian,Endometrial
Benigntumour-Benignbreastdisease,Ovarianfunctionalcyst,
Fibromyomauterus
Protects-PID,Anemia,Endometriosis,PCOD,Acne,hirsutism,
Rheumatoidarthritis,Osteoporosis

215.TrueaboutamniocentesisandChorionic
villussampling:
a)In1sttrimester,amniocentesisisbetterthanChorionicvillus
sampling
b)2ndtrimesteramniocentesishaslessfoetallossthanChorionic
villussampling
c)Amniocentesismayresultinoligohydraminos
d)Amniocentesisinlsttrimesterhaslowerriskoffoetallossthan
2ndtrimesteramniocentesis
e)Amniocentesisinlsttrimesterhasmoreriskoftalipes
CorrectAnswer-B:C:E
Ans.b.2ndtrimesteramniocentesishaslessfoetallossthan
Chorionicvillussampling,c.Amniocentesismayresultin
oligohydraminos&e.Amniocentesisinlsttrimesterhasmore
riskoftalipes
AdvantagesofCVSoveramniocentesis:
ThemainadvantageofCVSisthat,resultsareavailableearlierin
pregnancy.whichlessenparentalanxietywhenresultsarenormal.
Allowsearlierandsafermethodsofpregnancyterminationwhen
resultsareabnormal.
CVSRisks:
Chancesoffetalloss/abortion.
Ifperformedearlierthan9weeks(typicallyaround7weeks),
increasedchancesoforomandibularhypogenesisandlimb
reductiondefects.
Itcancauseruptureofmembranes.leakageofamnioticfluidand

infection.
RhisoimmunizationcanoccurinRhnegativefemales.
ComplicationsofAmniocentesis:
Chorioamnionitis(0.1%)
Procedurerelatedfetalloss
Leakageofamnioticfluidoccursinabout2%patients,butisusually
selfresolving.Persistentleakageofamnioticfluidcausing
oligohydramnios
Talipesequinovarus
Vaginalbleeding(2-3%)
Rhisoimmunization
Pretermlabor
Respiratorydistress
Intrauterinedeath(IUD)

216.Duringexternalradiationtherapyfor
cervicalcancerwhichlymphnodeis/are
excluded?

a)Externaliliaclymphnode
b)Commoniliaclymphnode
c)Internaliliaclymphnode
d)Sacrallymphnode
e)Obturatorlymphnode
CorrectAnswer-C
Ans.(c)Internaliliaclymphnode
Thegoalofexternalirradiationincervicalcanceristosterilize
metastaticdiseasetopelviclymphnodesandtheparametriaand/or
todecreasethesizeofthecervixtoallowoptimalplacementof
intracavitaryradioactivesources.
Patientswithknownorsuspectedmetastaticdiseasetoperiaortic
lymphnodesmaybeconsideredforextendedfieldirradiation.
LymphaticSpreadlncervicalcancer:
Thecervixisdrainedbypreureteral,Postureteral,anduterosacral
lymphaticchannels.
Thefollowingareconsideredfirststationnodes:obturatorexternal
lilac,hypogastric,parametrial,presacral,andcommoniliac.
Para-aorticnodesaresecondstation,arerarelyinvolvedinthe
absenceofprimarynodaldisease,andareconsideredmetastases.

217.Whichofthefollowingistrueabout
vulvodynia:
a)Surgeryisusuallydoneforlocalizedvulvallesion
b)Painwithoutanysignificantlesion
c)Maybeassociatedwithirritablebowelsyndrome
d)Tricyclicantidepressantisuseful
e)Psychologicalfactorisassociated
CorrectAnswer-B:C:D:E
Ans.(b)Painwithoutanysignificantlesion,(c)Maybe
associatedwithirritablebowelsyndrome,(d)Tricyclic
antidepressantisuseful,(e)Psychologicalfactorisassociated
Vulvodyniaischronicpainofthevulva(externalfemalegenitalia)in
theabsenceoflocalizedinfection.Itisoftenassociatedwithirritable
bowelsyndrome(IBS)
Aetiology:
Swellingoforinjurytothenervesofthevulva.
Spasmsorweaknessofthemusclesthatsupporttheorgansofthe
pelvis.
Afamilyhistoryofvulvodynia.
Symptoms:
Painisthemainsymptomofvulvodynia.Dependingonthe
person,thepainmay:
Befeltonlyinonespot,suchasneartheopeningofthevagina,and
onlywhensomethingtouchesthatarea.Thisiscalledlocalized
vulvodynia.
Painmaybefeltonoraroundmostofthevulva,evenwhennothing

touchesthoseareas.Thisiscalledgeneralizedvulvodynia.
Beconstantorcomeandgoformonthsorevenyears.
Bemildorverybad.
Befeltduringandaftersex.
Flareupwhenyousitonabicycle,putinatampon,orwipeyour
vulva.
Othersymptomsmayinclude:
Burningorstinging.
Itching.
Swelling.
Throbbing.
Rawness.
Treatment:
Medicines:
Physicaltherapy
Behaviourtherapy
Psychosexualcounselling
Oestrogencreams
Lidocainejelly
Surgeryiscontraindicated
TricyclicAntidepressants(amitriptyline,gabapentin)
Seizuremedicines
Nerveblocks
Medicatedcreams
Antihistaminescanhelprelieveitching.

218.Indicationsforin-vitrofertilization(IVF):
a)Bilateraltubeblockage
b)Normalmalefactor
c)Hostilecervicalfactor
d)Proximaltubalblock
e)Prematureovarianinsufficiency
CorrectAnswer-A:D:E
Ans.(a)Bilateraltubeblockage,(d)Proximaltubalblock,(e)
Prematureovarianinsufficiency
Indicationsofin-vitroFertilization(lVF)
Mildendometriosis
Blockedfallopiantubesorfailedtubalsurgery
Failedintrauterineorfallopianinsemination
Immunologicalfactor
lnmaleandfemale
Abnormalsemenfinding.
Idiopathicorunexplainedmaleorfemaleinfertility
Donorsemenorsperm

219.Whichofthefollowingis/aretrueabout
managementofpregnancywithNYHA
class3or4heartdisease:

a)Deliveryshouldbedoneinspecialisedhospitals
b)Oftentoleratemajorsurgicalprocedurespoorly
c)Caesareandeliveryislimitedtoobstetricalindications
d)Mortalityis5-20%
e)Epiduralispreferredfordelivery&labourpainmanagement
CorrectAnswer-A:B:C:D:E
Ans.A,Deliveryshouldbedone...B,Oftentoleratemajor
surgical...C,Caesareandeliveryislimited
...D,Mortality...E,Epiduralispreferredfordelivery...
NewYorkHeartAssociation(NYHA)classification:
NYHAClassI:Asymptomatic(mortality0-1%)
NYHAClassII:Symptomswithgreaterthannormalactivity(
mortality5-15%)
NYHAClassIII:Symptomswithregularactivity(mortality25-50%)
NYHAClassIV:Symptomsatrest(mortality25-50%)
ManagementofNYHAclassIII&IV
Generalmanagement:Multidisplinaryteamapproach
Placeoftherapeutictermination
AdmissionforgradeIIIandIV:Throughoutpregnancy.
CardiacindicationsofCS:
Coarctationofaorta
Aorticdissectionoraneurysm,
Aortopathywithaorticroot>4cm

Warfarintreatmentwithin2weeks
Anaesthesia:GAorepidural(preferred)
Vaginaldeliveryispreferredinmostcases,andlaborinductioncan
usuallybed.onesafely
Inabilitytotoleratemajorsurgicalprocedures.

220.Whichofthefollowingischangesduring
pregnancy:
a)Inlasttrimesterbloodvolumeincreaseby50%
b)Cardiacoutputincreaseby20%inlasttrimester
c)HemodynamicchangesinpregnancycancauseCHFfollowing
duringlabourandfollowingdeliveryinpre-existingcardiac
lesions
d)Hypercoagulabilityoccurs
e)None
CorrectAnswer-A:C:D
Ans.(A)Inlasttrimesterbloodvolumeincreaseby50%;(C)
HemodynamicchangesinpregnancycancauseCHFfollowing
duringlabourandfollowingdeliveryinpre-existingcardiac
lesions;(D)Hypercoagulabilityoccurs
HEMATOLOGICALCHANGES:

PARAMETERS
CHANGES
Bloodvolume(mL)
by1500(30?40%)
Plasmavolume(mL)
by1250(40?50%)
RedCellvolume(mL)
by350(20?30%)
TotalHb(g)
by85(18?20%)
SerumIron
TIBC
Hematocrit
Diminished
Erythropoietin
(Neutrophilicleukocytosis-8,000to
WBCcount

WBCcount
20,000/mm3)
Plateletcountand
unchanged
volume
VII,VIII,X,plasmafibrinogen:
Coagulationfactors
AntithrombinIII,XI,XIII:
ESR
Plasmafibrinolytic
activity
PlasmaProtein
(Albumin30%;Globulin;A:G)
Concentration
CARDIOVASCULARCHANGES:
Cardiacoutput
by40%(maximumat30thweek)
(L/min)
Strokevolume
by27%
(mL)
Heartrate
by17%
(perminute)
Unaffectedormid-pregnancydropof
Bloodpressure
diastolicpressureby5?10mmHg
Venous
100%
pressure
Colloidoncotic
pressure
by14%
(mmHg)
Systemic
vascular
by21%
resistance
(SVR)
Pulmonary
vascular
by34%
reistance(PVR)
Totalextracellularvolume by16%
Shiftofapicalimpulselaterallyandupwardsinthe
left4'hintercostalspace


221.TruestatementaboutBreechdelivery:
a)Vasapreviaisacomplication
b)Fetalcongenitalmalformationincreasesbreechrisk
c)Increasesfetalandmaternalmorbidity
d)Oligohydramniosincreasesbreechrisk
e)Increasesriskofhipjointdislocationofbaby
CorrectAnswer-B:C:D:E
Ans.(b)Fetalcongenitalmalformationincreasesbreechrisk(c)
Increasesfetalandmaternalmorbidity(d)Oligohydramnios
increasesbreechrisk(e)lncreasesriskofhipjointdislocation
ofbaby
ETIOLOGY
:
Prematurity
Factorspreventingspontaneousversion:
Breechwithextendedlegs
Twins
Oligohydramnios
Septateorbicornuateuterus
Shortcord,relativeorabsolute
IUDoffetus.
Favourableadaptation:
Hydrocephalus
Placentaprevia
Contractedpelvis
Cornu-fundalattachmentoftheplacenta
Unduemobilityofthefetus
Hydramnios,

Multiparawithlaxabdominalwall.
Fetalabnormality:Trisomies13,18,21,anencephalyandmyotonic
dystrophy
BIRTHINJURIESASSOCIATEDWITHBREECHDELIVERY
COMPLICATIONS
Braindamage
Spinalcordinjury
Fetaldistress
Umbilicalcordprolapse
Seizures
Cerebralpalsy
Compressedumbilicalcord
Nervedamage
Umbilicalcordwrappedaroundbaby'sneck
Oxygendeprivation

222.Complicationsofshoulderdystocia?
a)Humerusfracture
b)Brachialplexusinjury
c)Birthasphyxia
d)Sacroiliacjointdislocationofmother
e)All
CorrectAnswer-E
Ans.E.All
Complicationofshoulderdystocia:
Maternal:
PPH
Rectovaginalfistula
Symphysealseparationordiathesis,withorwithouttransient
femoralneuropathy
3rdor4thdegreeepisiotomyortear
Uterinerupture
Fetal:
Brachialplexuspalsy
Claviclefracture
Fetaldeath
Fetalhypoxia,withorwithoutpermanentneurologicdamage
Fractureofthehumerus

223.Whichofthefollowingistrueabout
monozygotictwinformation:
a)Ifdivisionoccursafterembryonicdiscformation,itresultsin
conjointtwin
b)Ifdivisionoccurbefore72hrs,itresultsinformationof
diamniotic-dichorionictwins
c)Ifdivisionoccursb/w4-8days,itresultsinformationof
monochorionicmonoamniotictwin
d)Ifdivisionoccursafter8days-itresultsinformationof
monochorionicmonoamniotictwin
e)None
CorrectAnswer-A:B:C:D
Ans.a.Ifdivisionoccursafterembryonicdiscformation,it
resultsinconjointtwinb.Ifdivisionoccurbefore72hrs,it
resultsinformationofdiamniotic-dichorionictwinsd.If
divisionoccursafter8days-itresultsinformationof
monochorionicmonoamniotictwin
DEVELOPMENT:
Ifthedivisiontakesplacewithin72hoursafterfertilizationthe
resultingembryoswillhavetwoseparateplacenta,chorionsand
amnions(D/D)
Ifthedivisiontakesplacebetweenthe4thand8thdayafterthe
formationofinnercellmasswhenchorionhasalreadydeveloped
diamnioticmonochorionictwinsdevelop(D/M)
Ifthedivisionafter8thdayoffertilization,whentheamnioticcavity
hasalreadyformed,amonoamnioticmonochorionictwinsdevelop


(M/M)

224.Acantholysisis/arenotseenin:
a)Lichenplanus
b)Bullouspemphigoid
c)Dermatitisherpetiformis
d)Hailey-Haileydisease
e)Pemphigusvulgaris
CorrectAnswer-A:B:C
Ans.(A)Lichenplanus(B)Bullouspemphigoid(C)Dermatitis
herpetiformis
Acantholysis:
Separationofepidermalcellsfromeachother.
AcantholyticdisordersincludesPemphigusfamily(including
paraneoplasticpemphigus),eosinophilicspongiosis,Darier's
disease,Hailey-Hailey'sdisease(Familialbenignchronic
pemphigus)andtransientacantholyticdermatosis(Grouer's
disease),aswellasspecifichistologicalpatternssuchasfocal
acantholyticdyskeratosisandepidermolytichyperkeratosis.

225.Cutaneousmarker(s)ofinternal
malignancyis/areallexcept:
a)Tripepalms
b)SignofLeser-Trelat
c)Dermatomyositis
d)Migratorythrombophlebitis
e)Seborrheicpatchatback
CorrectAnswer-E
Ans.E.Seborrheicpatchatback
CutaneousMarkersofInternalMalignancies:
Causes:
Metastases:Toskin.
Genodermatoses:Withanincreasedpredispositiontointernal
neoplasia.
Exposuretocarcinogens:Whichresultinskinchangesaswellas
internalneoplasia.
Paraneoplasticsyndromes:Arecutaneousreactionpatterns
associatedwithinternalneoplasia.

226.Nikolsky'ssignis/areseeninallexcept:
a)Pemphigus
b)Hailey-Haileydisease
c)Staphylococcalscaldedskinsyndrome
d)Toxicepidermalnecrolysis
e)Groverdisease
CorrectAnswer-D:E
Ans.(D)Toxicepidermalnecrolysis
Nikotsky'ssign:
Applicationoftangentialpressureonnormalskin(usuallyonpretibial
area)resultsinformationofnewbulla.
Seenin:
Staphylococcalscalded-skinsyndrome
Epidermalnecrolysis.
Pemphigus
Stevens-Johnsonsyndrome.
Grover'sdisease

227.Trueaboutacuteparonychia:
a)Pusundernailbed
b)Pusmayextendtobaseofnail
c)Swellingofnailfold
d)Candidaismostcommoncausativeorganism
e)None
CorrectAnswer-A:B:C
Ans.A,PusundernailbedB,Pusmayextendtobaseofnail&
C,Swellingofnailfold
AcuteParonychia:
Paronychia:
Inflammationofnailfolds.
Etiology:StaphyLococcusenterthenailfold
Clinicalfeature:Nailfoldisswollen,redandtender.Pusvisible
undernailfold/nailbed.

228.CnrnnonentofAdvancedcardiovascular
lifesupport(ACLS)inaccordanceto
AHA2015guideline:

a)Chestcompression100-150perminute
b)Chestcompressionatleast5cm/2inch
c)VasopressorsisusedtomaintainMAP>70mmHginnon-
responsivetofluids
d)1Breathevery8seconds
e)Vasopressinisusedasvasopressor
CorrectAnswer-B
Ans.B.Chestcompressionatleast5cm/2inch
Basiclifesupport(BLS),advancedcardiovascularlifesupport
(ACLS),andpost-cardiacarrestcarealldescribeasetofskillsand
knowledgeappliedsequentiallyduringthetreatmentofpatientswho
haveacardiacarrest.
ACLScomprisesthelevelofcarebetweenBLSandpost-cardiac
arrestcare
Updaterecommendationsforadvancedcardiaclifesupport
2015:
Thecombinedusevasopressinandepinephrineoffersnoadvantage
tousingstandard-doseepinephrineincardiacarrest.
VasopressinhasbeenremovedfromtheAdultCardiacArrest
Algorithm-2015update.
AdvancedCardiacLifeSupport:
Continuouschestcompressionsatarateoflil)/rninto120/min,
vnthoutpausesforventilation.Theproviderdeliveringventilation

shouldprovide1breathevery6seconds(10breathsperminute).
Itmaybereasonabletoavoidandimmediate$rcorrecthypotension
(SBp<90mmHg,MAp<65mmHg)duringpost-cardiacarrest
care.

229.Anaestheticagents(s)having
epileptogenicpotential:
a)Atracurium
b)Etomidate
c)Enflurane
d)Pethidine
e)Propofol
CorrectAnswer-A:C:D
Ans.(A)Atracurium(C)Enflurane(D)Pethidine
Etomidate:
Doesnothaveepileptogenicpotential.
Enflurane:Athighdosesitproducesspikeandwavepatternin
EEGwhichculminatesintofranktonic-clonicseizure.
Atracurium:Itsmetabolicproductlaudanosine(Laudanosine
Toxicity)-seizuresprecipitated.
Ketamlnecanelicitseizuresinpatientswithanepilepticdiathesis.
Propofol:Significantanticonvulsantactivity.

230.Trueaboutxenonisare:
a)Environmentfriendly
b)Cheap
c)Lowbloodsolubility
d)Inert
e)Stable
CorrectAnswer-A:C:D:E
Ans.(A)Environmentfriendly(C)Lowbloodsolubility(D)Inert
(E)Stable
Advantagesanddisadvantagesofxenon(Xe)anesthesia:
Advantages:
Inert(probablynontoxicwithnometabolism).
Minimalcardiovasculareffects.
LowbloodsolubilitY.
Rapidinductionandrecovery
Doesnottriggermalignanthyperthermia
Environmentalfriendly.
Nonexplosive
Disadvantages:
Highcost
Lowpotency{MAC=70%)

231.Ingastubing,rateofturbulentflow
dependsupon:
a)Viscosityofgas
b)Pressuregradient
c)Lengthoftube
d)Radiusoftube
e)Densityofgas
CorrectAnswer-B:E
Ans.(B)Pressuregradient(E)Densityofgas
Turbulent
Turbulentflowisproducedifflowrateisveryhighorifgaspasses
throughbends,constrictions.
Flowisrough.
Reynold'snumbermustexceedto2000forturbulence.
Turbulentflowismoredependondensity

232.Gasstoredinliquidstateincylinders:
a)Nitrogen
b)Helium
c)CO2
d)Cyclopropane
e)Nitrousoxide
CorrectAnswer-C:D:E
Ans,(C)CO2(D)Cyclopropane(E)Nitrousoxide
Oxygen,nitrogen,airandheliumarestoredincylindersasgases.
Nitrousoxide,carbondioxideandcyclopropanearestoredinas
liquidinequilibriumwithsaturatedvapour.
ColourofCylinders:
O2-Blackbodywithwhiteshoulder
N2O-Blue
CO2-Grey
Cyclopropane-orange
Helium-Brown
Air-Greybodywithblackandwhiteshoulders
Entonox-Bluebodywithblueandwhiteshoulders(50%O2.+50%
N2O).

233.Trueaboutcaudalanesthesiain
children:
a)Averagedistancefromtheskintotheanteriorwallofthesacral
canalis21mm
b)0.5mL/kgdoseofbupivacaineissufficientforlumberand
sacraldermatomesblock
c)Beyond6-7yearsofage,itisdifficulttogiveandisless
successfulincomparisontoyoungerchildren
d)2-3cmofepiduralcatheterisadvancesthroughepiduralspace
incontinuosinfusion
e)Distancefromtheupperborderofthesacralhiatustothedural
sacis30?10.4mm
CorrectAnswer-A:C:D:E
Ans.(A)Averagedistancefromtheskintotheanteriorwallof
thesacralcanalis21mm(C)Beyond6-7yearsofage,itis
difficulttogiveandislesssuccessfulincomparisonto
youngerchildren(D)2-3cmofepiduralcatheterisadvances
throughepiduralspaceincontinuosinfusion(E)Distancefrom
theupperborderofthesacralhiatustotheduralsacis
30?10.4mm
CaudalAnesthesia
Normallengthofcathetertobeintroducedintotheepiduralspacels
2to3cm,asforanyepiduralblock.
Dosageprescriptionscheme:
1. With0.5mL/kg,allsacraldermatomesareblocked.
2. With1.0ml/kgallsacralandlumbardermatomesareblocked.

3. With1.25ml/kg,theupperlimitofanesthesiaisatleastmidthoracic.
Drugused:Thedoseof0.25%bupivacainets0.5-O.75ml/kg
Extraduralspacebelowsacralhiatusmayrangefrombeingdeepto
excessivelyshallow-itsaveragelengthls10-15cm.
ItsanatomyismoreeasilyappreciatedininfantsandchilDren
Indications:
Useforpattants<8yearsoldtoprovideintraoperativeand
postoperativeanalgesiaforabdominalandlowerextremitysurgery.
Technique:
Advanceneedleandcatheter2to4mm.

234.Chestroentgenogramfeature(s)of
stage-2SarcoidosisinScaddingscoring
system:

a)Bilateralhilarlymphadenopathy
b)Mediastinallymphadenopathy
c)Upperlobeparenchymalinfiltrates
d)Enlargedparatrachealnodes
e)Pulmonaryfibrosis
CorrectAnswer-A:C
Ans.A,Bilateralhilarlymphadenopathy&C,Upperlobe
parenchymalinfiltrates
StandardscoringsystemdescribedbyScaddingin1961forchest
roentgenograms.
Stage1-Hilaradenopathyalone,oftenwithrightparatracheal
involvement.
Stage2-Combinationofadenopathyplusinfiltrates.
BHLandparenchymalinfiltrates.
Patientsmaypresentwithbreathlessnessorcough.
Themajorityofcasesresolvespontaneously.
Stage3revealsinfiltratesalone.
Stage4consistsoffibrosis.
Usuallytheinfiltratesinsarcoidosisarepredominantlyanupperlobe
process.
Onlyinafewnoninfectiousdiseasesisanupperlobepredominance
noted.

235.Standardtreatmentofwhole-brain
radiotherapy(WBRT)forbrain
Metastasis:

a)20grays(Gy)in10fractions
b)30grays(Gy)in10fractions
c)30grays(Gy)in5fractions
d)15grays(Gy)in10fractions
e)None
CorrectAnswer-B
Ans.B,30grays(Gy)in10fractions
Whole-brainradiotherapy(WBRT)to30grays(Gy)in10fractions-
Standardtreatmentinpatientswithmultiplebrainmetastases.
Currentstudyinvestigatedthepotentialbenefitofdoseescalation
beyond30Gy.

236.Defencemechanisminobsessive-
compulsivedisorder(OCD)is/are:
a)Undoing
b)Reactionformation
c)Suppression
d)Isolationofaffect
e)Projection
CorrectAnswer-A:B:D
Ans.A,UndoingB,Reactionformation&D,Isolationofaffect
DefenseMechanism:
SynopsisofPsychiatrybyKaplanandSadock11th/160
Displacement-Phobio(Especiallyinchildren)&OCD.
Reactionformation
Undoing
Inhibition
Isolation
Dissociation

237.Beck'scognitivetriadofdepression
includes:
a)Self
b)Future
c)Pastexperience
d)Worldandenvironment
e)Others
CorrectAnswer-A:B:D
Ans.A,SelfB,Future&D,Worldandenvironment
AaronBeckpostulatedacognitivetriadofdepression.
Consistsof,
Viewsabouttlwself-anegativeselfprecept.
Aboutenvironment-atendencytoexperiencetheworldashostile
and.demanding.
Aboutfuture-expectationofsufferingandfailure.
Therapyconsistsofmodifyingthesedistortions.
Cognitivetriad:Beliefsaboutoneself,theworld,andthefuture.

238.Whichisnotabrainstimulation
technique:
a)Electroconvulsivetherapy
b)Magneticseizuretherapy
c)Deepbrainstimulation
d)Rapidtranscranialmagneticstimulation
e)Cognitivetherapy
CorrectAnswer-E
Ans.E.Cognitivetherapy
SynopsisofPsychiatrybyKaplonandSadockp1065-81
BrainStimulationTechnique:
Electricalcurrentsormagneticfieldstoalterneuronalfiring.
Transcranialtechniquesinclude:
Cranialelectricalstimulation(CES)
Electroconvulsivetherapy(ECT)
Transcranialdirectcurrentstimulation(IDCS,alsodirectcurrent
polarization)
Transcranialmagneticstimulation(TMS)
Magneticseizuretherapy(MST)
Surgicaltechniques:
Corticalbrainstimulation(CBS).
Deepbrainstimulation(DBS)
Vagusnervestimulation(VNS).

239.Feature(s)ofdeliriumtremensis/are:
a)Mostcommoncomplicationinalcoholwithdrawal
b)Auditoryhallucinationmayoccur
c)Visualhallucinationmaybepresent
d)Predominantlylow-voltagefastactivityonEEG
e)Mostseverealcoholwithdrawalsyndrome
CorrectAnswer-B:C:D:E
Ans.B,AuditoryhallucinationmayoccurC,Visualhallucination
maybepresentD,Predominantlylow-voltagefastactivityon
EEG&E,Mostseverealcoholwithdrawalsyndrome
Deliriumtremens:
Predominantlylow-voltagefastactivity.
Mostseverealcoholwithdrawalsyndrome.
Visual(andauditory)hallucinations.
Classicsignofalcoholwithdrawalistremulousness,

240.Personalitytypeassociatedwith
coronaryarterydisease:
a)A
b)B
c)C
d)D
e)E
CorrectAnswer-A
Ans.A.A
Psychosocialfactors,includingtypeApersonality,anger,hostility,
andanxiety,havebeenimplicatedinthepathogenesisof
cardiovasculardisease.
invalidquestionid

This post was last modified on 11 August 2021